You are on page 1of 475

Learn. Integrate. Apply.

An Integrative Question-Based USMLE Resource

USMLE Step 1 NBME Top Concepts

Rahul Damania, MD
DEDICATION
To my students, mentors, and family. Without
your support, insight, and guidance, our mission
of inspiring the next generation of physicians
would not be possible.

Designed by:
Ray Ann S. Sampil

The United States Medical Licensing Examination (USMLE®) is a joint program of the Federation of State
Medical Boards (FSMB®) and National Board of Medical Examiners (NBME®). None of the trademark
holders are affiliated with HyGuru.

In this edition, all images used are for the benefit of integrative student learning and for educational
purposes only. No copyright claims are intended. The images are collated based on three publicly accessed
resources: public domain, Creative common Attribution License
(www.creativecommons.org/licenses/by/4.0/legal code), Creative Commons Attribution-Share Alike license,
www.creativecommons.org/licenses/by-sa/4.0/legal code.

uploaded by medbooksvn
TABLE OF CONTENTS
CHAPTER 1 | CARDIAC PHYSIOLOGY 1

Cardiac Cycle 2
Isovolumetric Ventricular Contraction 4
Rapid Ventricular Ejection 5
Isovolumetric Ventricular Relaxation 6
Ventricular Filling 7
Atrial Systole 9
Jugular Venous Tracing 10
Pulsus Paradoxes 11
Pressure Volume Loops 12
Cardiac Action Potentials 14
Answer Key 17

CHAPTER 2 | NBME TOP CONCEPTS: CARDIOLOGY 18

Digeorge Syndrome 19
Branchial Pouch Derivatives 20
Lipid Lowering Drugs 21
Shock 24
Thermoregulation 27
S3,S4,HOCOM,DCM,Murmurs 28
Vasculitis 32
Post Mi Complications 33
Answer Key 35

CHAPTER 3 | HIGH YIELD IMAGES & PE FOR THE USMLE STEP 1: INTRODUCTION 37

USMLE Test Taking Strategy Approach to Multimedia Questions 38


Test Taking Strategy for Image Questions 39
Top Images You Will Encounter on the USMLE 40
Answer Key 52

CHAPTER 4 | HIGH YIELD IMAGES & PE FOR THE USMLE STEP 1: CARDIOLOGY 56

Murmurs & Maneuvers 57


Right Heart Failure Vs. Left Heart Failure 62
Transudative Vs. Exudative For The USMLE 63
Mechanism Of Edema 64
Turner Syndrome 65
Rapid Review Genetic Heart Lesions & Cardiac Anomalies 66
Genetic Conditions & Heart Lesions 67
Pathophysiology Of Tetralogy Of Fallout 68
Pathophysiology Of Tet Spells 70
Endocarditis 71
Microbiology Integration 72
Infective Endocarditis On USMLE 73
Rheumatic Fever 74
Erythema Marginatam 76
Answer Key 77
CHAPTER 5 | NBME TOP CONCEPTS: RESPIRATORY 79

A-A Gradient (Hypoxemia) 80


Regional Circulation For The USMLE 86
Physical Exam Mcqs (Resp) 90
Restrictive Vs. Obstructive Disease 93
Lung Tumors 97
Acute Respiratory Distress Syndrome 100
Answer Key 102

CHAPTER 6 | HIGH YIELD IMAGES & PE FOR THE USMLE STEP 1: RESPIRATORY 106

Overview 107
Pneumothorax 110
Pneumonia For The USMLE 114
Clubbing 120
Acute Respiratory Distress Syndrome 121
Answer Key 123

CHAPTER 7 | NBME TOP CONCEPTS: ENDOCRINOLOGY 125

Hormone Signaling 126


Thyroid Disorders 129
Hypothyroid 131
Hyperthyroid 133
Pth And Calcium 135
Men Syndromes 138
Islet Cell Tumors 140
Dka Vs. Hhs 141
Diabetes Pharmacology 144
Aldosterone Disorders 145
Answer Key 146

CHAPTER 8 | NBME TOP CONCEPTS: GASTROENTEROLOGY 148

Esophageal Issues 149


Acid Secretion In Stomach 153
Approach To Abdominal Pain On The USMLE 155
Hernias 157
Meckel's Diverticulum 158
Hirschsprung's Disease 160
Inflammatory Bowel Disease 161
Bile Acid Metabolism 164
Hepatitis B 165
Vesicular Steatosis 166
Answer Key 167

CHAPTER 9 | HIGH YIELD IMAGES & PE FOR THE USMLE STEP 1: GASTROENTEROLOGY 169

Oral Pathology 170


Esophageal Pathology 174
Stomach Pathology 178
Intestinal Pathology 182
Answer Key 183

uploaded by medbooksvn
CHAPTER 10 | NBME TOP CONCEPTS: HEMATOLOGY 185

Heme Synthesis 186


CYP Inducers 187
Lead Poisoning 188
Approach To The Blood Smear 189
Intro To Anemia 192
Microcytic Anemia 193
Acute Phase Reactants 195
Summary Of Microcytic 196
Macrocytic Anemia 197
B12 Physiology 199
Normocytic Anemia 200
HUS/TTP Polycythemia 203
Platelet Pathology 205
Warfarin Vs. Heparin 207
Multiple Myeloma 208
Summary & Courses 209
Answer Key 210

CHAPTER 11 | HIGH YIELD IMAGES & PE FOR THE USMLE STEP 1: HEMATOLOGY 213

Hematology Overview 214


Peripheral Blood Smear 215
RBC Inclusions 216
Sickle Cell Disease 219
Complications Of Asplenia 222
Summary Of RBC Inclusions 223
Mechanisms Of Extravascular RBC Hemolysis 224
Mechanisms Of Intravascular RBC Hemolysis 225
Normocytic Anemia 226
Aplastic Anemia 228
Cold And Warm AIHA 230
Hus/TTP 231
Cells Of The Immune System 232
Causes Of Eosinophilia On USMLE 233
Causes Of Plasma Cells On USMLE 234
Causes Of Mast Cells On USMLE 235
Answer Key 236

CHAPTER 12 | NBME TOP CONCEPTS: NEUROLOGY 238

Brain Hematomas 239


Herniation Syndromes & Cerebral Physiology 243
Cranial Nerve Path (1) 246
Bell's Palsy 249
Cranial Nerve Path 250
Multiple Sclerosis 251
Dementia 252
Dopamine Pathways 254
Trinucleotide Repeat Disorders 255
Neurocutaneous Syndromes 256
Brain Tumors 259
Stroke 261
Answer Key 263
CHAPTER 13 | HIGH YIELD IMAGES & PE FOR THE USMLE STEP 1: NEUROLOGY 266

Neuroanatomy 267
Trinucleotide Repeat Disorders 268
Pituitary Masses 269
Thalamic Stroke 270
Central Pontine Myelinolysis 271
Internal Capsule And Lacunar Strokes 272
Intracranial Tumours For The USMLE 274
Multiple Sclerosis 275
Cerebral Edema 276
Intracranial Pressure 277
Cerebral Autoregulation 278
Baroreceptor Reflex 279
Upper Motor Neuron Signs & Posturing 281
Spinal Cord Pathologies 282
Pseudotumor Cerebri 285
Hydrocephalus 286
CNS Infections 288
Answer Key 289

CHAPTER 14 | NBME TOP CONCEPTS: RENAL 291


Renal Casts 292
Kidney Stones 295
Urea Cycle + Stones (Pt.2) 298
Into To Nephritic/Nephrotic 301
Nephrotic Syndorme 302
Renal Failure 311
Diuretics 312
Answer Key 314

CHAPTER 15 | HIGH YIELD IMAGES & PE FOR THE USMLE STEP 1: RENAL 316

Renal Stones 317


Polycystic Kidneys 322
Nephrotic And Nephritic Syndromes 328
Tumors Of The Urinary Tract 337
Answer Key 339

CHAPTER 16 | NBME TOP CONCEPTS: REPRODUCTIVE 341

Disorders Of Sexual Development 342


Turner's Syndrome Integration 345
Testosterone Disorders 347
Mullerian Agenesis Vs. Ais 348
Summary Of Disorders Of Sexual Development 350
Polycystic Ovarian Syndrome 351
Ovarian Physiology 354
Ovarian Tumors 356
Repro Ligaments 360
Uterine Disorders 361
Cervical Disorders (HPV) 366
Answer Key 367

uploaded by medbooksvn
CHAPTER 17 | NBME TOP CONCEPTS: MUSCULOSKELETAL & RHEUMATOLOGY 370

NMJ Disorders 371


Bullous Diseases 373
Hemoptysis & Hematuria 375
Skin Cancers (With Neural Crest) 376
RA Vs. OA 382
Seronegative Spondyloarthritis 385
Lupus 388
Anti Body Rapid Fire (USMLE) 391
Answer Key 393

CHAPTER 18 | NBME TOP CONCEPTS: PSYCHIATRY 397

Mood Disorders 398


Bipolar + Li Pharm 399
Psychotic Disorders 404
Eating Disorders 408
Drugs Of Abuse (Toxicology) 411
Answer Key 414

CHAPTER 19 | NBME TOP CONCEPTS: ONCOLOGY 418

Cardiac Oncology 419


Endocrine Oncology 421
Gastrointestinal Oncology 423
Lymphoma & Vascular Tumors 428
Renal Oncology 431
Respiratory Oncology 435
Neuro Oncology 437
Breast Oncology 439
Neoplasia (Get Path Integration) 441
Answer Key 442

CHATER 20 | NBME TOP CONCEPTS: DERMATOLOGY 445

Approach To Urticaria 446


Dermatology Nomenclature 448
Plaques 450
Viral & Bacterial Exanthems 451
Rubella 453
Measles 454
Roseola 455
Erythema Infectiosum 456
Systematic Manifestations Of Herpes 458
Varicella Infections On USMLE 459
Erysipelas Vs. Cellulitis 461
Staph And Strep Infections 462
Nikolsky Sign Manifestations For The Usmle 463
Neonatal Rashes 464
Precursor Lesions To Neoplasia 465
Summary 466
Answer Key
CHAPTER 1

Cardiac Physiology
for the USMLE

1
uploaded by medbooksvn
NBME Top Concepts ➤ Cardiac Physiology Cardiac Cycle

Dr. Carl Wiggers

Dr. Carl Wiggers


• Earliest Descriptions published in 1915
• A comprehensive approach on learning the electrical & mechanical
activity of the heart.
• Forms the basis for understanding Cardiac Physiology for the
USMLE.

Creative Commons Attribution License 4.0

Mastering the Cardiac Cycle

Creative Commons Attribution License 4.0

STRATEGIES ON MASTERING THE CARDIAC CYCLE

Active Recall of Events


• Ask yourself ‘Where is the blood flow?’
• Isolate each curve in your mind understanding pressure and volume
• Electrical activity of the heart precedes mechanical activity of the heart!

Cardiac Physiology: Cardiac Cycle

2
NBME Top Concepts ➤ Cardiac Physiology Cardiac Cycle

Major Events and Curves of the Cardiac Cycle

Creative Commons Attribution License 4.0

Cardiac Physiology: Cardiac Cycle

3
uploaded by medbooksvn
NBME Top Concepts ➤ Cardiac Physiology Isovolumetric Ventricular Contraction

Isovolumetric Ventricular Contraction


• Isovolumetric Ventricular Contraction
– What is the heart doing?
• Ventricles contracting What does this say about pressure?
– Increases
– On the EKG, what wave does this correlate to?
• QRS complex
– What heart sound do you hear clinically at the beginning of this phase?
• 1st heart sound

Creative Commons Attribution License 4.0

Cardiac Physiology: Isovolumetric Ventricular Contraction

4
NBME Top Concepts ➤ Cardiac Physiology Rapid Ventricular Ejection

Rapid Ventricular Ejection


• Rapid Ventricular Ejection
– What is the heart doing?
• Ventricles contract (max pressure) blood is flowing into the aorta
• Aortic pressure is increasing
• What is happening with ventricular volume?
– Decreasing
– On the EKG, what wave does this correlate to?
• ST Segment
– Which valve is open during this time?
• Aortic Valve

Creative Commons Attribution License 4.0

Review Systolic Events of the Cardiac Cycle

Cardiac Physiology: Rapid Ventricular Ejection

5
uploaded by medbooksvn
NBME Top Concepts ➤ Cardiac Physiology Isovolumetric Ventricular Relaxation

Isovolumetric Ventricular relaxation


• Isovolumetric Ventricular Relaxation
– What is the heart doing?
• Ventricular pressure decreases
– What valve just closed during this phase?
• QRS complex
– What heart sound do you hear clinically?
• 2nd heart sound

Creative Commons Attribution License 4.0

Cardiac Physiology: Isovolumetric Ventricular Relaxation

6
NBME Top Concepts ➤ Cardiac Physiology Ventricular Filling

Rapid Ventricular Filling


• Rapid Ventricular Filling
– What is the heart doing?
• Ventricles are filling passively with blood
• Ventricular volume is increasing
• Ventricular pressure is low and constant to keep the passive flow going
– What valve is open?
• Mitral
– What heart sound do you hear clinically?
• 3rd heart sound

Reduced Ventricular Filling


• Reduced Ventricular Filling
– What is the heart doing?
• Ventricles are relaxed
– What valve is open?
• Mitral
• This is the longest phase of the cardiac cycle.
• A patient is undergoing an exercise stress test on a treadmill. What effect will this have on this
phase of the cardiac cycle?
– Decreased time available for ventricular filling.

Ventricular Filling

Creative Commons Attribution License 4.0

Cardiac Physiology: Ventricular Filling

7
uploaded by medbooksvn
NBME Top Concepts ➤ Cardiac Physiology Ventricular Filling

NBME Style Question

A middle-aged male presents with shortness of breath while lying flat. He is found to have a 3/6
holosystolic murmur heard best at the apex. Dilated cardiomyopathy is suspected. Which of the
following physical exam findings would be less likely to be associated with this presentation?

A. Apical impulses shifted to the axillary line at the sixth intercostal space.
B. S4 gallop.
C. Bibasilar crackles.
D. Peripheral edema.
E. Hepatomegaly.

Cardiac Physiology: Ventricular Filling

8
NBME Top Concepts ➤ Cardiac Physiology Atrial Systole

Atrial Systole
• Atrial Systole
– What is the heart doing?
• Atria contracting final phase of ventricular filling
– On the EKG, what wave does this correlate to?
• P wave and PR interval
– Clinically, what can you hear on cardiac exam?
• 4th heart sound

Creative Commons Attribution License 4.0

Review Diastolic Events of the Cardiac Cycle

Cardiac Physiology: Atrial Systole

9
uploaded by medbooksvn
NBME Top Concepts ➤ Cardiac Physiology Jugular Venous Tracing

Jugular Venous Tracing

Creative Commons Attribution License 4.0

NBME Style Question


A 30 year old male is hit in the chest with a baseball bat. He is tachycardic and he has shallow
respirations. On palpation of his abdomen, a prominent jugular vein is appreciated. Bedside echo
shows a collapsed atria on diastole. What is the most likely vital sign change which may be present
in this patient?

A.Decrease in diastolic blood pressure by 6 mmHg during exhalation.


B.Increase in diastolic blood pressure by 10 mmHg during inhalation.
C.Decrease in systolic blood pressure by 10 mmHg during exhalation.
D. Decrease in systolic blood pressure by 10 mmHg during inhalation.

Cardiac Physiology: Jugular Venous Tracing

10
NBME Top Concepts ➤ Cardiac Physiology Pulsus Paradoxes

Pulsus Paradoxes

Decrease in SBP by 10 mmHg during Inspiration

Cardiac Tamponade on the USMLE Step 1:

Recognize Triads
• Physical exam shows a prominent jugular vein.
• BP < 90/60
• Heart sounds are in-audible or muffled.

Creative Commons Attribution License 4.0

Cardiac Physiology: Pulsus Paradoxes

11
uploaded by medbooksvn
NBME Top Concepts ➤ Cardiac Physiology Pressure Volume Loops

Pressure Volume Loops


A patient has a rumbling diastolic murmur heard
at the apex. The snap of this murmur occurs at
which point?
• D.Mitral valve opening.

• Cardiac event:
• A. Mitral valve closure.
• B. Aortic valve open.
• C. Aortic valve close.
• D.Mitral valve opening.
Creative Commons Attribution License 4.0

Pressure Volume Loops for the USMLE

USMLE Vignettes relevant to this change:


• β1 mediated activity
• [Ca2+]
• PO4 phospholamban
• “A patient started on a medication for heart-
failure and suddenly develops vision changes and
increased [K]?”
• Digoxin
• Inhibits the Na+/K+ ATPase. Creative Commons Attribution License 4.0

• Makes Ca2+ more difficult to leave myocyte.


• contractility.

Changes in the ESPVR Changes in EDPVR

Creative Commons Attribution License 4.0

Cardiac Physiology: Pressure Volume Loops

12
NBME Top Concepts ➤ Cardiac Physiology Pressure Volume Loops

Pressure Volume Loops for the USMLE


USMLE Vignettes relevant to this change:
• Endi Diastolic Volume
• Venous Tone:
• Exercise
• USMLE Vignette:
• A patient who recently was placed on hemo-dialysis
for CKD.
The patient is noted to have hyperdynamic precordium.
Bounding pulses. Diagnosis? Creative Commons Attribution License 4.0

• A-V fistula = preload


• Passive leg raise

Summary of Effects

Positive Pressure Ventilation in the Cardiac Intensive Care Unit. J Am Coll Cardiol 2018;72:1532-1553.

Pressure Volume Loops for the USMLE


USMLE Vignettes relevant to this change:
• Afterload is largely dependent upon aortic pressure.
• blood pressure (essential hypertension)
• “A 50-year-old male who presents with passing out. He
has CP non-tender to palpation. Walk test notes dyspnea.
A murmur at the R-second intercostal space is noted.
What is the likely mechanism?
• Age related calcific stenosis Creative Commons Attribution License 4.0

• A-V fistula = preload

Cardiac Physiology: Pressure Volume Loops

13
uploaded by medbooksvn
NBME Top Concepts ➤ Cardiac Physiology Cardiac Action Potentials

Compare and Contrast Two Action Potentials

Creative Commons Attribution License 4.0

Cardiac Action Potential


In ventricular muscle, what ion determines the phase 4 of the cardiac action potential?
• Potassium permeability
• K equilibrium potential is -84 mV

What ion determines phase 0 of the ventricular action potential?


• Na influx

What phase of the ventricular action potential defines the difference between skeletal action
potential vs. ventricular action potential?
• Phase 2 Cardiac action potential has a plateau phase which
is due to Ca influx and K efflux.
• Other differences: Ca induced Ca release, and gap-junctions

Cardiac Physiology: Cardiac Action Potentials

14
NBME Top Concepts ➤ Cardiac Physiology Cardiac Action Potentials

Compare and Contrast Two Action Potentials

Creative Commons Attribution License 4.0

Cardiac Action Potential


A farmer is working in the field and suddenly experiences lacrimation, rhinorrhea, and diarrhea. He
begins to have sweating and bronchospasm. In the ED, his HR is found to be 45.
• An increase in which NT defines his symptoms?
• Ach (due to inhibition of AchE).
• What phase of the nodal action potential does his bradycardia
affect?
• Phase 4

Cardiac Physiology: Cardiac Action Potentials

15
uploaded by medbooksvn
NBME Top Concepts ➤ Cardiac Physiology Cardiac Action Potentials

Regional Circulation for the USMLE

Cardiac Physiology: Cardiac Action Potentials

16
NBME Top Concepts ➤ Cardiac Physiology Answer Key

Ventricular Filling

A middle-aged male presents with shortness of breath while lying flat. He is found to have a 3/6
holosystolic murmur heard best at the apex. Dilated cardiomyopathy is suspected. Which of the
following physical exam findings would be less likely to be associated with this presentation?

A. Apical impulses shifted to the axillary line at the sixth intercostal space
B. S4 gallop
C. Bibasilar crackles
D. Peripheral edema
E. Hepatomegaly

Jugular Venous Tracing

A 30-year-old male is hit in the chest with a baseball bat. He is tachycardic and he has shallow
respirations. On palpation of his abdomen, a prominent jugular vein is appreciated. Bedside echo
shows a collapsed atria on diastole. What is the most likely vital sign change which may be present
in this patient?

A.Decrease in diastolic blood pressure by 6 mmHg during exhalation.


B.Increase in diastolic blood pressure by 10 mmHg during inhalation.
C.Decrease in systolic blood pressure by 10 mmHg during exhalation.
D.Decrease in systolic blood pressure by 10 mmHg during inhalation.

Cardiac Physiology: Answer Key

17
uploaded by medbooksvn
CHAPTER 2

NBME Top Concepts:


Cardiology

18
NBME Top Concepts ➤ Cardiology DiGeorge Syndrome

NBME Style Question


A 2-months-old female presents with seizures. She is lethargic and hypertonic. The patient is also
found to have respiratory distress. The exam is notable for a cleft palate, and a murmur is heard on
the clinical exam. What is the likely embryological mechanism?

A. Defective neural crest migration.


B. Neural crest defect.
C. Failure of the 3rd and 4th arch to develop.
D. Failure of 3rd and 4th pouch to develop.
E. Failure of 3rd and 4th cleft to develop.

HyGuru Test-Taking Tip


RECOGNIZING MULTI-SYSTEM INVOLVEMENT

Whenever you see multiple organ systems involved in exam questions, think about a:
• Syndrome
• Systemic Condition

DiGeorge Syndrome:
• Recurrent infections Infectious Disease
• Hypocalcemia Endocrine.

Cardiology: DiGeorge Syndrome

19
uploaded by medbooksvn
NBME Top Concepts ➤ Cardiology Branchial Pouch Derivatives

Embryology of the Pharyngeal System

Creative Commons Attribution License 4.0

Pharyngeal Pouches

Creative Commons Attribution License 4.0

Cardiology: Branchial Pouch Derivatives

20
NBME Top Concepts ➤ Cardiology Lipid Lowering Drugs

NBME Style Question

A 40-year-old female with a history of Grave’s disease is noted to have intermittent chest pain
with activity. She has a history of elevated LDL. She takes PTU for hyperthyroidism. She is started on
a medication to control her dyslipidemia. Which of the following best explains the mechanism of
action of this medication?

A. Activation of HMG-CoA Synthase.


B. Upregulation of hepatocyte LDL-receptor.
C. Downregulation of bile-acid synthesis.
D. In-activation of reverse transport in cholesterol metabolism.
E. Upregulation of bile-acid metabolism.

Relevant Drugs

Creative Commons Attribution License 4.0

PPAR integration
PPAR-alpha:
• Upregulates LPL
• Increases TG clearance
• Upregulates HDL synthesis

Pharm correlate: fibrates


• PPAR gamma:
• Increases insulin sensitivity
• Increases adiponectin levels
• Pharm correlate: pioglitazone, rosiglitazone

Cardiology: Lipid Lowering Drugs

21
uploaded by medbooksvn
NBME Top Concepts ➤ Cardiology Lipid Lowering Drugs

Hepatotoxicity
Which of the anti-hyperlipidemics can be hepatotoxic?
Statins
Ezetimibe
Niacin
What are ways the test maker may as this on the exam?
Increases in AST and ALT.

Muscle Myopathy

Which of the anti-hyperlipidemics can cause myopathy?


Statins
Fibrates
What are ways the test maker may as this on the exam?
Increases in CPK.

Anti-Hyperlipidemics
An experimental trial is studying the modulation of receptor mediated endocytosis via clathrin
coated pits as it relates to lipid metabolism. Inhibition of receptor degradation via inactivation of the
endo-lysosomal construct is noted to decrease LDL receptors. What is the likely agent studied?
PCSK9 Inhibitors:
Alirocumab, Evolocumab.

PCSK9

Creative Commons Attribution License 4.0

Cardiology: Lipid Lowering Drugs

22
NBME Top Concepts ➤ Cardiology Lipid Lowering Drugs

USMLE Test Taking Strategy Experimental Questions


Think like the test-maker: Strategy:
Test makers want a student to: 1. Don’t panic
• Organize elements of an experiment. 2. Organize cohorts, groups, or data sets:
• Interpret relevant data. Ex: Treatment group vs. control
• Relate the experiment to the content you have Ex: Filling out ‘givens’
studied. 3. Relate to content studied.

Cardiology: Lipid Lowering Drugs

23
uploaded by medbooksvn
NBME Top Concepts ➤ Cardiology Shock

NBME Style Question

A 50-year-old male presents with severe chest pain while mowing his lawn. His vital signs:
110/min, 20 breaths/min, 105/70. He is cool and clammy. He is noted to have an elevated PCWP
and left-atrial pressure of 20. What are the most likely pressure changes in the pulmonary
vasculature seen?

Mastering Shock for the USMLE

Isolate the primary etiology of each shock. Understand the monitoring parameters:

Parameters to watch out for: CO: blood pumping out of the heart
SV x HR
Cardiac Output: TPR: resistance of the vessels

Blood coming out of the heart. Usually at the level of the pre-capillary arteriole
PCWP: pressure in the pulmonary artery
TPR: Can be a surrogate for left atrial pressure
LVEDV: blood remaining in LV after filling/diastole
Usually at the level of the pre-capillary arteriole. MVO2: how well are tissues extracting oxygen

PCWP: Measured at the right atrium:


If tissues are extracting a lot of O2 -> MVO2
Pressure in the pulmonary artery surrogate for LAP will be low

SVO2: If tissues are extracting less O2 -> MVO2


will be high
Related to oxygen extraction What is the primary etiology of each shock and
extrapolate
• If tissues are extracting a lot of O2 MVO2 will be low
• If tissues are extracting less O2 MVO2 will be high.

Cardiology: Shock

24
NBME Top Concepts ➤ Cardiology Shock

Mixed Venous Oxygen Saturation

Creative Commons Attribution License 4.0

Shock for the USMLE

Creative Commons Attribution License 4.0

Cardiology: Shock

25
uploaded by medbooksvn
NBME Top Concepts ➤ Cardiology Shock

Cardiac Output for the USMLE

Cardiology: Shock

26
NBME Top Concepts ➤ Cardiology Thermoregulation

Shivering
Response to Shivering
An athlete is submerged in an ice-bath
What is the likely physiologic response?
• Shivering posterior hypothalamus regulates this.
• ɑ-motor neurons & gamma-motor are activated skeleton muscle contraction and
heat production.
• Thyroid hormone increases metabolic rate by increasing Na/K-ATPase
• Brown fat via beta-3 (Gs) mediated activity.

Heat Stroke
A football player presents in summer after fainting. He is noted to have temp 104F, tachypnea,
and rigid muscles. CPK is elevated. The patient is also noted to have increased Cr. What is the likely
mechanism?
Abnormal hypothalamic thermoregulatory response.

Heat Dissipation
Vasodilation. Mechanism?
• Hypothalamus mediated peripheral vasodilation
• Exposes blood to cooler air

Sweating. Mechanism?
• Dissipation of heat usually patients in exam questions will have an absence of sweating

Pathophysiology:
• High body temp proteins denature cell membranes damage multi-organ failure.

Cardiology: Thermoregulation

27
uploaded by medbooksvn
NBME Top Concepts ➤ Cardiology S3 vs. S4

NBME Style Question

A 62-year-old man dies while playing tennis. At autopsy, examination shows cardiac valve defect
and concentric left ventricular hypertrophy. Which of the following valve abnormalities is involved in
his sudden death?

A.Aortic Insufficiency
B.Aortic Stenosis
C.Mitral insufficiency
D.Mitral stenosis
E.Pulmonic insufficiency
F.Pulmonic stenosis

Concentric Hypertrophy

Creative Commons Attribution License 4.0

S3-Gallop

During what phase of the cardiac cycle will you hear this sound?
• Early diastolic sound rapid filling of the left ventricle

On USMLE multi-media questions where will be the most likely position to hear this sound?
• Apex especially in the left-lateral

USMLE Vignettes:
• Mitral regurgitation (the best indicator of MR severity)
• Dilated Cardiomyopathy.

Cardiology: S3 vs. S4

28
NBME Top Concepts ➤ Cardiology S3 vs. S4

Atria rapidly filling into a dilated left-ventricle.

Creative Commons Attribution License 4.0

S4

During what phase of the cardiac cycle will you hear this sound?
• late diastolic sound atrial contraction

USMLE Vignettes:
• “Reduced ventricular compliance”
• Hypertrophy.

Left Ventricular Hypertrophy Questions on the USMLE

Creative Commons Attribution License 4.0

Cardiology: S3 vs. S4

29
uploaded by medbooksvn
NBME Top Concepts ➤ Cardiology S3 vs. S4

S3 vs. S4

Cardiology: S3 vs. S4

30
NBME Top Concepts ➤ Cardiology Murmurs

Murmurs

A 15-year-old female has a grade 2/6 holosystolic murmur is


heard best over the left fifth intercoastal space adjacent to the
sternum. It increases with inspiration, this murmur is most
consistent with an abnormality of which of following valves?

A.Aortic
B.Mitral
C.Pulmonary
D.Tricuspid
Creative Commons Attribution License 4.0

Murmurs for the USMLE


Locate the Area of Auscultation
Characterize Systolic VS. Diastolic
• Systolic:
• Aortic Stenosis
• Mitral Regurgitation
• VSD
• HOCOM
• MVP

• Diastolic:
• Mitral stenosis
• Aortic Regurgitation

Summary of Maneuvers
Less Blood in the Heart:
• MVP louder (click is earlier)
• HOCOM

Afterload (high):
• Regurgitant murmurs louder

Cardiology: Murmurs

31
uploaded by medbooksvn
NBME Top Concepts ➤ Cardiology Vasculitis

How to Recognize Vasculitis on the USMLE


Histology warm-up: what are the three normal layers of an arterial blood vessel?
• Intima
• Media
• Adventitia

USMLE presentations:
• Fever, fatigue, weight loss, and myalgias non-specific systems full of multi-system involvement.

Giant Cell Temporal Arteritis

An elderly female presents with muscle pain and difficulty chewing. She states that she had a
transient loss of vision during driving. Her ESR is elevated. What is the next best step in
management?

• Corticosteroids Giant Cell Temporal Arteritis


• Granulomas that affect the branches of the carotid (temporal, ophthalmic)

This vasculitis is associated with this USMLE vignette/disease:


A 70-year-old patient with joint pain. She has a fever and weight loss. Has difficulty climbing stairs
and combing hair. Exam shows pain and stiffness in the shoulders and hip. ESR is elevated, and CK
is normal. What is the most likely diagnosis?
• Polymyalgia rheumatica associate with Temporal Giant Cell arteritis.

Hemoptysis and Hematuria

Cardiology: Vasculitis

32
NBME Top Concepts ➤ Cardiology Post Myocardial Infarction Complications

Myocardial Infarction
• A patient presents with substernal chest pain which he noticed after a hike with his grandchildren.
He said that the pain stopped after he rested on a bench. He is a smoker, and his exam is normal.
What is the likely diagnosis?
• Stable angina:
• Deep poorly localized pain that relieves with rest or nitroglycerin brought on by activity
• Treatment: Vasodilate by increase NO in vascular smooth muscle

• What would you see on pathology:


Atherosclerosis of the coronary artery lumen > 75% however less than 100%

• Plaques can rupture and then usually have super imposed thrombi. If after they rupture and almost
occlude the whole lumen of the coronary vessel, what pathology does this refer to:
• Unstable angina (negative troponins)
• Subendocardial infarction (positive troponins) NSTEMI

• Plaques that occlude the whole lumen: transmural infarction.

• What is the most common cause of death after myocardial ischemia caused by coronary artery
disease?
• Arrhythmia VF

• A patient presents dead to the ER, autopsy shows complete occlusion of the LAD. What is the likely
cause of death?
• VF most common prehospital cause of death in MI patients

• What is the most common in-hospital death following MI?


• Ventricular failure (LV failure/cardiogenic shock)
• Remember it takes time (3-7 days) to get structural issues:
• Ventricular rupture, mural thrombus, tamponade.

• A patient’s heart is studied on autopsy after he myocardial suffered a myocardial infarction. He


presented to the hospital within four hours of the acute attack. What would light-microscopy
immediately after the event yield?
• No visible changes.
• First changes occur 4-12 hour range:
• Wavy fibers with elongated myocytes

• Cell death ensues:


• The nucleus shrivels and darkens until there is no more euchromatin (pyknosis) fragments
(karyorrhexis) then vanishes (karyolysis)
• Mitochondrial vacuolization is a sign of irreversible cell injury

Cardiology: Post Myocardial Infarction Complications

33
uploaded by medbooksvn
NBME Top Concepts ➤ Cardiology Post Myocardial Infarction Complications

Myocardial Infarction
• Inflammation ensues and what pathological change occurs one day?
• Neutrophillic infilration and coagulation necrosis

• At one week macrophages become the prominent cell and at two weeks granulation tissue
becomes neovascularized
• Structural complications like free wall rupture, septum rupture, or pap muscle rupture occur
approx. 1 week after MI

• At a month’s time, a patient will have scar formation

• What antiarrhythmic is best after MI?


• Lidocaine, MexileTine.
“I’d Buy Liddy’s Mexican Tacos.”
• Class IB Na channel blockers.

Timeline of Myocardial Infarction

Contraction Bands

Creative Commons Attribution License 4.0

Cardiology: Post Myocardial Infarction Complications

34
NBME Top Concepts ➤ Cardiology Answer Key

DiGeorge Syndrome

A 2-months-old female presents with seizures. She is lethargic and hypertonic. The patient is
also found to have respiratory distress. The exam is notable for a cleft palate, and a murmur is
heard on the clinical exam. What is the likely embryological mechanism?

A. Defective neural crest migration.


B. Neural crest defect.
C. Failure of the 3rd and 4th arch to develop.
D. Failure of 3rd and 4th pouch to develop.
E. Failure of 3rd and 4th cleft to develop.

Lipid Lowering Drugs

A 40-year-old female with a history of Grave’s disease is noted to have intermittent chest pain
with activity. She has a history of elevated LDL. She takes PTU for hyperthyroidism. She is started
on a medication to control her dyslipidemia. Which of the following best explains the mechanism
of action of this medication?

A. Activation of HMG-CoA Synthase.


B. Upregulation of hepatocyte LDL-receptor.
C. Downregulation of bile-acid synthesis.
D. In-activation of reverse transport in cholesterol metabolism.
E. Upregulation of bile-acid metabolism.

Shock

A 50-year-old male presents with severe chest pain while mowing his lawn. His vital signs:
110/min, 20 breaths/min, 105/70. He is cool and clammy. He is noted to have an elevated PCWP
and left-atrial pressure of 20. What are the most likely pressure changes in the pulmonary
vasculature seen?

Cardiology: Answer Key

35
uploaded by medbooksvn
NBME Top Concepts ➤ Cardiology Answer Key

S3 vs. S4

A 62-year-old man dies while playing tennis. At autopsy, examination shows cardiac valve defect
and concentric left ventricular hypertrophy. Which of the following valve abnormalities is involved
in his sudden death?

A.Aortic Insufficiency
B.Aortic Stenosis
C.Mitral insufficiency
D.Mitral stenosis
E.Pulmonic insufficiency
F.Pulmonic stenosis

Cardiology: Answer Key

36
CHAPTER 3

High Yield Images


& PE for the USMLE
Step 1 (Introduction)

37
uploaded by medbooksvn
High Yield Images & PE for the USMLE Step 1 ➤ Introduction USMLE Test Taking Strategy Approach

How multimedia questions are written on the NBME/USMLE?

Types of Media:
• Static Images
• Patient Photographs
• Videos
• Interactive Media
• Sound files

Content Areas Conducive to the Use of Media

• Dermatologic & MSK findings


• Cardiology (heart sounds)
• Neurological exam findings
• Ethical and Communication Scenarios

Creative Commons Attribution License 4.0

Test Taking Approach

An image will not be tested in isolation – the vignette matters!

Example Item using Media

Creative Commons Attribution License 4.0

High Yield Images & PE Findings: USMLE Test Taking Strategy Approach

38
High Yield Images & PE for the USMLE Step 1 ➤ Introduction Test Taking Strategy for Image Questions

Test Taking Strategy for Image Questions

Application of Test-Taking Strategy

A 40-year-old male with AIDS has 6-week history of


weakness in his right hand and mild headache. A CT
head is shown. What is the likely causal Organism?
▹Define the image:
▹Radiographic
▹Of what:
▹Brain
▹Look for normal, abnormal, asymmetry:
▹Circular, enhancing intraparenchymal mass.
be tested in isolation – the vignette matters!

Creative Commons Attribution License 4.0

High Yield Images & PE Findings: Test Taking Strategy for Image Questions

39
uploaded by medbooksvn
High Yield Images & PE for the USMLE Step 1 ➤ Introduction Test Taking Strategy for Image Questions

Ring Enhancing Brain Lesions for the USMLE

Creative Commons Attribution License 4.0

High Yield Images & PE Findings: Test Taking Strategy for Image Questions

40
High Yield Images & PE for the USMLE Step 1 ➤ Introduction Top Images You Will Encounter on the USMLE

General Layout
A 45-year-old male has a MVA and suffers a right
femoral shaft fracture. He has a surgery, and 3 days post
op he presents with tachycardia, tachypnea and pleuritic
chest pain. He is hypoxemic and has a rash shown. CT
shows blockage of pulmonary artery flow. What is the
most likely cause of the patient’s hypoxemia?

A. Fat Embolism
B. Acute Contact Dermatitis
C. Meningococcemia
D. Anesthesia Reaction
Creative Commons Attribution License 4.0

NBME Style Question


A 60-year-old male with history of alcoholism presents
with vomiting bright red blood. The patient is tachycardic
and hypotensive. In the ED, despite resuscitation, the
patient passes away. An autopsy is performed to identify
the cause of death. An image of the esophagus is shown.
Which of the following mechanisms most likely
contributed to this patient’s hemorrhage?

A. Eosinophilic Esophagitis
B. Deficiency of Vitamin K dependent clotting factors
C. Peptic Ulcer Disease
D. Barret’s Esophagus
E. Imbalance of pressures in the caval-portal system

Creative Commons Attribution License 4.0

High Yield Images & PE Findings: Top Images You Will Encounter on the USMLE

41
uploaded by medbooksvn
High Yield Images & PE for the USMLE Step 1 ➤ Introduction Top Images You Will Encounter on the USMLE

Imbalance of Two Systems = Portal HTN

Creative Commons Attribution License 4.0

Esophageal Varices

NBME Style Question


A patient is noted to have ten to fifteen 2 cm lesions on
her trunk, neck, and lower extremities. The lesions have
been present since birth. A lesion on her extremity is
shown. Multiple family members are also known to have
this lesion. Given this finding, the patient is most likely to
have which associated pathology?

A. Renal angiomyolipomas
B. Hamartomas seen in the iris
C. Charcot Leyden crystals in sputum Creative Commons Attribution License 4.0

D.Ashleaf spots

High Yield Images & PE Findings: Top Images You Will Encounter on the USMLE

42
High Yield Images & PE for the USMLE Step 1 ➤ Introduction Top Images You Will Encounter on the USMLE

Neuro-fibromatosis

Creative Commons Attribution License 4.0

NBME Style Question

A 17 year old boy presents to the emergency


department after having a tonic-clonic seizure. He is
noted to have a skin exam shown in the
photomicrograph. Upon further history, which of the
following findings would likely to be found?

A. Hearing loss
B. Episodic depression
C. Individualized learning plan in childhood
D. Family history of substance use Creative Commons Attribution License 4.0

High Yield Images & PE Findings: Top Images You Will Encounter on the USMLE

43
uploaded by medbooksvn
High Yield Images & PE for the USMLE Step 1 ➤ Introduction Top Images You Will Encounter on the USMLE

Tuberous Sclerosis

Creative Commons Attribution License 4.0

NBME Style Question

A 50-year-old male presents with weakness. He is


noted to have hypertension & hypokalemia. An
abdominal CT scan reveals an adrenal mass. The
patient undergoes biopsy of the mass. An EM image
is shown of the adrenal cortex. The patient’s
pathology is most closely related to which labeled
zone in the photo-micrograph?

A. A
B. B
C. C
D. D
E. E

Creative Commons Attribution License 4.0

High Yield Images & PE Findings: Top Images You Will Encounter on the USMLE

44
High Yield Images & PE for the USMLE Step 1 ➤ Introduction Top Images You Will Encounter on the USMLE

Hypertension and Hypokalemia

Creative Commons Attribution License 4.0

NBME Style Question


A 60 -year-old male presents for follow up. The patient has
suffered from a myocardial infarction nine-months prior, and since
then has been seen as an outpatient for cardiac rehabilitation. His
recovery was uneventful. A photomicrograph of the area of
infarction is shown. Which of the following macrophage products
is primary responsible for the tissue labeled X?

A. Fibroblast Growth Factor


B. Interferon Gamma
C. Leukotrienes D4
D. Nitric oxide
Creative Commons Attribution License 4.0

Timeline of Myocardial Infarction

Creative Commons Attribution License 4.0

High Yield Images & PE Findings: Top Images You Will Encounter on the USMLE

45
uploaded by medbooksvn
High Yield Images & PE for the USMLE Step 1 ➤ Introduction Top Images You Will Encounter on the USMLE

NBME Style Question


A 10-year-old female has fever, tiredness, and a petechial skin
rash. Her laboratory studies show a low hemoglobin, low leukocyte
count, and low platelets. A bone marrow biopsy is performed.
What is the most likely diagnosis?

A. Immune thrombocytopenic pupura


B. Cyanocobalamin deficiency
C. Aplastic anemia
D. Acute lymphoblastic leukemia (ALL)
Creative Commons Attribution License 4.0

Aplastic Anemia

NBME Style Question

A 12-year-old female recently was treated for


pneumonia. The patient was found to have serologies
for mycoplasma positive in the blood. A few days into
her illness, she presents with multiple itchy targetoid
lesions. Exam of the lesions is shown. Which of the
following complications may be associated with this
patient’s condition?

A. Hemoptysis and hematuria


B. Superimposed HSV infection
C. Anemia with Coombs positivity
Creative Commons Attribution License 4.0

D. Positive Nikolsky sign

High Yield Images & PE Findings: Top Images You Will Encounter on the USMLE

46
High Yield Images & PE for the USMLE Step 1 ➤ Introduction Top Images You Will Encounter on the USMLE

Mnemonic (Rash on Palms and Soles)

NBME Style Question

A 55-year-old man is in the neuro ICU after suffering


a stroke. The patient undergoes an MRI of the brain.
The imaging is shown. Which of the following
symptoms may this patient have?

A. Bitemporal Hemianopsia
B. L sided weakness of face and dysarthria
C. R sided weakness of face and dysarthria
D. Pure sensory loss of the contralateral body
E. Weakness of the trunk

Creative Commons Attribution License 4.0

High Yield Images & PE Findings: Top Images You Will Encounter on the USMLE

47
uploaded by medbooksvn
High Yield Images & PE for the USMLE Step 1 ➤ Introduction Top Images You Will Encounter on the USMLE

Localizing Strokes

NBME Style Question


A patient three months post renal transplant has
increasing dyspnea. An X-ray shows atypical, interstitial
infiltrates. A broncho-alveolar lavage and lung biopsy is
performed. Sample is shown in the photomicrograph. What
is the likely organism?

A. CMV
B. EBV
C. Hepatitis B
D. HTLV-1
E. Papillomavirus
F. Reed Sternburg Creative Commons Attribution License 4.0

NBME Style Question


A teenage girl is noted to have sore throat, fever, and malaise. She
has tender cervical lymphadenopathy. Abdominal exam reveals a LUQ
mass. Laboratory studies show normal hemoglobin, slight elevation in
leukocyte count with lymphocytic predominance. A peripheral blood
smear is shown. What is the most likely diagnosis?

A. Acute lymphoblastic Leukemia


B. Acute Myelogenous Leukemia
C. Infectious mono-nucleosis
D. Hemolytic crisis with reticulocytosis
Creative Commons Attribution License 4.0

High Yield Images & PE Findings: Top Images You Will Encounter on the USMLE

48
High Yield Images & PE for the USMLE Step 1 ➤ Introduction Top Images You Will Encounter on the USMLE

EBV+

NBME Style Question


An 86-year-old male presents with this wife for
progressive memory loss. Wife states that the
patient is unable to balance his checkbooks. A
medication is prescribed to improve these patient’s
symptoms. The patient on follow up one year dies.
Autopsy is performed. A sample brain tissue is
shown. Which of the following is the most likely
cause of this patient’s demise?

A. CJD
B. Hypothyroidism Creative Commons Attribution License 4.0

C. Fronto-temporal dementia
D. Alzheimer's disease

High Yield Images & PE Findings: Top Images You Will Encounter on the USMLE

49
uploaded by medbooksvn
High Yield Images & PE for the USMLE Step 1 ➤ Introduction Top Images You Will Encounter on the USMLE

Frontotemporal vs. Alzheimers

NBME Style Question


A patient is found to have the following
physical exam finding shown in the image. Which
of the following nerve muscle pairs are likely
injured?

A. Dorsal scapular – rhomboid major


B. Thoracodorsal – trapezius
C. Thoracodorsal – serratus anterior
D. Long thoracic – serratus anterior
E. Long thoracic – trapezius
F. Dorsal scapular – subscapularis
Creative Commons Attribution License 4.0

High Yield Images & PE Findings: Top Images You Will Encounter on the USMLE

50
High Yield Images & PE for the USMLE Step 1 ➤ Introduction Top Images You Will Encounter on the USMLE

Long Thoracic Nerve

Creative Commons Attribution License 4.0

NBME Style Question


A patient presents with the physical exam finding shown after excision of a right sided neck mass.
Which of the following nerve fibers are likely affected?

A. Post synaptic parasympathetic fibers of the vagus nerve.


B. Post synaptic fibers of the superior cervical ganglion.
C. Pre synaptic fibers from the recurrent laryngeal nerve.
D. Splanchnic nerve fibers from the cervical sympathetic trunk.

Creative Commons Attribution License 4.0

High Yield Images & PE Findings: Top Images You Will Encounter on the USMLE

51
uploaded by medbooksvn
High Yield Images & PE for the USMLE Step 1 ➤ Introduction Answer Key

A 60-year-old male with history of alcoholism presents with vomiting bright red blood. The patient
is tachycardic and hypotensive. In the ED, despite resuscitation, the patient passes away. An
autopsy is performed to identify the cause of death. An image of the esophagus is shown. Which of
the following mechanisms most likely contributed to this patient’s hemorrhage?

A. Eosinophilic Esophagitis
B. Deficiency of Vitamin K dependent clotting factors
C. Peptic Ulcer Disease
D. Barret’s Esophagus
E. Imbalance of pressures in the caval-portal system

A patient is noted to have ten to fifteen 2 cm lesions on her trunk, neck, and lower extremities.
The lesions have been present since birth. A lesion on her extremity is shown. Multiple family
members are also known to have this lesion. Given this finding, the patient is most likely to have
which associated pathology?

A. Renal angiomyolipomas
B. Hamartomas seen in the iris
C. Charcot Leyden crystals in sputum
D.Ashleaf spots

A 17 year old boy presents to the emergency department after having a tonic-clonic seizure. He is
noted to have a skin exam shown in the photomicrograph. Upon further history, which of the
following findings would likely to be found?

A. Hearing loss
B. Episodic depression
C. Individualized learning plan in childhood
D. Family history of substance use

A 50-year-old male presents with weakness. He is noted to have hypertension & hypokalemia. An
abdominal CT scan reveals an adrenal mass. The patient undergoes biopsy of the mass. An EM
image is shown of the adrenal cortex. The patient’s pathology is most closely related to which
labeled zone in the photo-micrograph?

A. A
B. B
C. C
D. D
E. E

High Yield Images & PE Findings: Answer Key

52
High Yield Images & PE for the USMLE Step 1 ➤ Introduction Answer Key

A 60 -year-old male presents for follow up. The patient has suffered from a myocardial infarction
nine-months prior, and since then has been seen as an outpatient for cardiac rehabilitation. His
recovery was uneventful. A photomicrograph of the area of infarction is shown. Which of the
following macrophage products is primary responsible for the tissue labeled X?

A. Fibroblast Growth Factor


B. Interferon Gamma
C. Leukotrienes D4
D. Nitric oxide

A 10-year-old female has fever, tiredness, and a petechial skin rash. Her laboratory studies show
a low hemoglobin, low leukocyte count, and low platelets. A bone marrow biopsy is performed. What
is the most likely diagnosis?

A. Immune thrombocytopenic pupura


B. Cyanocobalamin deficiency
C. Aplastic anemia
D. Acute lymphoblastic leukemia (ALL)

A 12-year-old female recently was treated for pneumonia. The patient was found to have
serologies for mycoplasma positive in the blood. A few days into her illness, she presents with
multiple itchy targetoid lesions. Exam of the lesions is shown. Which of the following complications
may be associated with this patient’s condition?

A. Hemoptysis and hematuria


B. Superimposed HSV infection
C. Anemia with Coombs positivity
D. Positive Nikolsky sign

A 55-year-old man is in the neuro ICU after suffering a stroke. The patient undergoes an MRI of the
brain. The imaging is shown. Which of the following symptoms may this patient have?

A. Bitemporal Hemianopsia
B. L sided weakness of face and dysarthria
C. R sided weakness of face and dysarthria
D. Pure sensory loss of the contralateral body
E. Weakness of the trunk

High Yield Images & PE Findings: Answer Key

53
uploaded by medbooksvn
High Yield Images & PE for the USMLE Step 1 ➤ Introduction Answer Key

A patient three months post renal transplant has increasing dyspnea. An X-ray shows atypical,
interstitial infiltrates. A broncho-alveolar lavage and lung biopsy is performed. Sample is shown in
the photomicrograph. What is the likely organism?

A. CMV
B. EBV
C. Hepatitis B
D. HTLV-1
E. Papillomavirus
F. Reed Sternburg

A teenage girl is noted to have sore throat, fever, and malaise. She has tender cervical
lymphadenopathy. Abdominal exam reveals a LUQ mass. Laboratory studies show normal
hemoglobin, slight elevation in leukocyte count with lymphocytic predominance. A peripheral blood
smear is shown. What is the most likely diagnosis?

A. Acute lymphoblastic Leukemia


B. Acute Myelogenous Leukemia
C. Infectious mono-nucleosis
D. Hemolytic crisis with reticulocytosis

An 86-year-old male presents with this wife for progressive memory loss. Wife states that the
patient is unable to balance his checkbooks. A medication is prescribed to improve these patient’s
symptoms. The patient on follow up one year dies. Autopsy is performed. A sample brain tissue is
shown. Which of the following is the most likely cause of this patient’s demise?

A. CJD
B. Hypothyroidism
C. Fronto-temporal dementia
D. Alzheimer's disease

A patient is found to have the following physical exam finding shown in the image. Which of the
following nerve muscle pairs are likely injured?

A. Dorsal scapular – rhomboid major


B. Thoracodorsal – trapezius
C. Thoracodorsal – serratus anterior
D. Long thoracic – serratus anterior
E. Long thoracic – trapezius
F. Dorsal scapular – subscapularis

High Yield Images & PE Findings: Answer Key

54
High Yield Images & PE for the USMLE Step 1 ➤ Introduction Answer Key

A patient presents with the physical exam finding shown after excision of a right sided neck mass.
Which of the following nerve fibers are likely affected?

A. Post synaptic parasympathetic fibers of the vagus nerve.


B. Post synaptic fibers of the superior cervical ganglion.
C. Pre synaptic fibers from the recurrent laryngeal nerve.
D. Splanchnic nerve fibers from the cervical sympathetic trunk.

High Yield Images & PE Findings: Answer Key

55
uploaded by medbooksvn
CHAPTER 4

High Yield USMLE


Images & PE Findings:
Cardiology

56
High Yield USMLE Images & PE Findings ➤ Cardiology Murmurs & Maneuvers

NBME Style Question


An 80-year-old male presents with SOB and fatigue. The patient was brought into the emergency
department last week due to lightheadedness during activity. The patient is noted to have a harsh
ejection-type murmur at the base of the heart radiating to the neck. Given this murmur, which
additional finding may be seen in this patient?

A. S3
B. S4
C. Recurrent streptococcal infections in PMH
D. Myxomatous valve degeneration due to prolapse

Murmurs on the USMLE

Creative Commons Attribution License 4.0

Test Taking Approach to Murmurs

Cardiology: Murmurs & Maneuvers

57
uploaded by medbooksvn
High Yield USMLE Images & PE Findings ➤ Cardiology Murmurs & Maneuvers

Systolic vs. Diastolic Murmurs for the USMLE

Maneuvers

Cardiology: Murmurs & Maneuvers

58
High Yield USMLE Images & PE Findings ➤ Cardiology Murmurs & Maneuvers

Murmurs

An 80-year-old male with a cresendo-decresendo systolic murmur that increases with expiration.
He has a history of chest pain, and SOB; now he presents after passing out. His murmur is heard best
at the right second intercostal space with radiation to the carotids. What feature may correlate to
increased severity of this murmur?
• A longer and later peak of the murmur intensity of murmur also relates to LV-aorta gradient
Aortic Stenosis
• What if this patient was a middle-aged male with aortic stenosis. What would be the likely
pathophysiologic mechanism?
• Bicuspid aortic valve.

Bicuspid Aortic Valve

Creative Commons Attribution License 4.0

NBME Style Question


A patient presents due to syncope. He has no history of trauma and is otherwise healthy. The
patient has a harsh systolic murmur throughout the precordium. Lungs are clear and there are no
signs of JVD or pitting edema. An echo shows interventricular septal enlargement and low EF. What
is the likely mechanism behind the patient’s reduced cardiac output?

A. Aortic valve cusp increasing size of LVOT


B. Aortic valve dilation and blood in intima of aorta
C. Calcified fused aortic valve leaflets
D. Mitral leaflet obstructing LVOT

Cardiology: Murmurs & Maneuvers

59
uploaded by medbooksvn
High Yield USMLE Images & PE Findings ➤ Cardiology Murmurs & Maneuvers

NBME Style Question


A patient dies of ventricular arrythmia. Post portem autopsy shows an abnormal interventricular
septum, three times as thick as the normal interventricular septum. A genetic analysis of this gross
pathological finding would likely show which of the following abnormalities?

A. Infiltrative disease due to sarcoid.


B. Genetically mutated potassium channels.
C. Granulomatous inflammation of myocardium.
D. Genetically abnormal myocardial muscle proteins.

Hypertrophic Obstructive Cardiomyopathy


• What is the micro-pathology finding for HOCOM?
– Myofiber disarray
• What is the likely genetic mutation behind HOCOM?
– Cardiac sarcomere proteins
• Beta-myosin heavy chain
• Causes diastolic dysfunction and mitral valve regurgitation secondary to anterior motion of the
anterior leaflet of mitral valve
– Characteristic of diastolic heart failure – high LV end diastolic pressures
• What PE maneuvers increase intensity of this murmur?
– Maneuvers that decrease preload:
• Standing
• Valsalva

High Yield NBME Images | Hypertrophic Cardiomyopathy

Cardiology: Murmurs & Maneuvers

60
High Yield USMLE Images & PE Findings ➤ Cardiology Murmurs & Maneuvers

PE Features of Congestive Heart Failure

A 50-year-old obese male with a history of


smoking presents with increased SOB and exam
showing rales with pitting edema. The patient is also
noted to have the following PE finding. On echo, he is
noted to have a low EF on echocardiography. What is
the likely murmur heard on a physical exam?
• S3 Early Diastolic Murmur
• Represents the la filling into a dilated LV.

Right Heart Failure vs. Left Heart Failure for the USMLE

Cardiology: Murmurs & Maneuvers

61
uploaded by medbooksvn
High Yield USMLE Images & PE Findings ➤ Cardiology Right Heart Failure vs. Left Heart Failure

Right Heart Failure

NBME Style Question


A patient presents to the ICU with heart failure exacerbation he has CKD and hypertension. Despite
aggressive resuscitation that patient dies. Autopsy shows alveolar cells containing increased
amount of golden brown cytoplasmic granules that stain Fe2+. Which of the following mechanisms
is associated with this autopsy finding?

A. Pulmonary embolism
B. LV ejection fraction of 30%
C. Pneumoconiosis
D. Airway hyper-reactivity

Cardiology: Right Heart Failure vs. Left Heart Failure

62
High Yield USMLE Images & PE Findings ➤ Cardiology Transudative vs. Exudative for the USMLE

Transudative vs. Exudative for the USMLE

Cardiology: Transudative vs. Exudative for the USMLE

63
uploaded by medbooksvn
High Yield USMLE Images & PE Findings ➤ Cardiology Mechanisms of Edema

Mechanisms of Edema

Creative Commons Attribution License 4.0

Cardiology: Mechanisms of Edema

64
High Yield USMLE Images & PE Findings ➤ Cardiology Turner Syndrome

Turner Syndrome

Creative Commons Attribution License 4.0

Cardiology: Turner Syndrome

65
uploaded by medbooksvn
High Yield USMLE Images & PE Findings ➤ Cardiology Genetic Heart Lesions & Cardiac Anomalies

Turner Syndrome

Cardiology: Genetic Heart Lesions & Cardiac Anomalies

66
High Yield USMLE Images & PE Findings ➤ Cardiology Genetic Conditions & Heart Lesions

Genetic Conditions & Heart Lesions


Neonate with seizures secondary to hypocalcemia and recurrent
fungal infections. A genetic etiology is suspected. What is the likely
cardiac lesion associated with this genetic etiology?
• Conotruncal Cardiac Defects
• Tetralogy of Fallot
• Truncus Arteriosus
• Interrupted Aortic Arch.

Creative Commons Attribution License 4.0

Cardiology: Genetic Conditions & Heart Lesions

67
uploaded by medbooksvn
High Yield USMLE Images & PE Findings ➤ Cardiology Pathophysiology of Tetralogy of Fallot

Pathophysiology of Tetralogy of Fallot

฀ Shaped Heart

Creative Commons Attribution License 4.0

Cardiology: Pathophysiology of Tetralogy of Fallot

68
High Yield USMLE Images & PE Findings ➤ Cardiology Pathophysiology of Tetralogy of Fallot

NBME Style Question


A child presents with cyanosis. He has a history of 22q11 deletion and an unrepaired heart lesion.
The parents notice the cyanosis exacerbated by increased agitation. Which of the following
symptoms is the primary driver for repair of this child’s heart lesion?

A. VSD size
B. Tricuspid insufficiency
C. Right ventricular outflow tract obstruction
D. Degree of step down of PaO2 due to R L shunting

Cardiology: Pathophysiology of Tetralogy of Fallot

69
uploaded by medbooksvn
High Yield USMLE Images & PE Findings ➤ Cardiology Pathophysiology of Tet Spells

Pathophysiology of Tet Spells

Cardiology: Pathophysiology of Tet Spells

70
High Yield USMLE Images & PE Findings ➤ Cardiology Endocarditis

NBME Style Question


A 50-year-old male presents with two weeks of fever and fatigue. The patient is noted to have a
mitral valve lesion diagnosed on echocardiography at age 35. The patient presents now with
increased CRP, increased ESR and a blood culture positive for gram positive cocci that are alpha
hemolytic and optichin resistant. Which of the following additional history findings would be present
in this patient?

A. History of dental procedure


B. History of peritoneal dialysis
C. History of prostate hyperplasia
D. History of IV Drug abuse

Mitral Valve Prolapse


• What is the most common cause of mitral regurgitation?
• Mitral Valve Prolapse
• Bulging MVs into LA during LV systole; redundant valve tissue
• What disorders may be associated?
• Marfan, EDS, Klinefelter caused by defective embryogenesis of mesenchyme
• What makes this murmur louder?
• Valsalva and standing.

Cardiology: Endocarditis

71
uploaded by medbooksvn
High Yield USMLE Images & PE Findings ➤ Cardiology Microbiology Integration

Microbiology Integration
• Big Picture:
• Subacute bacterial endocarditis usually presents as indolent symptoms of fever + murmur
(clinical picture)
• USMLE puts dental surgery + hx of MVP + clinical picture:
• S. viridans
• Dextrans adhere to tooth enamel and fibrin-platelet aggregates
• USMLE puts abdominal surgery or on hemo-dialysis + clinical picture:
• Enterococcus
• Nonhemolytic (Group D), catalase-negative, gram-positive cocci in pairs and chains
• USMLE puts history of mechanical valve + clinical picture:
• S epidermidis
• What is the micro lab characteristic of strept viridans and staph epidermidis?
• Strept Viridans: catalase negative, alpha-hemolytic
• Green zone of hemolysis on blood agar
• Staph Epidermidis: catalase positive, coagulase negative.

Cardiology: Microbiology Integration

72
High Yield USMLE Images & PE Findings ➤ Cardiology Infective Endocarditis on USMLE

Infective Endocarditis on USMLE – watch for fever + murmur

Cardiology: Infective Endocarditis on USMLE

73
uploaded by medbooksvn
High Yield USMLE Images & PE Findings ➤ Cardiology Rheumatic Fever

NBME Style Question


A child who has immigrated from West Bengal presents to the pediatrician with fatigue and joint
pain. The patient has not had any immunizations or birth records sent to the physician. On exam, the
patient has a rumbling diastolic murmur heard best at the apex. The patient has a normal
neurological exam. Which of the following mechanisms may be contributing to this pathology?

A. Type 1 HS
B. Type 2 HS
C. Type 3 HS
D. Type 4 HS

Rheumatic Fever
A patient who recently immigrated to the USA presents with abnormal involuntary jerking
movements. He has a history of sore throat 9 months prior. What is the likely area of the brain which
may be affected?
• Autoimmune destruction of basal ganglia as they cross react with streptococcal antigen (M-
protein) and brain parenchyma molecular mimicry.

JONES Criteria for Rheumatic Fever

Creative Commons Attribution License 4.0

Cardiology: Rheumatic Fever

74
High Yield USMLE Images & PE Findings ➤ Cardiology Rheumatic Fever

JONES Criteria for Rheumatic Fever

Cardiology: Rheumatic Fever

75
uploaded by medbooksvn
High Yield USMLE Images & PE Findings ➤ Cardiology Erythema Marginatam

Erythema Marginatam

Cardiology: Erythema Marginatam

76
High Yield USMLE Images & PE Findings➤ Cardiology Answer Key

Murmurs & Maneuvers


An 80-year-old male presents with SOB and fatigue. The patient was brought into the emergency
department last week due to lightheadedness during activity. The patient is noted to have a harsh
ejection-type murmur at the base of the heart radiating to the neck. Given this murmur, which
additional finding may be seen in this patient?

A. S3
B. S4
C. Recurrent streptococcal infections in PMH
D. Myxomatous valve degeneration due to prolapse

A patient presents due to syncope. He has no history of trauma and is otherwise healthy. The
patient has a harsh systolic murmur throughout the precordium. Lungs are clear and there are no
signs of JVD or pitting edema. An echo shows interventricular septal enlargement and low EF. What
is the likely mechanism behind the patient’s reduced cardiac output?

A. Aortic valve cusp increasing size of LVOT


B. Aortic valve dilation and blood in intima of aorta
C. Calcified fused aortic valve leaflets
D. Mitral leaflet obstructing LVOT

A patient dies of ventricular arrythmia. Post portem autopsy shows an abnormal interventricular
septum, three times as thick as the normal interventricular septum. A genetic analysis of this gross
pathological finding would likely show which of the following abnormalities?

A. Infiltrative disease due to sarcoid.


B. Genetically mutated potassium channels.
C. Granulomatous inflammation of myocardium.
D. Genetically abnormal myocardial muscle proteins.

Right Heart Failure vs. Left Heart Failure


A patient presents to the ICU with heart failure exacerbation he has CKD and hypertension. Despite
aggressive resuscitation that patient dies. Autopsy shows alveolar cells containing increased
amount of golden brown cytoplasmic granules that stain Fe2+. Which of the following mechanisms
is associated with this autopsy finding?

A. Pulmonary embolism
B. LV ejection fraction of 30%
C. Pneumoconiosis
D. Airway hyper-reactivity

Cardiology: Answer Key

77
uploaded by medbooksvn
High Yield USMLE Images & PE Findings➤ Cardiology Answer Key

Pathophysiology of Tetralogy of Fallot


A child presents with cyanosis. He has a history of 22q11 deletion and an unrepaired heart lesion.
The parents notice the cyanosis exacerbated by increased agitation. Which of the following
symptoms is the primary driver for repair of this child’s heart lesion?

A. VSD size
B. Tricuspid insufficiency
C. Right ventricular outflow tract obstruction
D. Degree of step down of PaO2 due to R L shunting

Endocarditis
A 50-year-old male presents with two weeks of fever and fatigue. The patient is noted to have a
mitral valve lesion diagnosed on echocardiography at age 35. The patient presents now with
increased CRP, increased ESR and a blood culture positive for gram positive cocci that are alpha
hemolytic and optichin resistant. Which of the following additional history findings would be present
in this patient?

A. History of dental procedure


B. History of peritoneal dialysis
C. History of prostate hyperplasia
D. History of IV Drug abuse

Rheumatic Fever
A child who has immigrated from West Bengal presents to the pediatrician with fatigue and joint
pain. The patient has not had any immunizations or birth records sent to the physician. On exam, the
patient has a rumbling diastolic murmur heard best at the apex. The patient has a normal
neurological exam. Which of the following mechanisms may be contributing to this pathology?

A. Type 1 HS
B. Type 2 HS
C. Type 3 HS
D. Type 4 HS

Cardiology: Answer Key

78
CHAPTER 5

Top NBME Concepts:


Respiratory

79
uploaded by medbooksvn
NBME Top Concepts ➤ Respiratory A-a Gradient (Hypoxemia)

Recognition of Hypoxemia
USMLE Questions will test hypoxemia by showing low SpO in exam questions (i.e. <90%)
or low P O (<60 mmHg).

Use this as a pertinent positive as you paraphrase the vignette.

Pathophysiology of hypoxemia

NBME Style Question


A 52-year-old male is found unresponsive. His medical history is unknown. On PE, he is
unresponsive despite sternal rub. He is cool. His laboratory studies are notable for low pH, low PaO ,
low HCO , high CO . Which of the following diagnoses is most likely to be seen in this patient?

A. Respiratory Alkalosis
B. Metabolic Alkalosis
C. Hyperoxia
D. High lactate

Respiratory: A-a Gradient (Hypoxemia)

80
NBME Top Concepts ➤ Respiratory A-a Gradient (Hypoxemia)

Hypoxemia

Hypoventilation
An obese patient who presents to the ED after being at a party. He is somnolent and has pinpoint
pupils. RR = 8. What would be the likely acid base abnormality?
Respiratory acidosis
• Unique characteristic no change in A-a gradient.

High Altitude
An athlete who trains in Colorado. On day two of training, he is found to have an RR= 22. He is in
training above sea level for 3 weeks. Labs show elevated Hct. What is the likely mechanism?
Increased EPO secretion from peritubular capillary cells
• Unique characteristic no change in A-a gradient
Hypoxemia Increased synthesis of 2,3 BPG
Increased RR decreased cerebral perfusion light-headed.

Respiratory: A-a Gradient (Hypoxemia)

81
uploaded by medbooksvn
NBME Top Concepts ➤ Respiratory A-a Gradient (Hypoxemia)

High Altitude

Source: BRS Physiology 2019. Costanzo.

Physiology Integration
Where are peripheral chemoreceptors located?
Carotid body and aortic arch. What are the CN?
• IX, and X respectively.
What are their major stimuli?
• pH (carotid body > arch), pCO2, and primary sensitivity PaO2 (<60-80 mm Hg)
Why 60-80 mmHg?
• It is at this point where the Hb saturation of O2 drops at a fast rate.

Hypoxemia leading to Increased HCT

Respiratory: A-a Gradient (Hypoxemia)

82
NBME Top Concepts ➤ Respiratory A-a Gradient (Hypoxemia)

High Altitude

Source: BRS Physiology 2019. Costanzo.

Physiology Integration
Where are peripheral chemoreceptors located?
Carotid body and aortic arch. What are the CN?
• IX, and X respectively.
What are their major stimuli?
• pH (carotid body > arch), pCO2, and primary sensitivity PaO2 (<60-80 mm Hg)
Why 60-80 mmHg?
• It is at this point where the Hb saturation of O2 drops at a fast rate.

Hypoxemia leading to Increased HCT

Respiratory: A-a Gradient (Hypoxemia)

83
uploaded by medbooksvn
NBME Top Concepts ➤ Respiratory A-a Gradient (Hypoxemia)

NBME Style Question

A 45-year-old was hospitalized for hypoxemia. The patient has a history of smoking. He is noted to
have increased sputum production. His pulse oximetry is noted to be <90%. He has increased work
of breathing on the exam and respiratory distress. He breathes with pursed lips and has a copious
purulent cough. Which of the following mechanisms is the primary cause of hypoxemia?

A. Alveolar hypoventilation
B. Decreased oxygen extraction
C. Primary diffusion impairment
D. V/Q mismatch

V/Q Mismatch

A 45-year-old male has an MVA and suffers a right


femoral shaft fracture. He has surgery, and 3 days
post-op he presents with tachycardia, tachypnea, and
pleuritic chest pain. He is hypoxemic and has a rash
shown. CT shows blockage of pulmonary artery flow.
What is the most likely cause of the patient’s
hypoxemia?
V/Q mismatch Dead Space
• A-a gradient is increased
• Diagnosis: fat embolism.
Creative Commons Attribution License 4.0

Two Extremes of V/Q mismatch

Respiratory: A-a Gradient (Hypoxemia)

84
NBME Top Concepts ➤ Respiratory A-a Gradient (Hypoxemia)

V/Q Mismatch – Shunt (Concept: Hypoxemic Vasoconstriction)

Graphical Representation of Shunt (Concept: Hypoxemic Vasoconstriction)

Respiratory: A-a Gradient (Hypoxemia)

85
uploaded by medbooksvn
NBME Top Concepts ➤ Respiratory Regional Circulation for the USMLE

Regional Circulation for the USMLE

Cardiology Integration
What would chronic hypoxic vasoconstriction do to right ventricular afterload?

Creative Commons Attribution License 4.0

Respiratory: Regional Circulation for the USMLE

86
NBME Top Concepts ➤ Respiratory Regional Circulation for the USMLE

Summary of V/Q Mismatch

Creative Commons Attribution License 4.0

NBME Style Question


A 62-year-old male comes to the clinic due to a 6-month history of progressive exertional dyspnea.
He has no environmental exposures. The patient dies of respiratory failure. Autopsy findings is
notable for heterogenous lung parenchyma with predominantly subpleural areas of dense collagen
deposition, lymphocyte infiltration, and fibroblast proliferation. Which of the following is the most
likely diagnosis?

A. Alpha-1 anti-trypsin
B. Chronic bronchitis
C. Asbestosis
D. Idiopathic pulmonary fibrosis
E. Chronic hypersensitivity pneumonitis

Respiratory: Regional Circulation for the USMLE

87
uploaded by medbooksvn
NBME Top Concepts ➤ Respiratory Regional Circulation for the USMLE

Diffusion Limitation

Creative Commons Attribution License 4.0

Hypoxemia

Respiratory: Regional Circulation for the USMLE

88
NBME Top Concepts ➤ Respiratory Regional Circulation for the USMLE

NBME Style Question


A 5-year-old boy is brought to the office due to intermittent episodes of “blue-ness.” He was
adopted and his medical records are unknown. The boy assumes a squatting position to mitigate his
”blue episodes.” Physical examination reveals a prominent right ventricular impulse and a harsh
systolic murmur. Which of the following embryological events is the most likely mechanism that
caused this patient’s condition?

A. Anomalous pulmonary venous return


B. Aortic arch constriction
C. Deviation of the infundibular septum
D. Endocardial cushion defect
E. Open ductus arteriosus

Tetralogy of Fallot

Respiratory: Regional Circulation for the USMLE

89
uploaded by medbooksvn
NBME Top Concepts ➤ Respiratory Physical Exam MCQs

Physical Exam in USMLE Vignettes


USMLE Questions will have the physical exam findings after vital signs & before labs.

Understanding the USMLE ”anatomy” of a vignette comfort in NBME questions.

USMLE Test Taking Strategy: Physical Exam of the Lung

Creative Commons Attribution License 4.0

Physical Exam of the Lung

Respiratory: Physical Exam MCQs

90
NBME Top Concepts ➤ Respiratory Physical Exam MCQs

NBME Style Question


A 72-year-old woman comes to the physician because of a 3-day history of fever, shortness of
breath, difficulty swallowing, chest pain, and cough. She is frail. Physical examination shows
tachypnea and equal pulses bilaterally. Percussion of the chest shows dullness over the right lower
lung field. Laboratory studies show arterial hypoxemia and decreased PCO2. A chest x-ray shows an
area of opacification in the lower region of the right lung. Which of the following is the most likely
cause of this patient's condition?

A. Alveolar proteinosis
B. Aspiration
C. Cigarette smoking
D. Emphysema
E. Vasculitis

Anatomy Integration
Think aspiration pneumonia in the following
patients on USMLE:
• Altered consciousness dementia, drug
intoxication
• Dysphagia due to neuro deficits stroke
• Large stomach volume & reflux.

Creative Commons Attribution License 4.0

NBME Style Question


An 18-year-old male presents with sudden onset of chest pain. He recently was at a football game
with his friends. He has smoked a pack of cigarettes daily. He has pain worse on inspiration. His RR
is elevated. Physical examination shows a thin, tall patient in acute distress. SpO2 = 88%. The
trachea is midline. There are decreased breath sounds and hyper-resonance to percussion on the R
lung field. Which of the following mechanisms explain these patient's symptoms?

A. Apical subpleural bleb rupture


B. Hyperinflation and air-trapping
C. Panacinar emphysema
D. Trauma leading to tension pneumothorax

Respiratory: Physical Exam MCQs

91
uploaded by medbooksvn
NBME Top Concepts ➤ Respiratory Physical Exam MCQs

Anatomy Integration

Respiratory: Physical Exam MCQs

92
NBME Top Concepts ➤ Respiratory Restrictive vs. Obstructive Disease

Concept Differentiation

NBME Style Question


A 35-year-old was hospitalized for abnormal liver enzymes. He has a history of jaundice as a
neonate. His past chart review shows a decreased FEV1 on spirometry. He is a lifetime non-smoker.
Which of the following pathophysiologic mechanisms explains this patient’s underlying condition?

A. Increase ROS causing breakdown of lung elastin.


B. Decrease in lung defense mechanisms.
C. Decrease in proteases increasing lung compliance.
D. Decrease elastase activity.

Emphysema

Respiratory: Restrictive vs. Obstructive Disease

93
uploaded by medbooksvn
NBME Top Concepts ➤ Respiratory Restrictive vs. Obstructive Disease

NBME Style Question


A 38-year-old female presents with shortness of breath. She has decreased exercise tolerance. He
currently has a BMI of 41 kg/m2. His RR is 22/min. Physical exam is notable for central obesity.
Which of the following spirometry changes would be most likely seen in this patient?

A. Decreased FRC
B. Increased Residual Volume
C. Decreased FEV1 : FVC ratio
D. High FEV1
E. Increased chest-wall compliance

Functional Residual Capacity for the USMLE

Creative Commons Attribution License 4.0

Graphs on the USMLE

Creative Commons Attribution License 4.0

Respiratory: Restrictive vs. Obstructive Disease

94
NBME Top Concepts ➤ Respiratory Restrictive vs. Obstructive Disease

USMLE Vignette

Creative Commons Attribution License 4.0

Asthma

Respiratory: Restrictive vs. Obstructive Disease

95
uploaded by medbooksvn
NBME Top Concepts ➤ Respiratory Restrictive vs. Obstructive Disease

USMLE Vignette

Respiratory: Restrictive vs. Obstructive Disease

96
NBME Top Concepts ➤ Respiratory Lung Tumors

USMLE Test Taking Strategy


USMLE Questions will presentations related to:
• Constitutional Symptoms
• Paraneoplastic Syndromes:
• Physiology tie-ins.

NBME Style Question


A 59-year-old male presents with right scapular pain. He also is feeling numb in his fourth and fifth
fingers in his right forearm. He has had cough with no sputum, weight loss, and occasional night
sweats. Travel history is insignificant. He is a 2-pack year smoker. Physical exam is notable for
orbital puffiness. Which of the following is the most likely etiology of this patient’s symptoms?

A. Airway obstruction.
B. Superior sulcus tumor.
C. Pleural effusion.
D. Pericardial effusion.

Small Cell Lung Cancer


A 73-year-old smoker comes in with a dry cough and weight loss. He is found to have a large lung
mass on imaging and a tumor with a high mitotic count and increased nuclear to cytoplasmic ratio.
His sodium levels are low. What is the likely stain which will be positive in this patient’s oncological
lesion?
• Neuroendocrine in origin NCAM, CD56, chromogranin, synaptophysin.

Small Round Blue Cell Tumors (Neural Crest or Neuro-Endocrine in Embryology)

Creative Commons Attribution License 4.0

Respiratory: Lung Tumors

97
uploaded by medbooksvn
NBME Top Concepts ➤ Respiratory Lung Tumors

Small Cell

Squamous Cell

Creative Commons Attribution License 4.0

Respiratory: Lung Tumors

98
NBME Top Concepts ➤ Respiratory Lung Tumors

Adenocarcinoma

Respiratory: Lung Tumors

99
uploaded by medbooksvn
NBME Top Concepts ➤ Respiratory Acute Respiratory Distress Syndrome

NBME Style Question


A patient is admitted for pancreatitis. He has a history of alcohol use. He is noted to have mild
respiratory distress on admission. On day three of hospitalization, he develops hypoxemia and is
transferred to the ICU. He is intubated and chest-x-ray reveals new bilateral pulmonary opacities.
Which of the following parameters is most likely to be normal in this patient?

A. Capillary permeability
B. Work of breathing
C. V/Q ratio
D. Lung compliance
E. Pulmonary capillary wedge pressure

Acute Respiratory Distress Syndrome

Creative Commons Attribution License 4.0

NBME Style Question


A 24-hour neonate presents to the neonatal ICU for tachypnea and hypoxia. The patient was born
at 32 weeks to a mother who had T2 DM. Prenatal ultrasounds were notable for polyhydramnios. The
patient was delivered via C-section. On exam, the patient has peri-oral cyanosis and a mild murmur at
the infraclavicular border with a rumbling sound. Which of the following pathophysiologic
mechanisms describe this patient's clinical state?

A. Uncontrolled pulmonary artery smooth muscle proliferation.


B. Increased L to R cardiac shunting.
C. Increase alveolar wall surface tension.
D. Oligohydramnios leading to low surfactant.

Respiratory: Acute Respiratory Distress Syndrome

100
NBME Top Concepts ➤ Respiratory Acute Respiratory Distress Syndrome

Neonatal Respiratory Distress Syndrome

Creative Commons Attribution License 4.0

Respiratory: Acute Respiratory Distress Syndrome

101
uploaded by medbooksvn
NBME Top Concepts ➤ Respiratory Answer Key

A-a Gradient (Hypoxemia)


A 52-year-old male is found unresponsive. His medical history is unknown. On PE, he is
unresponsive despite sternal rub. He is cool. His laboratory studies are notable for low pH, low PaO ,
low HCO , high CO . Which of the following diagnoses is most likely to be seen in this patient?

A. Respiratory Alkalosis
B. Metabolic Alkalosis
C. Hyperoxia
D. High lactate
A 45-year-old was hospitalized for hypoxemia. The patient has a history of smoking. He is noted to
have increased sputum production. His pulse oximetry is noted to be <90%. He has increased work
of breathing on the exam and respiratory distress. He breathes with pursed lips and has a copious
purulent cough. Which of the following mechanisms is the primary cause of hypoxemia?

A. Alveolar hypoventilation
B. Decreased oxygen extraction
C. Primary diffusion impairment
D. V/Q mismatch

Regional Circulation for the USMLE


A 62-year-old male comes to the clinic due to a 6-month history of progressive exertional dyspnea.
He has no environmental exposures. The patient dies of respiratory failure. Autopsy findings is
notable for heterogenous lung parenchyma with predominantly subpleural areas of dense collagen
deposition, lymphocyte infiltration, and fibroblast proliferation. Which of the following is the most
likely diagnosis?

A. Alpha-1 anti-trypsin
B. Chronic bronchitis
C. Asbestosis
D. Idiopathic pulmonary fibrosis
E. Chronic hypersensitivity pneumonitis
A 5-year-old boy is brought to the office due to intermittent episodes of “blue-ness.” He was
adopted and his medical records are unknown. The boy assumes a squatting position to mitigate his
”blue episodes.” Physical examination reveals a prominent right ventricular impulse and a harsh
systolic murmur. Which of the following embryological events is the most likely mechanism that
caused this patient’s condition?

A. Anomalous pulmonary venous return


B. Aortic arch constriction
C. Deviation of the infundibular septum
D. Endocardial cushion defect
E. Open ductus arteriosus

Respiratory: Answer Key

102
NBME Top Concepts ➤ Respiratory Answer Key

Physical Exam MCQs


A 72-year-old woman comes to the physician because of a 3-day history of fever, shortness of
breath, difficulty swallowing, chest pain, and cough. She is frail. Physical examination shows
tachypnea and equal pulses bilaterally. Percussion of the chest shows dullness over the right lower
lung field. Laboratory studies show arterial hypoxemia and decreased PCO2. A chest x-ray shows an
area of opacification in the lower region of the right lung. Which of the following is the most likely
cause of this patient's condition?

A. Alveolar proteinosis
B. Aspiration
C. Cigarette smoking
D. Emphysema
E. Vasculitis

An 18-year-old male presents with sudden onset of chest pain. He recently was at a football game
with his friends. He has smoked a pack of cigarettes daily. He has pain worse on inspiration. His RR
is elevated. Physical examination shows a thin, tall patient in acute distress. SpO2 = 88%. The
trachea is midline. There are decreased breath sounds and hyper-resonance to percussion on the R
lung field. Which of the following mechanisms explain these patient's symptoms?

A. Apical subpleural bleb rupture


B. Hyperinflation and air-trapping
C. Panacinar emphysema
D. Trauma leading to tension pneumothorax

Restrictive vs. Obstructive Disease


A 35-year-old was hospitalized for abnormal liver enzymes. He has a history of jaundice as a
neonate. His past chart review shows a decreased FEV1 on spirometry. He is a lifetime non-smoker.
Which of the following pathophysiologic mechanisms explains this patient’s underlying condition?

A. Increase ROS causing breakdown of lung elastin.


B. Decrease in lung defense mechanisms.
C. Decrease in proteases increasing lung compliance.
D. Decrease elastase activity.

Respiratory: Answer Key

103
uploaded by medbooksvn
NBME Top Concepts ➤ Respiratory Answer Key

Restrictive vs. Obstructive Disease


A 38-year-old female presents with shortness of breath. She has decreased exercise tolerance. He
currently has a BMI of 41 kg/m2. His RR is 22/min. Physical exam is notable for central obesity.
Which of the following spirometry changes would be most likely seen in this patient?

A. Decreased FRC
B. Increased Residual Volume
C. Decreased FEV1 : FVC ratio
D. High FEV1
E. Increased chest-wall compliance

Lung Tumors
A 59-year-old male presents with right scapular pain. He also is feeling numb in his fourth and fifth
fingers in his right forearm. He has had cough with no sputum, weight loss, and occasional night
sweats. Travel history is insignificant. He is a 2-pack year smoker. Physical exam is notable for
orbital puffiness. Which of the following is the most likely etiology of this patient’s symptoms?

A. Airway obstruction
B. Superior sulcus tumor
C. Pleural effusion
D. Pericardial effusion

Acute Respiratory Distress Syndrome


A patient is admitted for pancreatitis. He has a history of alcohol use. He is noted to have mild
respiratory distress on admission. On day three of hospitalization, he develops hypoxemia and is
transferred to the ICU. He is intubated and chest-x-ray reveals new bilateral pulmonary opacities.
Which of the following parameters is most likely to be normal in this patient?

A. Capillary permeability
B. Work of breathing
C. V/Q ratio
D. Lung compliance
E. Pulmonary capillary wedge pressure

Respiratory: Answer Key

104
NBME Top Concepts ➤ Respiratory Answer Key

Acute Respiratory Distress Syndrome


A 24-hour neonate presents to the neonatal ICU for tachypnea and hypoxia. The patient was born
at 32 weeks to a mother who had T2 DM. Prenatal ultrasounds were notable for polyhydramnios.
The patient was delivered via C-section. On exam, the patient has peri-oral cyanosis and a mild
murmur at the infraclavicular border with a rumbling sound. Which of the following pathophysiologic
mechanisms describe this patient's clinical state?

A. Uncontrolled pulmonary artery smooth muscle proliferation


B. Increased L to R cardiac shunting
C. Increase alveolar wall surface tension
D. Oligohydramnios leading to low surfactant

Respiratory: Answer Key

105
uploaded by medbooksvn
CHAPTER 6

High Yield USMLE


Images & PE Findings:
Respiratory

106
High Yield Images & PE Findings ➤ Respiratory Overview

High Yield images & PE Features

• Physical Exam Findings in Select Lung Diseases


• Pneumonia & Pneumothorax
• Clubbing
• Acute Respiratory Distress Syndrome

NBME Style Question


A six week old child presents in winter with cough and congestion for 3 days. He has subcostal
retractions and is tachypneic. His oxygen saturation is 91%. PE shows diffuse expiratory wheezes
and rales. CXR shows no focality. Which of the following is the most likely pathogen behind his
current condition?

A. Adenovirus
B. Mycoplasma
C. CMV
D. RSV
E. HiB

Understanding Lung PE for the USMLE

Creative Commons Attribution License 4.0

Respiratory: Overview

107
uploaded by medbooksvn
High Yield Images & PE Findings ➤ Respiratory Overview

USMLE Vignette

A 3-year-old male presents with acute onset fever and


tachypnea. A translator is used during the exam as the patient
and his family have recently immigrated from overseas. On
exam he has stridor, drooling , and is laying on his forearms. A
lateral X-ray is shown. What is the likely morphology of the
organism behind his symptoms?
• Gram negative coccobacilli
• Encapsulated, requiring Factor V (NAD+) & X
(hematin)
• What is the characteristic of the vaccine?
• Capsular polysaccharide + protein conjugate
• T cell activation & class switching.
Creative Commons Attribution License 4.0

Epiglottitis: Anatomy integration

Creative Commons Attribution License 4.0

Respiratory: Overview

108
High Yield Images & PE Findings ➤ Respiratory Overview

Encapsulated Bacteria Vaccines

A child with history of multiple admissions due to pain in his extremities presents with fever,
tachycardia, and hypotension. He is found to have a blood culture positive for a gram-positive lancet
shaped diplococci. What is the likely pathogenesis?
• Sickle cell functionally aplenia due to repeat microinfarction of splenic vascular beds
• Decreased immunity towards encapsulated bacteria.

Respiratory: Overview

109
uploaded by medbooksvn
High Yield Images & PE Findings ➤ Respiratory Pneumothorax

NBME Style Question


An 18-year-old male presents with sudden onset chest pain. There is no trauma. He has pleuritic
chest pain. He has an uncle who had similar presentations however he passed away three years
prior due to aortic dissection. On patient his R chest is hyper-resonant. Which of the following
mechanisms contributed to this patient’s presentation?

A. Apical subpleural bleb rupture


B. Penetrating Trauma
C. Hereditary intra-pulmonary fibrosis
D. Pan-acinar Emphysema
E. Air trapping

Pneumothorax

Pneumothorax for the USMLE

Respiratory: Pneumothorax

110
High Yield Images & PE Findings ➤ Respiratory Pneumothorax

Atelectasis
Big Picture: Loss of lung volume due to alveolar collapse

A 28-year-old female presents with cough and


fever. She is found to have chest-pain that is
worse on inspiration. She has also had a two-
week history of arthralgias bilaterally. A CXR is
performed and findings are shown. An ANA
shows a speckled pattern. What is the most-
sensitive lab test which may confirm the
diagnosis?
• Anti-dsDNA antibody or Anti-Sm (non-histone
nuclear proteins).

Respiratory: Pneumothorax

111
uploaded by medbooksvn
High Yield Images & PE Findings ➤ Respiratory Pneumothorax

Lupus

Creative Commons Attribution License 4.0

Lupus

Creative Commons Attribution License 4.0

Respiratory: Pneumothorax

112
High Yield Images & PE Findings ➤ Respiratory Pneumothorax

Renal Manifestations of Lupus for the USMLE

Creative Commons Attribution License 4.0

Respiratory: Pneumothorax

113
uploaded by medbooksvn
High Yield Images & PE Findings ➤ Respiratory Pneumonia for the USMLE

Pneumonia
A 77-year-old male brought in by EMS after awakening with fever, chills, and chest pain worse on
inspiration. He has copious sputum production. On auscultation, the patient has decreased breath
sounds in the right lower lung field. What are other physical exam features which may be present?
• Crackles
• Dullness to percussion
• Bronchial breath sounds
• Increased tactile fremitus

Anatomic Distributions of Various Forms of PNA

Ivan Damjanov, MD, PhD: Pathophysiology, Philadelphia, Saunders Elsevier, 2009, p 171, Fig. 5-23.

Lobar Pneumonia

Creative Commons Attribution License 4.0

Respiratory: Pneumonia for the USMLE

114
High Yield Images & PE Findings ➤ Respiratory Pneumonia for the USMLE

Microbiology Integration

Walker BR, Colledge NR, Ralston SH, Penman ID: Davidson’s Principles and Practice of Medicine, 22nd ed, St. Louis, Churchill Livingstone Elsevier, 2014, p 136, Fig. 6.3.

Think Like the Test Maker


• Big Picture:
• Fever, SOB, pleuritic chest pain, and sputum + focal lung exam

• USMLE loves to test the microbiology:


• Neonate + pneumonia =
• GBS and EColi

• College kid + pneumonia =


• Mycoplasma

• Alcoholic + pneumonia =
• Anaerobic (Peptostreptococcus, Fusobacterium, Prevotella, Bacteroides)

• Hereditary disorder + pancreatic insufficiency + recurrent pneumonia =


• Cystic Fibrosis Staph-Aureus (young), Pseudomonas, and B. Cepacia.

• HIV + pneumonia =
• Pneumocystis Pneumonia (“ground glass”, “silver stain”, “TMP-SMX”)

• Smoker + COPD + pneumonia =


• Legionella (“silver stain”), H. Influenzae, M. Catarrhalis

Respiratory: Pneumonia for the USMLE

115
uploaded by medbooksvn
High Yield Images & PE Findings ➤ Respiratory Pneumonia for the USMLE

Microscopic Pathophysiology of PNA for the USMLE

Bronchopneumonia
• Big Picture:
• Acute bronchitis subsequent extension of the infection into surrounding alveoli

Creative Commons Attribution License 4.0

Respiratory: Pneumonia for the USMLE

116
High Yield Images & PE Findings ➤ Respiratory Pneumonia for the USMLE

NBME Style Question


A 20 year old male presents with twelve days of cough, low grade fever and malaise. Cough is
non-productive. His lung examination shows scattered rales. He is also noted to have anemia, and
elevated LDH. Scleral icterus is also noted on repeat eye exam. What is the likely mechanism behind
this presentation?

A. Type 1 HS.
B. Type 2 HS.
C. Type 3 HS.
D. Type 4 HS.

Heme Integration: Autoimmune Hemolytic Anemia

Creative Commons Attribution License 4.0

Respiratory: Pneumonia for the USMLE

117
uploaded by medbooksvn
High Yield Images & PE Findings ➤ Respiratory Pneumonia for the USMLE

Mycoplasma Pneumoniae on the USMLE

Creative Commons Attribution License 4.0

NBME Style Question


A 5-year-old male presents with cough at night. He has had this for the past five months. The
patient has a history of seasonal allergies and scaly rash on the flexural surfaces. His lung exam
shows no wheezing. Spirometry is notable for a decreased FEV1. The patient may likely have which
cytokine elevated in the blood?

A. IL-1
B. IL-5
C. IL-12
D. IFN – gamma

Pathology of Asthma

• What characteristic pathological hallmarks may be seen in


the sputum?
• Curschmann spirals
• epithelium shed from mucus plugs
• Charcot Leyden crystals
• Needle crystalized aggregates of MBP from eosinophil
• Hyperplasia of mucosal glands, along with smooth muscle
hypertrophy.

Creative Commons Attribution License 4.0

Respiratory: Pneumonia for the USMLE

118
High Yield Images & PE Findings ➤ Respiratory Pneumonia for the USMLE

Asthma
What would microscopic pathology of a bronchus show?
• Mucus in lumen
• Inflammation and basement
• Membrane thickening
• Enlarged mucous glands
• Smooth muscle hyperplasia.

van Damjanov, MD, PhD: Pathology for the Health Professions, 4th ed, Philadelphia, Saunders Elsevier, 2012, p 178, Fig. 8-16.

Pulsus Paradoxes

Creative Commons Attribution License 4.0

Respiratory: Pneumonia for the USMLE

119
uploaded by medbooksvn
High Yield Images & PE Findings ➤ Respiratory Clubbing

Obstructive Lung Disease


Patient with Cystic Fibrosis presents with foul-smelling, green sputum cough. A CT of chest is
ordered. What is the likely diagnosis?
• Bronchiectasis

Creative Commons Attribution License 4.0

Pathophysiology of Pronchiectasis

Creative Commons Attribution License 4.0

Respiratory: Clubbing

120
High Yield Images & PE Findings ➤ Respiratory Acute Respiratory Distress Syndrome

NBME Style Question


A 50-year-old female is admitted for CVA tenderness and fever. On day three of hospitalization she
is admitted to the ICU as the patient has increased hypoxemia. The patient has multi-focal airspace
disease on X-ray which are new. What additional pathological finding may be found in this patient?

A. Exudative air space disease


B. Transudative air space disease
C. Non-caseating granulomas
D. Red infarct of lobar segment of lung

ARDS
A patient presents with pancreatitis and is suddenly intubated with a prolonged course in the
intensive care unit. He requires increased oxygen requirement, and has new infiltrates on CXR. What
is the likely pathological finding this patient may have on alveolar analysis?
• Hyaline membranes surrounding alveolo-capillary membrane ARDS which decreases lung
compliance
• What is pulmonary capillary wedge pressure?
• Normal
• Elevated PCWP more suggestive of left ventricular failure.

Pathogenesis of ARDS

Ivan Damjanov, MD. PhD: Pathophysiology, Philadelphia, Saunders Elsevier, 2009, p 169, Fig. 5-20.

Respiratory: Acute Respiratory Distress Syndrome

121
uploaded by medbooksvn
High Yield Images & PE Findings ➤ Respiratory Acute Respiratory Distress Syndrome

Acute Respiratory Distress Syndrome

Physiology Integration: Pulmonary Compliance

Respiratory: Acute Respiratory Distress Syndrome

122
High Yield Images & PE Findings ➤ Respiratory Answer Key

Pulmonary Physical Exam for the USMLE


A six week old child presents in winter with cough and congestion for 3 days. He has subcostal
retractions and is tachypneic. His oxygen saturation is 91%. PE shows diffuse expiratory wheezes
and rales. CXR shows no focality. Which of the following is the most likely pathogen behind his
current condition?

A. Adenovirus
B. Mycoplasma
C. CMV
D. RSV
E. HiB

Pneumothorax
An 18-year-old male presents with sudden onset chest pain. There is no trauma. He has pleuritic
chest pain. He has an uncle who had similar presentations however he passed away three years
prior due to aortic dissection. On patient his R chest is hyper-resonant. Which of the following
mechanisms contributed to this patient’s presentation?

A. Apical subpleural bleb rupture


B. Penetrating Trauma
C. Hereditary intra-pulmonary fibrosis
D. Pan-acinar Emphysema
E. Air trapping

Pneumonia for the USMLE


A 20 year old male presents with twelve days of cough, low grade fever and malaise. Cough is
non-productive. His lung examination shows scattered rales. He is also noted to have anemia, and
elevated LDH. Scleral icterus is also noted on repeat eye exam. What is the likely mechanism behind
this presentation?

A. Type 1 HS.
B. Type 2 HS.
C. Type 3 HS.
D. Type 4 HS.

Respiarotory: Answer Key

123
uploaded by medbooksvn
High Yield Images & PE Findings ➤ Respiratory Answer Key

Pneumonia for the USMLE


A 5-year-old male presents with cough at night. He has had this for the past five months. The
patient has a history of seasonal allergies and scaly rash on the flexural surfaces. His lung exam
shows no wheezing. Spirometry is notable for a decreased FEV1. The patient may likely have which
cytokine elevated in the blood?

A. IL-1
B. IL-5
C. IL-12
D. IFN – gamma

Acute Respiratory Distress Syndrome


A 50-year-old female is admitted for CVA tenderness and fever. On day three of hospitalization she
is admitted to the ICU as the patient has increased hypoxemia. The patient has multi-focal airspace
disease on X-ray which are new. What additional pathological finding may be found in this patient?

A. Exudative air space disease


B. Transudative air space disease
C. Non-caseating granulomas
D. Red infarct of lobar segment of lung

Respiarotory: Answer Key

124
CHAPTER 7

NBME Top Concepts:


Endocrinology

125
uploaded by medbooksvn
NBME Top Concepts ➤ Endocrinology Hormone Signaling

Test Taking Strategy: Approaching Hormone Signaling


Group hormones based on area of release.
• Anterior pituitary
• Posterior pituitary
• Thyroid gland
Define the hormone:
• Integrate the mechanism of action
All Endocrinology questions:
• Think about feedback!

NBME Style Question


A middle-aged male presents with shortness of breath while lying flat. He is found to have a 3/6
holosystolic murmur heard best at the apex. Dilated cardiomyopathy is suspected. Which of the
following physical exam findings would be less likely to be associated with this presentation?

A. Apical impulses shifted to the axillary line at the sixth intercostal space.
B. S4 gallop.
C. Bibasilar crackles.
D. Peripheral edema.
E. Hepatomegaly.

GPCR - Gs

Endocrinology: Hormone Signaling

126
NBME Top Concepts ➤ Endocrinology Hormone Signaling

GPCR - Gq

HyGuru Test Taking Tip: Webs of Knowledge

Tyrosine MAP Kinase

Endocrinology: Hormone Signaling

127
uploaded by medbooksvn
NBME Top Concepts ➤ Endocrinology Hormone Signaling

Tyrosine Kinase Associated JAK STAT

Endocrinology: Hormone Signaling

128
NBME Top Concepts ➤ Endocrinology Thyroid Disorders

Test Taking Strategy: Hyper vs. Hypothyroidism


If you are rev-ed up…
Hyperthyroidism
• Graves = exophthalmos
• Arrythmias
If you are slowed down…
Hypothyroidism
• Hypercholesterolemia

NBME Style Question


A 29-year-old woman presents with nervousness, heat intolerance, and weight loss. Laboratory
examination reveals elevated serum thyroxine (T4) and triiodothyronine (T3) levels, while the level of
serum thyroid-stimulating hormone (TSH) is decreased. Histologic sections from her thyroid gland
reveal increased cellularity with scalloping of the colloid at the margins of the follicles. Which of the
following types of autoantibodies is most specific for this individual disease?

A. Antimicrosomal antibodies
B. Antithyroglobulin antibodies
C. Antithyroid peroxidase antibodies
D. TSH-receptor-blocking antibodies
E. TSH-receptor-stimulating antibodies

Thyroid Hormone
Where is it secreted from?
• Follicles in the thyroid.
•Remember these are the only steroid like hormones which are pre-synthesized and stored.
• What is more secreted form?
• T4
What is rate limiting enzyme for thyroid synthesis?
• Thyroid peroxidase:
• Oxidation
• Organification
• Coupling
What is mechanism behind thyroid hormone increasing the basal metabolic rate?
• Increases Na/K ATPase activity
• What type of receptor does thyroid bind to?
• Intracellular.

Endocrinology: Thyroid Disorders

129
uploaded by medbooksvn
NBME Top Concepts ➤ Endocrinology Thyroid Disorders

Tyrosine Kinase Associated JAK STAT

Thyroid & Pregnancy


A female on OCP or who is pregnant will have which type of thyroid profile?
• High Total T4 and normal free T3 and T4
Key point is that estrogen will increase thyroid binding globulin increasing total T4 however not
affecting the free hormone
• Free hormone is important for feedback

Endocrinology: Thyroid Disorders

130
NBME Top Concepts ➤ Endocrinology Hypothyroid

Hypothyroid
Pt with weight gain, and fatigue, complaining of weakness, has heavy periods and deepening of the
voice. How are their lipid profiles?
• High LDL
A patient who has not seen a physician in 20 years presents with AMS and dry skin. DTR is delayed
and she has cool, yellowed skin. Non-pitting edema in her face and extremities. What is the likely
diagnosis?
• Myxedema coma
• Non-pitting edema due to increase in GAGs and MPS.

Pathophysiology of Hypothyroidism

USMLE Point:
Another thing to keep in mind for hypothyroid questions is a high-yield psychiatry differential.
What may be the likely DSM diagnosis that may mimic hypothyroid?
• Depression can have fatigue, depressed mood, weight gain
How do you tease them out?
• Somatic symptoms such as constipation and physical exam signs like decreased reflexes are
not necessarily characteristic of depression and point more to hypothyroid.

Hashimoto’s
What are the antibodies positive on lab testing?
• TPO Ab and antimicrosomal Ab
What will the FNA of the thyroid show?
• Lymphocytic infiltration with germinal centers along with Hurthle cells
• Atrophic epithelial cells with eosinophilic metaplasia and prominent nucleoli.

Endocrinology: Hypothyroid

131
uploaded by medbooksvn
NBME Top Concepts ➤ Endocrinology Hypothyroid

Hashimoto Thyroiditis

Creative Commons Attribution License 4.0

Endocrinology: Hypothyroid

132
NBME Top Concepts ➤ Endocrinology Hyperthyroid

Hyperthyroid
Female with weight loss, tremor, and has palpitations and occasional chest pain. What psychiatric
disorder may be also considered as a psychiatry differential?
• Anxiety
• Typically no weight loss with anxiety
USMLE Presentations (besides Graves):
• Watch for the obese patient who takes thyroid hormone for weight loss
• Pt on the test with hyperthyroidism can also present as atrial fibrillation due to thyroid effect on
the heart.

Delineating Lab Tests in USMLE Thyroid Questions


Are patients with hyperthyroid going to be “revved up” or “chilled out”?
• Revved up!

Endocrinology: Hyperthyroid

133
uploaded by medbooksvn
NBME Top Concepts ➤ Endocrinology Hyperthyroid

Grave’s Disease
USMLE points:
• IgG to TSH receptor (TSI antibody)
• Stimulatory antibody
What physical exam finding may be pathognomonic for Grave’s?
• Exophthalmos (pathognomonic). Mechanism?
• Proliferation of retro-orbital fibroblasts causing an increase in GAGs
What does FNA of the thyroid show?
• Scalloped appearance of colloid

Scalloped Thyroid

Creative Commons Attribution License 4.0

Endocrine Pharmacology
What is the mechanism of the pharmacological agents used to decrease thyroid synthesis?
• Block thyroid peroxidase
• Methimazole & PTU
Hyperthyroidism + on methimazole or PTU fever, sore throat. What lab test may allude to the
underlying diagnosis?
• Low WBC (check CBC).

Endocrine Anatomy
A patient with a recent history of follicular thyroid cancer presents for post-operative evaluation after
thyroidectomy. He is noted to have difficulty in articulation, speaks in a soft, muffled, and hoarse
voice. The affected structure is related to which of the following embryological derivatives?

A. Pharyngeal arch 6
B. Pharyngeal pouch 4
C. Pharyngeal cleft 6
D. Pharyngeal arch 3
E. Pharyngeal pouch 3

Endocrinology: Hyperthyroid

134
NBME Top Concepts ➤ Endocrinology PTH and Calcium

Parathyroid Hormone
Where is the hormone released from?
• Chief cells of the parathyroid gland
• What pouches are they derived from?
• 3rd and 4th
What effect does it have on the bone?
• Increases calcium and phosphate release from bone
• What is the mechanism?
• PTH is going to bind to PTH receptor on osteoblasts
• Induces osteoblasts to make RANK-L
• RANK-L then binds to a RANK receptor on premature osteoclasts with help of IL-1
they create an acidic environment to increase PO4 and Ca concentration in the blood.

Osteoblast Osteoclast RANK

Estrogen increases
osteoprogerin inhibits
RANK/RANK-L inhibition of
osteoclast differentiation
protective against osteoporosis

Creative Commons Attribution License 4.0

Parathyroid Hormone
What effect does it have on the kidney?
• Increases Ca2+ reabsorption where?
• Distal convoluted tubule
• Inhibits phosphate reabsorption where?
• Proximal convoluted tubule
• Stimulates active vitamin D synthesis. Mechanism?
• Stimulation of 1-alpha-hydroxylase
• 25 Vit D 1,25 Vit D.

Endocrinology: PTH and Calcium

135
uploaded by medbooksvn
NBME Top Concepts ➤ Endocrinology PTH and Calcium

Parathyroid Review

Active Vitamin D Integration


What is the difference between D2 and D3?
• D2 is found in vegetables and dietary supplements
• D3 is the form we sunbathe in:
• Mechanism: 7 dehydrocholesterol which is in the skin gets converted to D3
What is the storage form known as:
• 25, OH Vitamin D in the liver, goes to the kidney to become 1,25 OH Vitamin D
What effect does the active 1,25 Vitamin D have on the gut?
• Increases Ca and PO4 reabsorption.

Vitamin D Pathophysiology
A patient with sarcoidosis on the USMLE may present with hypercalcemia. What is the mechanism?
• Granulomas have intrinsic 1-alpha-hydroxylase activity
A child presents with poor growth and recurrent respiratory infections. He is found to have low
Vitamin D. His most recent admission was for pseudomonal PNA and he also has foul smelling
stools. What is the likely etiology behind his low Vitamin D?
• Pancreatic insufficiency causing malabsorption Cystic Fibrosis.

Endocrinology: PTH and Calcium

136
NBME Top Concepts ➤ Endocrinology PTH and Calcium

1a Hydroxylase

Creative Commons Attribution License 4.0

Parathyroid Adenoma
Pt presents with increased thirst and constipation. He has a history of peptic ulcer disease and
kidney stones. He as increased ALP and urine cAMP are elevated. Why does this patient have
increased cAMP in his urine?
• cAMP is high because PTH is a Gs mechanism
Lab values with primary hyperparathyroidism:
• High calcium
• Low phosphate
What MEN is parathyroid adenoma related to:
• MEN-1

Endocrinology: PTH and Calcium

137
uploaded by medbooksvn
NBME Top Concepts ➤ Endocrinology MEN Syndromes

NBME Style Question


A 23-year-old man sees a physician because he was awakened on several occasions by severe
headaches, anxiety, and heart palpitations. Vital signs are within normal limits. On physical
examination, he has pectus excavatum, a high arched palate, bilateral pes cavus, and scoliosis. He is
noted to have oral lesions on the buccal surface. Which of the following laboratory measures would
likely be elevated in this patient?

A. Calcitonin
B. Calcium
C. Insulin
D. Phosphate
E. Thyroglobulin

MEN Syndromes

Marfan’s Syndrome Integration


Marfan’s Syndrome: Marfanoid Habitus:
• Chromosome 15 • Homocystinuria (lens downward)
• Patho: Fibrillin mutation (elastin) • MEN 2b
• Ophtho: Lens upward dislocation
• Cardiac: Aortic dilation/regurgitation
• Resp: Apical bleb rupture

Endocrinology: MEN Syndromes

138
NBME Top Concepts ➤ Endocrinology MEN Syndromes

Adrenal Medulla
What are the cells that secrete this?
• Chrommafin cells. Different from ECL which release?
• Histamine
• Derived from neural crest
What is primarily secreted?
• 80% epinepherine and 20% norepinephrine
What is the metabolic byproducts made by NE and E?
• VMA & HVA.

Pheochromocytoma
A 30-year-old female presents with intermittent HA and palp. She is sweating profusely on exam.
She says this is not the first time she feels like this. What is next best step?
• Urine Metanephrines (and drug screen)
What amino acids are precursors to catecholamines?
• Phenylalanine and tyrosine.

Endocrinology: MEN Syndromes

139
uploaded by medbooksvn
NBME Top Concepts ➤ Endocrinology Islet Cell Tumors

Islet Cell Tumors

Biochemistry Integration of Insulin


What cell releases insulin?
• Beta cells of the pancreas.
Remember this is an ANABOLIC hormone. What are your other anabolic hormones?
• Insulin, Androgens, and GH.
• What biochemical pathways does it increase?
• Glycogenesis and Lipogenesis inhibits lipolysis, and ketoacid formation
• Upregulate glucokinase, glycogen synthase, Acetyl Co-A Carboxylase.
• Increases protein synthesis.
• Via uptake of AA.

Endocrinology: Islet Cell Tumors

140
NBME Top Concepts ➤ Endocrinology DKA vs. HHS

HyGuru Test Taking Tip:Dehydration


USMLE Questions:
• Vital signs:
• Tachycardia
• Hypotension
• +/- tachypnea (if underlying acidosis)
• Physical Exam:
• Dry mucous membranes
• Delayed capillary refill
• Infants:
• No tears when crying
• Sunken fontanelle.

Approach to Diabetes for the USMLE

Endocrinology: DKA vs. HHS

141
uploaded by medbooksvn
NBME Top Concepts ➤ Endocrinology DKA vs. HHS

NBME Style Question


A 27-year-old patient with insulin-dependent diabetes mellitus told his roommate that he could
not afford to refill his insulin prescription. When the roommate returned from a weekend trip on
Sunday evening, he found the patient unresponsive on the couch with deep labored breathing. Which
of the following arterial blood gases taken in the Emergency Department would be expected in this
patient?

A. pH = 7.22; PaCO2 = 60 mmHg, [HCO3–] = 26 mEq/L, Anion Gap = 12 mEq/L


B. pH = 7.02; PaCO2 = 60 mmHg, [HCO3–] = 15 mEq/L, Anion Gap = 12 mEq/L
C. pH = 7.10; PaCO2 = 20 mmHg, [HCO3–] = 6 mEq/L, Anion Gap = 30 mEq/L
D. pH = 7.51; PaCO2 = 49 mmHg, [HCO3–] = 38 mEq/L, Anion Gap = 14 mEq/L
E. pH = 7.40; PaCO2 = 20 mmHg, [HCO3–] = 10 mEq/L, Anion Gap = 26 mEq/L

Pathophysiology of Diabetic Ketoacidosis

Endocrinology: DKA vs. HHS

142
NBME Top Concepts ➤ Endocrinology DKA vs. HHS

Microbiology in DKA
A patient with DKA now presents with facial pain and purulent
nasal discharge. What is the likely diagnosis?
Mucormycosis caused by Mucor & Rhizopus.
• What is the morphology?
• Broad, non-septate hyphae which branch at right
angles.

Creative Commons Attribution License 4.0

HHS vs. DKA USMLE Step 1

Endocrinology: DKA vs. HHS

143
uploaded by medbooksvn
NBME Top Concepts ➤ Endocrinology Diabetes Pharmacology

USMLE Test Taking Strategy: Tackling Pharmacology Questions


USMLE Questions:
When you see a pharmacological agent in a vignette recall the MOA.
≥ 3 medications may indicate a drug interaction question.
• This may be related to:
• CYP related (i.e. inhibitor vs. inducer)
• Delineation of side-effects.

NBME Style Question


A patient with focal-segmental-glomerulosclerosis presents to the physician for routine check-up.
The patient has been having increased weight-gain due to his steroid bursts. He is noted to have an
elevated HgbA1c and is diagnosed with diabetes. His most recent Cr 3.1 mg/dL (H). After initiating
pharmacotherapy, he is noted to have an anion-gap metabolic acidosis. Which of the following
pharmacological agents would explain his current laboratory state?

A. Acarbose
B. Canagliflozin
C. Pioglitazone
D. Sulfonylureas
E. Metformin
F. Exenatide

Diabetes Pharmacology for the USMLE

Endocrinology: Diabetes Pharmacology

144
NBME Top Concepts ➤ Endocrinology Aldosterone Disorders

HyGuru | USMLE Test Taking Strategy: Aldosterone Questions

Aldosterone
1.Brings in Na+
2.Pees out K+
3.Pees out H+

Aldosterone Disorders

Endocrinology: Aldosterone Disorders

145
uploaded by medbooksvn
NBME Top Concepts ➤ Endocrinology Answer Key

Hormone Signalling
A 39-year-old man presents with severe writhing back pain, hematuria, and nausea. An
intravenous pyelogram (IVP) confirms a diagnosis of renal calculi. The presence of strongly
opaque stones on the plain film is suggestive of calcium oxalate stones, which have an increased
incidence with hypophosphatemia. The renal clearance of phosphate is increased by which of the
following hormones?

A. Aldosterone
B. Parathyroid hormone
C. Norepinephrine
D. Vasopressin
E. Angiotensin
F. Calcitonin

Thyroid Disorders
A 29-year-old woman presents with nervousness, heat intolerance, and weight loss. Laboratory
examination reveals elevated serum thyroxine (T4) and triiodothyronine (T3) levels, while the level
of serum thyroid-stimulating hormone (TSH) is decreased. Histologic sections from her thyroid
gland reveal increased cellularity with scalloping of the colloid at the margins of the follicles.
Which of the following types of autoantibodies is most specific for this individual disease?

A. Antimicrosomal antibodies
B. Antithyroglobulin antibodies
C. Antithyroid peroxidase antibodies
D. TSH-receptor-blocking antibodies
E. TSH-receptor-stimulating antibodies

A patient with a recent history of follicular thyroid cancer presents for post-operative evaluation
after thyroidectomy. He is noted to have difficulty in articulation, speaks in a soft, muffled, and
hoarse voice. The affected structure is related to which of the following embryological derivatives?

A. Pharyngeal arch 6
B. Pharyngeal pouch 4
C. Pharyngeal cleft 6
D. Pharyngeal arch 3
E. Pharyngeal pouch 3

Endocrinology: Answer Key

146
NBME Top Concepts ➤ Endocrinology Answer Key

MEN Syndromes

A 23-year-old man sees a physician because he was awakened on several occasions by severe
headaches, anxiety, and heart palpitations. Vital signs are within normal limits. On physical
examination, he has pectus excavatum, a high arched palate, bilateral pes cavus, and scoliosis. He is
noted to have oral lesions on the buccal surface. Which of the following laboratory measures would
likely be elevated in this patient?

A. Calcitonin
B. Calcium
C. Insulin
D. Phosphate
E. Thyroglobulin

DKA vs. HHS


A 27-year-old patient with insulin-dependent diabetes mellitus told his roommate that he could
not afford to refill his insulin prescription. When the roommate returned from a weekend trip on
Sunday evening, he found the patient unresponsive on the couch with deep labored breathing. Which
of the following arterial blood gases taken in the Emergency Department would be expected in this
patient?

A. pH = 7.22; PaCO2 = 60 mmHg, [HCO3–] = 26 mEq/L, Anion Gap = 12 mEq/L


B. pH = 7.02; PaCO2 = 60 mmHg, [HCO3–] = 15 mEq/L, Anion Gap = 12 mEq/L
C. pH = 7.10; PaCO2 = 20 mmHg, [HCO3–] = 6 mEq/L, Anion Gap = 30 mEq/L
D. pH = 7.51; PaCO2 = 49 mmHg, [HCO3–] = 38 mEq/L, Anion Gap = 14 mEq/L
E. pH = 7.40; PaCO2 = 20 mmHg, [HCO3–] = 10 mEq/L, Anion Gap = 26 mEq/L

Diabetes Pharmacology
A patient with focal-segmental-glomerulosclerosis presents to the physician for routine check-up.
The patient has been having increased weight-gain due to his steroid bursts. He is noted to have an
elevated HgbA1c and is diagnosed with diabetes. His most recent Cr 3.1 mg/dL (H). After initiating
pharmacotherapy, he is noted to have an anion-gap metabolic acidosis. Which of the following
pharmacological agents would explain his current laboratory state?

A. Acarbose
B. Canagliflozin
C. Pioglitazone
D. Sulfonylureas
E. Metformin
F. Exenatide

Endocrinology: Answer Key

147
uploaded by medbooksvn
CHAPTER 8

NBME Top Concepts:


Gastroenterology

148
NBME Top Concepts ➤ Gastroenterology Esophageal Issues

Approaching Dysphagia in USMLE Questions

HyGuru Test Taking Tip:Difficulty Swallowing

NBME Style Question


A 56-year-old man presents with weight loss, cough, and diffuse chest pain. He has been having
difficulty swallowing for one month. He is unable to drink or tolerate solid foods. Manometry shows
increased lower esophageal sphincter (LES) pressure. The high LES pressure is consistent even
after swallowing food. A radiological study in this patient would most likely be consistent with which
of the following findings?

a. Double bubble sign on abdominal x-ray.


b. Bird’s beak appearance on barium swallow.
c. Obstructive tumor on barium swallow.
d. Microcolon on abdominal x-ray.
e. Upper esophageal stricture in the esophagus on barium swallow.

Gastroenteroloy: Esophageal Issues

149
uploaded by medbooksvn
NBME Top Concepts ➤ Gastroenterology Esophageal Issues

Achalasia – Failure to Relax the LES

Creative Commons Attribution License 4.0

Mallory Weiss vs. Boerhaave Syndrome


Esophageal Pathologies

Slender female with painful coffee-ground emesis. Examination of the vomitus is consistent with
occult blood positivity. What metabolic abnormality may this patient develop?
• Mallory Weiss Tear metabolic alkalosis.
• Pathophysiology:
• Increased Intra-abdominal pressure linear lacerations at GE junction.

If this pt presents with crepitus near the clavicle region, What is the diagnosis?
• Borheave syndrome air in mediastinum subQ emphysema.

Gastroenteroloy: Esophageal Issues

150
NBME Top Concepts ➤ Gastroenterology Esophageal Issues

Acid Base Integration

Air in the Mediastinum

Gastroenteroloy: Esophageal Issues

151
uploaded by medbooksvn
NBME Top Concepts ➤ Gastroenterology Esophageal Issues

Barret’s Esophagus
Obese male with 8-week h/o of asthma like symptoms complains of intermittent squeezing chest
pain, especially at night. What pathologic term is used to describe the histological change this
patient may have on endoscopy and biopsy of the lower-esophageal region?
• Metaplasia (reversible). What is the tissue?
• Barret’s Esophagus.
• NKSE SCC with goblet cells.
• Barret’s Esophagus Esophageal adenocarcinoma
• Metaplasia Dysplasia Carcinoma.

Creative Commons Attribution License 4.0

Precursor Lesions to Cancer for the USMLE

Gastroenteroloy: Esophageal Issues

152
NBME Top Concepts ➤ Gastroenterology Acid Secretion in the GI system

Gastrointestinal Physiology

Creative Commons Attribution License 4.0

Creative Commons Attribution License 4.0

NBME Style Question


A 38-year-old male patient with a duodenal ulcer is treated successfully with cimetidine. Which of
the following mechanisms best describes the agent started in this patient?

a. Blocks muscarinic receptors on parietal cells


b. Blocks Gq receptors on parietal cells
c. Decreases intracellular cAMP levels
d. Activation of primary-active transport
e. Enhances the action of acetylcholine (ACh) on parietal cells

Gastroenteroloy: Acid Secretion in the GI system

153
uploaded by medbooksvn
NBME Top Concepts ➤ Gastroenterology Acid Secretion in the GI system

Studying Pharmacology for the USMLE

NBME Style Question


The following pairs of hormones are related to parietal cell secretion. Which of the following pairs
most correlate to the gastrointestinal physiology of parietal cells?

a. HCl and intrinsic factor


b. HCl and pepsinogen
c. HCl and HCO3−
d. HCO3− and intrinsic factor
e. Mucus and pepsinogen

Gastroenteroloy: Acid Secretion in the GI system

154
NBME Top Concepts ➤ Gastroenterology Approach to Abdominal Pain on the USMLE

Localization of Abdominal Pain for the USMLE

NBME Style Question


An 18 yr old male complains of nausea and lower abdominal pain. He has no vomiting or diarrhea.
He has been afebrile and denies sexual activity. Urine-analysis is negative for leukocyte esterase and
nitrites. Ultrasound with doppler shows reduced flow to the testes. Which of the following most
likely describes the pathophysiology in this patient?

A. Inflammation of the epididymis due to gram negative diplococci.


B. Colonization of the urethra due to gram negative rods.
C. Spermatic cord twisting with vascular pedicle compromise.
D. Neoplastic change to the testes with increased ɑ-fetoprotein.

Testicular Torsion

Gastroenteroloy: Approach to Abdominal Pain on the USMLE

155
uploaded by medbooksvn
NBME Top Concepts ➤ Gastroenterology Approach to Abdominal Pain on the USMLE

NBME Style Question


A 57-year-old male presents with a primary complaint of erectile dysfunction. After proper
evaluation, the patient is started on daily administration of sildenafil. This medication directly
causes accumulation of which of the following intracellular mediators?

A. Ca2+
B. cGMP
C. AMP
D. Nitrate
E. ANP

Sildenafil

NEJM

Gastroenteroloy: Approach to Abdominal Pain on the USMLE

156
NBME Top Concepts ➤ Gastroenterology Hernias

USMLE Vignette

Gastroenteroloy: Hernias

157
uploaded by medbooksvn
NBME Top Concepts ➤ Gastroenterology Meckel’s Diverticulum

Congenital Pathologies
A 3-year-old presents with abdominal pain, and rectal bleeding. A Tc-99 screen is done which reveals
the presence of gastric tissue 2 feet from the ileocecal valve. What is the likely mechanism behind
this pathology?
• Failure of the vitelline duct to obliterate completely Meckel’s Diverticulum
• Vitelline (omphalomesenteric duct) is the connection between the
yolk-sak and the mid-gut.
• Gastric tissue in the diverticulum ectopic tissue.
• What layers of epithelia is this diverticulum composed of?
• True diverticulum
• Mucosa, submucosa, muscular, serosa layers.

Histology Integration

Gastroenteroloy: Meckel’s Diverticulum

158
NBME Top Concepts ➤ Gastroenterology Meckel’s Diverticulum

Embryology Integration

Creative Commons Attribution License 4.0

Gastroenteroloy: Meckel’s Diverticulum

159
uploaded by medbooksvn
NBME Top Concepts ➤ Gastroenterology Hirschsprung’s Disease

Failure to Pass Meconium

Creative Commons Attribution License 4.0

Gastroenteroloy: Hirschsprung’s Disease

160
NBME Top Concepts ➤ Gastroenterology Inflammatory Bowel Disease

Inflammatory Bowel Disease


A 26-year-old female presents with abdominal pain and diarrhea. She has been having fatty stools
chronically. Barium enema shows narrowing at the level of the jejunum. The patient undergoes EGD.
What will be the histological findings behind this diagnosis?
• Full-thickness inflammation and non-caseating granulomas Crohn’s disease.

Chron’s Disease
What immunological mechanism contributes to the pathophysiology of Chron’s?
• Macrophages secrete IL-12 induces Th1
• Th1 cells secrete IL-2 and IFN gamma
• M (activated) in turn secrete TNF-alpha intestinal cell injury
What is the dermatological association with this disease?
• Pyoderma gangrenosum along with skin tags/fissures in perianal region.

Pyoderma Gangrenosum

Creative Commons Attribution License 4.0

Gastroenteroloy: Inflammatory Bowel Disease

161
uploaded by medbooksvn
NBME Top Concepts ➤ Gastroenterology Inflammatory Bowel Disease

Granulomas for the USMLE

Creative Commons Attribution License 4.0

Inflammatory Bowel Disease


The patient presents with blood-tinged diarrhea. He states that he has been having abdominal pain
and mucousy diarrhea. He has a flexible sigmoidoscopy is done which shows friable mucosal
pseudopolyps. What will be the histological findings behind this diagnosis?
• Ulcerative Colitis mucosal inflammation & crypt abscesses.

Ulcerative Colitis
This patient now presents with profuse bloody diarrhea, severe abdominal pain/bloating, and shock.
What is the likely diagnosis?
• Toxic Megacolon complication of UC
• What infectious etiology is most related to toxic megacolon?
•T. cruzi.

Gastroenteroloy: Inflammatory Bowel Disease

162
NBME Top Concepts ➤ Gastroenterology Inflammatory Bowel Disease

Crohn's vs. UC

Gastroenteroloy: Inflammatory Bowel Disease

163
uploaded by medbooksvn
NBME Top Concepts ➤ Gastroenterology Bile Acid Metabolism

NBME Style Question


A 40-year-old female presents with episodic right shoulder pain. She states her pain is worse after
meals. No trauma is noted in history. Her BMI is 35. Exam shows tenderness to palpation in the
RUQ. Which of the following best describes the pathophysiology of this patient’s symptoms?

A. Secretin stimulation of pancreatic secretion.


B. Musculoskeletal strain of the rotator cuff muscles.
C. CCK stimulation of gallbladder contraction.
D. Gastrin mediated relaxation of sphincter of Oddi.

Bile Acid Metabolism


Why does our body need this pathway?
• To break-down fat in the small intestine
Where does this process occur?
• Hepatocyte (liver); stored in GB; released in D
What is the rate limiting enzyme?
• 7-a-hydroxylase
When does this process occur?
• Increased parasympathetic activity
Process in a nutshell:
• Primary bile acids made conjugated to glycine and taurine bile salts micelles
intestinal brush border reabsorption enterohepatic circulation (terminal ileum).

Gastroenteroloy: Bile Acid Metabolism

164
NBME Top Concepts ➤ Gastroenterology Hepatitis B

Mastering Serology for Hepatitis B


Understand the basic principles:
▹Hepatitis BsAg and Hepatitis BeAg:
• You are infected and e-nfectious.
▹As soon as you have HepBsAg
• You make HepBcIgM
▹Once you have HepBsAb – you have resolved the infection or have been vaccinated.

In USMLE Questions watch for STI trigger + jaundice, etc. acute Hep B.

Gastroenteroloy: Hepatitis B

165
uploaded by medbooksvn
NBME Top Concepts ➤ Gastroenterology Vesicular Steatosis

NBME Style Question


A 37-year-old woman presents for evaluation of abnormal liver chemistries. She has long-standing
obesity (current BMI 38) and dyslipidemia. She takes no other medications and has a negative social
history. On examination, her liver span is 13 cm; she has no splenomegaly. Several sets of liver
enzymes have shown transaminases two to three times normal. Bilirubin and alkaline phosphatase
are normal. Hepatitis B surface antigen and hepatitis C antibody are normal, as are serum iron and
total iron-binding capacity. Which of the following is the likely pathology on liver biopsy?

a.Macrovesicular fatty liver


b.Microvesicular fatty liver
c.Portal triad hemachromatosis with necrosis
d.Cirrhosis
e.Copper deposition

NAFLD NASH
What is the pathophysiology?
• Insulin Resistance
• Low fatty acid oxidation – more fatty acid uptake.
• Non-alcoholic fatty liver-disease
• More free radical injury
• Causes cell death inflammation
• Non-alcoholic steatohepatitis
• Stellate Cells secrete TGF-β Fibrosis

USMLE Questions:
• Watch for metabolic syndrome:
• High-abdominal circumference
• Dyslipidemmia
• High blood pressure
• Insulin resistance

Gastroenteroloy: Vesicular Steatosis

166
NBME Top Concepts ➤ Gastroenterology Answer Key

Esophageal Issues
A 56-year-old man presents with weight loss, cough, and diffuse chest pain. He has been having
difficulty swallowing for one month. He is unable to drink or tolerate solid foods. Manometry shows
increased lower esophageal sphincter (LES) pressure. The high LES pressure is consistent even
after swallowing food. A radiological study in this patient would most likely be consistent with which
of the following findings?

a. Double bubble sign on abdominal x-ray.


b. Bird’s beak appearance on barium swallow.
c. Obstructive tumor on barium swallow.
d. Microcolon on abdominal x-ray.
e. Upper esophageal stricture in the esophagus on barium swallow.

Acid Secretion in Stomach


A 38-year-old male patient with a duodenal ulcer is treated successfully with cimetidine. Which of
the following mechanisms best describes the agent started in this patient?

a. Blocks muscarinic receptors on parietal cells


b. Blocks Gq receptors on parietal cells
c. Decreases intracellular cAMP levels
d. Activation of primary-active transport
e. Enhances the action of acetylcholine (ACh) on parietal cells

The following pairs of hormones are related to parietal cell secretion. Which of the following pairs
most correlate to the gastrointestinal physiology of parietal cells?

a. HCl and intrinsic factor


b. HCl and pepsinogen
c. HCl and HCO3−
d. HCO3− and intrinsic factor
e. Mucus and pepsinogen

Approach to Abdominal Pain on the USMLE


An 18 yr old male complains of nausea and lower abdominal pain. He has no vomiting or diarrhea.
He has been afebrile and denies sexual activity. Urine-analysis is negative for leukocyte esterase and
nitrites. Ultrasound with doppler shows reduced flow to the testes. Which of the following most
likely describes the pathophysiology in this patient?

A. Inflammation of the epididymis due to gram negative diplococci.


B. Colonization of the urethra due to gram negative rods.
C. Spermatic cord twisting with vascular pedicle compromise.
D. Neoplastic change to the testes with increased ɑ-fetoprotein.

Gastroenterology: Answer Key

167
uploaded by medbooksvn
NBME Top Concepts ➤ Gastroenterology Answer Key

Approach to Abdominal Pain on the USMLE


A 57-year-old male presents with a primary complaint of erectile dysfunction. After proper
evaluation, the patient is started on daily administration of sildenafil. This medication directly
causes accumulation of which of the following intracellular mediators?

A. Ca2+
B. cGMP
C. AMP
D. Nitrate
E. ANP

Bile Acid Metabolism


A 40-year-old female presents with episodic right shoulder pain. She states her pain is worse after
meals. No trauma is noted in history. Her BMI is 35. Exam shows tenderness to palpation in the
RUQ. Which of the following best describes the pathophysiology of this patient’s symptoms?

A. Secretin stimulation of pancreatic secretion.


B. Musculoskeletal strain of the rotator cuff muscles.
C. CCK stimulation of gallbladder contraction.
D. Gastrin mediated relaxation of sphincter of Oddi.

Vesicular Steatosis
A 37-year-old woman presents for evaluation of abnormal liver chemistries. She has long-standing
obesity (current BMI 38) and dyslipidemia. She takes no other medications and has a negative social
history. On examination, her liver span is 13 cm; she has no splenomegaly. Several sets of liver
enzymes have shown transaminases two to three times normal. Bilirubin and alkaline phosphatase
are normal. Hepatitis B surface antigen and hepatitis C antibody are normal, as are serum iron and
total iron-binding capacity. Which of the following is the likely pathology on liver biopsy?

a.Macrovesicular fatty liver


b.Microvesicular fatty liver
c.Portal triad hemachromatosis with necrosis
d.Cirrhosis
e.Copper deposition

Gastroenterology: Answer Key

168
CHAPTER 9

High Yield USMLE


Images & PE Findings:
Gastroenterology

169
uploaded by medbooksvn
High Yield Images & PE Findings ➤ Gastroenterology Oral Pathology

NBME Style Question


A 9-year-old male presents with fever, sore throat and rah. He is noted to have superficial ulcers
on the inside of his cheek. The patient has been exposed to other sick children in his after-school
day-care. The likely etiology is associated with which of the following conditions?

A. Keratitis
B. Myocarditis
C. Facial Nerve
D. Temporal Lobe Epilepsy
E. Super-imposed Bacterial Meningitis

Oral Pathology for the USMLE Step 1

Swartz MH: Textbook of Physical Diagnosis: History and Examination, 7th ed, Saunders Elsevier, 2014, p 296, Fig. 9-27.

NBME Style Question


A middle aged male presents with acute shortness of breath and drooling. He has a prodrome of
fevers and runny nose symptoms. He is audibly drooling and holding his mouth wide open. He has
severe respiratory distress. Patient decompensates in the trauma bay. An emergency airway
procedure known as a cricothyrotomy is performed. What is the likely anatomic landmarks for
successful airway securement?

A. Two cm inferior to the mandible


B. Above the cricoid cartilage and below the thyroid cartilage
C. Below the thyroid cartilage and above the thyroid cartilage
D. Buccopharyngeal fascia

Gastroenterology: Oral Pathology

170
High Yield Images & PE Findings ➤ Gastroenterology Oral Pathology

Retropharyngeal Abscess

Creative Commons Attribution License 4.0

NBME Style Question


A 50-year-old male presents with jaw pain. He is noted to have L submandibular swelling. There
has been no trauma. He has a history of dental caries and went 2 weeks ago for a dental extraction
on the L side. The patient’s exam reveals puss coming from the L submandibular region. The puss
contains sand/grain like particles. Upon analyzing the fluid, which of the following morphologies is
most likely to be seen?

A. Filamentous branching rod


B. Partially acid fast rod
C. Bi polar staining gram negative bacilli
D. Oxidase positive, gram negative bacili

Aphthous Ulcer

A patient presents with oral pain. They


are found to have a superficial painful
ulcer in the cheek described as a gray
ulcer with a red border. What is the likely
diagnosis?
• Aphthous ulcer

Creative Commons Attribution License 4.0

Gastroenterology: Oral Pathology

171
uploaded by medbooksvn
High Yield Images & PE Findings ➤ Gastroenterology Oral Pathology

Aphthous Ulcers
A patient presents with recurrent oral pain. They have recurrent scrotal pain and ulcerations in the
genital area despite abx therapy. Every few months, they have bilateral painful red eye. On exam, the
patient has bilateral knee and wrist pain. What is the likely diagnosis?
•Behçet’s disease systemic small vessel vasculitis w/ immune complex deposition
• Watch for aphthous ulcers, uveitis, and recurrent genital ulcers.

Oral Pathology

A patient with history of iron deficiency anemia


presents with oral pain. The patient is noted to also
have trouble swallowing. Upper endoscopy reveals a
web-like tissue in the esophagus. What is the likely
diagnosis?
• Plummer Vinson Syndrome.

Creative Commons Attribution License 4.0

Clinical Triad: Plummer Vinson syndrome

Creative Commons Attribution License 4.0

Gastroenterology: Oral Pathology

172
High Yield Images & PE Findings ➤ Gastroenterology Oral Pathology

Glossitis

Creative Commons Attribution License 4.0

Oral Pathology
A college student presents with neck pain and
fullness. Three weeks prior he had a self-resolving
episode of testicular pain. Serum studies show an
elevated amylase. A viral etiology is suspected. What is
the likely infectious etiology suspected in this patient?
• Paramyxovirus mumps
• What is the morphology?
• Negative sense ss negative RNA virus
• Characteristic of the vaccine?
• Live attenuated.
Kliegman, R: Nelson Textbook of Pediatrics, 19th ed, Philadelphia, Elsevier Saunders 2011, p 1080, Fig. 240.3l; from the Centers for Disease Control and Prevention: Public health image library [PHIL] [website]. http://phil.cdc.gov/phil/home. asp.

Gastroenterology: Oral Pathology

173
uploaded by medbooksvn
High Yield Images & PE Findings ➤ Gastroenterology Esophageal Pathology

Infectious Esophagitis for the USMLE

Swartz MH: Textbook of Physical Diagnosis: History and Examination, 7th ed, Saunders Elsevier, 2014, p 296, Fig. 9-27.

Esophageal Pathologies Framework

Ivan Damjanov, MD, PhD: Pathophysiology, Saunders Elsevier, 2009, p 254, Fig. 7-14.

Gastroenterology: Esophageal Pathology

174
High Yield Images & PE Findings ➤ Gastroenterology Esophageal Pathology

Esophageal Pathologies

A 5-year-old male presents with difficulty in swallowing. The patient has history of asthma and
eczema. The patient undergoes upper endoscopy which reveals furrowing in the proximal
esophagus. Biopsy shows >15 bilobed cells. Which cytokine is most likely related to this patient’s
current condition?
• IL-5 eosinophilic esophagitis
• Th2 cell-mediated disorder leading to eosinophilic infiltration of the esophageal mucosa.

Tracheoesophageal Fistula

Ivan Damjanov, MD, PhD: Pathology for the Health Professions, 4th ed, Saunders Elsevier, 2012, p 236, Fig. 10-5.

NBME Style Question


A patient presents with chest pain. He is noted to have had chest pain for the past 5 weeks. The
chest pain has no change with palpation. The chest pain does not have any relation to activity. The
pain is primarily worst at night. He has no dysphagia. He has a history of obesity and diet-controlled
diabetes. Which of the following is the most likely diagnosis?

A. Acute coronary syndrome


B. Costochondritis
C. GERD
D. Diffuse esophageal strictures

Gastroenterology: Esophageal Pathology

175
uploaded by medbooksvn
High Yield Images & PE Findings ➤ Gastroenterology Esophageal Pathology

Metaplasia is Reversible

Long Standing GERD can lead to Ulceration & Stricture

NBME Style Question


A middle age female presents with edema. Over the past three years she has noticed some
difficulty swallowing, and in tolerance to the cold. She describes her fingers turning blue, white, and
then red. She also is noticing palpable nodules on her hands. Which of the following antibodies is
likely positive in this patient?

A. Anti-centromere
B. Anti-dsDNA
C. Anti-Sm
D. Anti-SSA & SSB

Gastroenterology: Esophageal Pathology

176
High Yield Images & PE Findings ➤ Gastroenterology Esophageal Pathology

Gastric Hernias for the USMLE

Pulmonary Hypoplasia

A full-term female newborn develops respiratory distress


shortly after delivery. Physical examination shows cyanosis,
markedly decreased breath sounds bilaterally, and an
occasional bowel sound over the left hemithorax. A chest x-
ray taken while the patient is supine shows multiple cystic-
appearing areas on the left and decreased aeration on the
right. What is the likely mechanism?
• Failure of the pleuroperitoneal membrane to form
Congenital Diaphragmatic Hernia.

Gastroenterology: Esophageal Pathology

177
uploaded by medbooksvn
High Yield Images & PE Findings ➤ Gastroenterology Stomach Pathology

Pyloric Stenosis

A mother brings a 4 week old child into the clinic. He has been
feeding normally, however recently He has developed projectile, non-
billous vomiting. What is the physical exam finding associated with
this likely pathology?

• Palpable olive-like mass, and peristalsis of abdomen Pyloric


Stenosis
• Mechanism?
• Hypertrophy of the pyloric muscle congenitally normal
increases thickness over time.

Pyloric Stenosis

Gastroenterology: Stomach Pathology

178
High Yield Images & PE Findings ➤ Gastroenterology Stomach Pathology

NBME Style Question


A 48-year-old male presents with fever and altered mental status. He was noted to have a recent
admission to the hospital for SARS-CoV-2 infection. The patient recovered well and was discharged
home. He now presents with fever, hypotension, and warm extremities bilaterally. He is found to
have progressive hypotension, and elevations in lactate. He is transferred to the ICU. Which of the
following complications is most likely in this patient given his presentation?

A. Transudative fluid in lungs


B. Increased SVR
C. Increased Aerobic Metabolism
D. Decreased clearance of acid at the level of the stomach

Spectrum of Gastritis

Acute Gastritis for the USMLE

Gastroenterology: Stomach Pathology

179
uploaded by medbooksvn
High Yield Images & PE Findings ➤ Gastroenterology Stomach Pathology

Peptic Ulcer Disease

Microbiology Integration: H Pylori

Gastroenterology: Stomach Pathology

180
High Yield Images & PE Findings ➤ Gastroenterology Stomach Pathology

Perforated Duodenal Ulcer

Patient with history of PUD in the duodenum presents with acute


abdominal pain. Abdominal exam shows rigidity, and guarding, with
rebound tenderness. What is the likely diagnosis?
• Perforated duodenal ulcer
• Watch for free air under the diaphragm.

Creative Commons Attribution License 4.0

Complications of PUD

Causes of Upper GI Bleeding on the USMLE

Creative Commons Attribution License 4.0

Gastroenterology: Stomach Pathology

181
uploaded by medbooksvn
High Yield Images & PE Findings ➤ Gastroenterology Intestinal Pathology

Causes of lower GI Bleeding on the USMLE

Mechanisms of Diarrhea on the USMLE

NBME Style Question


A patient presents with chronic diarrhea. He is found to have blood and mucus in his diarrhea. The
patient recently returned from the Middle East. His H Pylori and C diff test are negative. The patient
undergoes colonscopy. Two flask-shaped ulcers are found. Which of the following conditions is this
patient most at risk for?

A. Cystic brain lesions


B. Liver Abscess
C. Bile duct obstruction
D. Granulomatous cystitis

Gastroenterology: Intestinal Pathology

182
High Yield Images & PE Findings ➤ Gastroenterology Answer Key

Oral Pathology
A 9-year-old male presents with fever, sore throat and rah. He is noted to have superficial ulcers
on the inside of his cheek. The patient has been exposed to other sick children in his after-school
day-care. The likely etiology is associated with which of the following conditions?

A. Keratitis
B. Myocarditis
C. Facial Nerve
D. Temporal Lobe Epilepsy
E. Super-imposed Bacterial Meningitis
A middle aged male presents with acute shortness of breath and drooling. He has a prodrome of
fevers and runny nose symptoms. He is audibly drooling and holding his mouth wide open. He has
severe respiratory distress. Patient decompensates in the trauma bay. An emergency airway
procedure known as a cricothyrotomy is performed. What is the likely anatomic landmarks for
successful airway securement?

A. Two cm inferior to the mandible


B. Above the cricoid cartilage and below the thyroid cartilage
C. Below the thyroid cartilage and above the thyroid cartilage
D. Buccopharyngeal fascia
A 50-year-old male presents with jaw pain. He is noted to have L submandibular swelling. There
has been no trauma. He has a history of dental caries and went 2 weeks ago for a dental extraction
on the L side. The patient’s exam reveals puss coming from the L submandibular region. The puss
contains sand/grain like particles. Upon analyzing the fluid, which of the following morphologies is
most likely to be seen?

A. Filamentous branching rod


B. Partially acid fast rod
C. Bi polar staining gram negative bacilli
D. Oxidase positive, gram negative bacili

Esophageal Pathology
A patient presents with chest pain. He is noted to have had chest pain for the past 5 weeks. The
chest pain has no change with palpation. The chest pain does not have any relation to activity. The
pain is primarily worst at night. He has no dysphagia. He has a history of obesity and diet-controlled
diabetes. Which of the following is the most likely diagnosis?

A. Acute coronary syndrome


B. Costochondritis
C. GERD
D. Diffuse esophageal strictures

Gastroenterology: Answer Key

183
uploaded by medbooksvn
High Yield Images & PE Findings ➤ Gastroenterology Answer Key

Esophageal Pathology
A middle age female presents with edema. Over the past three years she has noticed some
difficulty swallowing, and in tolerance to the cold. She describes her fingers turning blue, white, and
then red. She also is noticing palpable nodules on her hands. Which of the following antibodies is
likely positive in this patient?

A. Anti-centromere
B. Anti-dsDNA
C. Anti-Sm
D. Anti-SSA & SSB

Stomach Pathology
A 48-year-old male presents with fever and altered mental status. He was noted to have a recent
admission to the hospital for SARS-CoV-2 infection. The patient recovered well and was discharged
home. He now presents with fever, hypotension, and warm extremities bilaterally. He is found to
have progressive hypotension, and elevations in lactate. He is transferred to the ICU. Which of the
following complications is most likely in this patient given his presentation?

A. Transudative fluid in lungs


B. Increased SVR
C. Increased Aerobic Metabolism
D. Decreased clearance of acid at the level of the stomach

Intestinal Pathology
A patient presents with chronic diarrhea. He is found to have blood and mucus in his diarrhea. The
patient recently returned from the Middle East. His H Pylori and C diff test are negative. The patient
undergoes colonscopy. Two flask-shaped ulcers are found. Which of the following conditions is this
patient most at risk for?

A. Cystic brain lesions


B. Liver Abscess
C. Bile duct obstruction
D. Granulomatous cystitis

Gastroenterology: Answer Key

184
CHAPTER 10

NBME Top Concepts:


Hematology

185
uploaded by medbooksvn
NBME Top Concepts ➤ Hematology Heme Synthesis

Hemoglobin

Acute Intermittent Porphyria

Hematology: Heme Synthesis

186
NBME Top Concepts ➤ Hematology CYP Inducers

Guinness, Coronas, & PBRs Induce Chronic Alcoholism

Hematology: CYP Inducers

187
uploaded by medbooksvn
NBME Top Concepts ➤ Hematology Lead Poisoning

NBME Style Question


A 4-year-old male presents with cramping and abdominal pain. He has had generalized weakness
and decreased feeding. His father works as an industrial laborer. Family history is un-remarkable. He
has no other siblings. His labs are notable for an anemia with low-MCV and an abnormal blood
smear. Which of the following mechanisms may underlie this patient’s condition?

A. Excess blast proliferation


B. Auto-antibodies to intrinsic factor
C. Inhibition of ferrochelatase
D. MPO+ rods in immune cells leading to DIC

Glycine + Succinyl coA

Hematology: Lead Poisoning

188
NBME Top Concepts ➤ Hematology Approach to the Blood Smear

USMLE Test Taking Strategy

“Image should be correlated to the question, and should


confirm the concept being tested in the vignette”

Peripheral Blood Smears Creative Commons Attribution License 4.0

▸ Paraphrase the vignette to isolate the pathology


▸ Use the peripheral blood smear to confirm your
paraphrase
▸ Look at cells which are asymmetric from the
normal background.

NORMAL PERIPHERAL BLOOD SMEAR

NBME Style Question


A 30-year-old female presents for a follow up. She is status
post a splenectomy 1 year ago after a motor-vehicle accident.
She currently feels well and physical exam shows no
abnormalities. Lab studies are notable for a mild anemia. Her
peripheral blood smear is shown. An increase in which of the
following best explains the presence of these abnormal cells?

A. Aggregated iron deposits.


B. Clusters of remnant DNA. Creative Commons Attribution License 4.0

C. Denatured hemoglobin.
D. High erythrocyte cellular membrane to internal cell volume.

Hematology: Approach to the Blood Smear

189
uploaded by medbooksvn
NBME Top Concepts ➤ Hematology Approach to the Blood Smear

Target Cells
Target Cells: Creative Commons Attribution License 4.0

• Hemoglobin C disease:
• “A patient with hemolytic anemia,
splenomegaly, and Glu Lysine”
• Asplenia
• “Sickle cell or MVC patient”
• Liver Disease
• ”Patient with alcoholism”
• “Obstructive liver disease”
•Thalassemia
• “Chipmunk facies, hair-on-end appearance,
chronic transfusions,”

Pathophysiology Breakdown

Hematology: Approach to the Blood Smear

190
NBME Top Concepts ➤ Hematology Approach to the Blood Smear

USMLE Vignette

Creative Commons Attribution License 4.0

Hematology: Approach to the Blood Smear

191
uploaded by medbooksvn
NBME Top Concepts ➤ Hematology Intro to Anemia

Approach to Anemia

Creative Commons Attribution License 4.0

Hematology: Intro to Anemia

192
NBME Top Concepts ➤ Hematology Microcytic Anemia

NBME Style Question


An 18-year-old female presents with fatigue. She has had some weight gain. She is an avid tennis
player. Menarche started at 12. She is Tanner stage 3 on physical exam. Pale conjunctivae are also
noted on exam. Her TSH is slightly elevated with a low fT4. Which of the following additional
laboratory findings are most likely to be noted in this patient?

A. Normal MCV.
B. Elevated folate.
C. Decreased ferritin.
D. Hyper-segmented neutrophils on peripheral blood smear.
E. Decreased TIBC.

Iron Deficiency Anemia


Characteristically, patients will have low Hgb, low MCV, and high RDW.
▹ Represents a high-distribution of RBCs.
Think Like the Test Maker:
▹ “Female in the premenopausal period”
▹ “Elderly with anemia, fatigue, weight loss”
▹ “Pregnancy causing a dilutional anemia”
▹ “Third world country child”

Pathophysiology of Iron Deficiency Anemia

Hematology: Microcytic Anemia

193
uploaded by medbooksvn
NBME Top Concepts ➤ Hematology Microcytic Anemia

Anemia of Chronic Disease


A patient with RA is diagnosed with anemia. What may be the likely classification of this anemia?
▹Anemia of Chronic Disease
Pathophysiology:
▹Increased trapping of iron in bone -> IL-1
▹Hepcidin (acute phase reactant) increases which decreases iron absorption
Ferritin is high, TIBC is low

USMLE Point: look for PMH noting high inflammation states (SLE, RA, chronic infections).

Hematology: Microcytic Anemia

194
NBME Top Concepts ➤ Hematology Acute Phase Reactants

Positive & Negative Acute Phase Reactants


With inflammation what are markers that are abnormal?
• Positive acute phase reactants:
• CRP: non-specific sign, fixes complement and increases phagocytosis
• Ferritin: inhibits microbial iron utilization
• Fibrinogen: promotes endothelial repair
• Hepcidin: decreases iron absorption
• Serum amyloid A: can lead to amyloidosis

• Negative acute phase reactants:


• Albumin: conservation of AA
• Transferrin

Hematology: Acute Phase Reactants

195
uploaded by medbooksvn
NBME Top Concepts ➤ Hematology Summary of Microcytic

Summary of Microcytic Anemia for the USMLE

Hematology: Summary of Microcytic

196
NBME Top Concepts ➤ Hematology Macrocytic Anemia

Approach to Macrocytic Anemias

Characteristically, patients will have low Hgb, high MCV.


• Subset of macrocytic anemias megaloblastic anemia:
• B12
• Folate
Think Like the Test Maker:
• Contrast them by physical exam:
• B12 neurological deficits
• Folate no neurological deficits.

Hypersegmented Neutrophils

Creative Commons Attribution License 4.0

NBME Style Question


A 59-year-old female presents with progressive fatigue. She is noted to have a history of
hypothyroidism. Vital signs are 37.2C, 100/min, RR 15/min, BP 100/80. Laboratory studies are
notable for a low Hgb and an MCV of 114. Serum sodium is noted to be decreased. Which of the
following pathophysiologic mechanisms most likely is noted in this patient?

A. Autoimmune destruction of parietal cells.


B. Upregulation of renin-angiotensin-aldosterone system.
C. Chronic inflammation leading to elevated ferritin and anemia of chronic disease.
D. Decreased dietary folate.

Hematology: Macrocytic Anemia

197
uploaded by medbooksvn
NBME Top Concepts ➤ Hematology Macrocytic Anemia

Test TakingTtip for the USMLE

B12 Deficiency
Think Like the Test Maker:
• Who are patients on the USMLE that you may suspect B-12 deficiency, especially if they
present with anemia (MCV>100)?
• Vegan
• Note: Takes years of being a vegan as B12 is stored in the liver
• Patient with pernicious anemia
• Patient with gastrectomy
• Malabsorption:
• Crohn’s disease patient who has lesion of terminal ileum
• Fish tapeworm:
• Diphyllobothrium latum.

Hematology: Macrocytic Anemia

198
NBME Top Concepts ➤ Hematology B12 Physiology

Increase MMA Causes Neurological Deficits

Odd chain fatty acid metabolism does not plug into TCA

Hematology: B12 Physiology

199
uploaded by medbooksvn
NBME Top Concepts ➤ Hematology Normocytic Anemia

Test Taking Strategy: Recognition of Hemolysis on the USMLE

NORMOCYTIC
ANEMIAS

HEMOLYTIC
ANEMIAS

Where is the hemolysis occurring?

Hematology: Normocytic Anemia

200
NBME Top Concepts ➤ Hematology Normocytic Anemia

Intrinsic Hemolytic Anemias

NBME Style Question


A 20-year-old male presents with cough and fever.
The cough is non-productive. He has had no sick
contacts and attends college. He is noted to be fatigued.
Physical exam is notable for jaundice. Laboratory
studies are notable for anemia, a normal MCV, and low
haptoglobin. CXR is shown. Which of the following
additional laboratory tests will be present in this patient?

A. Low indirect bilirubin.


B. Increased LFTs with low FEV1.
C. Increased ASO titers. Creative Commons Attribution License 4.0

D. Coombs positivity.

Hematology: Normocytic Anemia

201
uploaded by medbooksvn
NBME Top Concepts ➤ Hematology Normocytic Anemia

Mycoplasma Pneumoniae on the USMLE

Creative Commons Attribution License 4.0

Hematology: Normocytic Anemia

202
NBME Top Concepts ➤ Hematology HUS/TTP

Pathophysiology of HUS & TTP

Test Taking Strategy: Recognition of TRIADS on the USMLE

What do you need to add to make it TTP?


▹ Fever (look for it in vital signs)
▹ Altered mental status (look for it in physical exam)

Hematology: HUS/TTP

203
uploaded by medbooksvn
NBME Top Concepts ➤ Hematology Polycythemia

Polycythemia

Definition:
• Increased erythrocytes
A patient with PMH notable for renal cell carcinoma presents with increased daytime sleepiness.
BMI > 30. He is noted to have an elevated Hct. What is the likely mechanism?
• Hypoxemia increasing EPO increased erythrocyte production.
If left un-treated, would his pulmonary vascular resistance increase or decrease?
• Increase RV afterload?
• Increased.

How to Approach Polycythemia on the USMLE

Creative Commons Attribution License 4.0

Hematology: Polycythemia

204
NBME Top Concepts ➤ Hematology Platelet Pathology

Organizing NBME Concepts for Hematology

NBME Style Question


A 12-year-old male presents with prolonged gum bleeding. He recently was at the dentist for a
tooth extraction. His father and paternal aunt have similar issues following minor surgical
procedures. He is started on desmopressin (DDAVP). Which of the following most likely highlights
the therapeutic mechanism of desmopressin in this patient?

A. Increases vWF release by vascular endothelial cells.


B. Increases release of factor 10 from vascular endothelial cells.
C. Decreases factor 8 release from vascular endothelial cells.
D. Promotes clotting via release of fibrin.

vWF Disease for the USMLE

Hematology: Polycythemia

205
uploaded by medbooksvn
NBME Top Concepts ➤ Hematology Platelet Pathology

Differentiating between Primary and Secondary Hemostatic Disorders

Hematology: Polycythemia

206
NBME Top Concepts ➤ Hematology Warfarin vs. Heparin

USMLE Pharmacology

Organizing NBME Concepts for Hematology

Hematology: Warfarin vs. Heparin

207
uploaded by medbooksvn
NBME Top Concepts ➤ Hematology Multiple Myeloma

USMLE Test Taking Strategy: Back Pain

NBME Style Question


A 72-year-old male presents with recurrent bacterial PNA. He is also noted to have abnormal urine
analysis notable for nitrites. His childhood PMH is normal. On exam, the patient endorses back pain.
He is found to have an abnormal peripheral blood smear. Which of the following findings are most
likely to be seen in this patient?

A. Hypocalcemia
B. Low serum free light-chain
C. Qualitative immunoglobulin defect
D. Normal creatinine

Multiple Myeloma
• What is the mechanism behind hypercalcemia? • What is the mechanism behind recurrent
• Increased osteoclastic activity infections?
• Breaks down bone • MM abnormal proliferation of plasma
• Release of calcium cells
• Monoclonal expansion of light chains
non-functional antibodies bad immune
system

Hematology: Multiple Myeloma

208
NBME Top Concepts ➤ Hematology Summary & Courses

Summary of Top NBME Hematology Concepts

Hematology: Summary & Courses

209
uploaded by medbooksvn
NBME Top Concepts ➤ Hematology Answer Key

Lead Poisoning
A 4-year-old male presents with cramping and abdominal pain. He has had generalized weakness
and decreased feeding. His father works as an industrial laborer. Family history is un-remarkable. He
has no other siblings. His labs are notable for an anemia with low-MCV and an abnormal blood
smear. Which of the following mechanisms may underlie this patient’s condition?

A. Excess blast proliferation


B. Auto-antibodies to intrinsic factor
C. Inhibition of ferrochelatase
D. MPO+ rods in immune cells leading to DIC

Approach to the Blood Smear


A 30-year-old female presents for a follow up. She is status post a splenectomy 1 year ago after a
motor-vehicle accident. She currently feels well and physical exam shows no abnormalities. Lab
studies are notable for a mild anemia. Her peripheral blood smear is shown. An increase in which of
the following best explains the presence of these abnormal cells?

A. Aggregated iron deposits.


B. Clusters of remnant DNA.
C. Denatured hemoglobin.
D. High erythrocyte cellular membrane to internal cell volume.

Microcytic Anemia
An 18-year-old female presents with fatigue. She has had some weight gain. She is an avid tennis
player. Menarche started at 12. She is Tanner stage 3 on physical exam. Pale conjunctivae are also
noted on exam. Her TSH is slightly elevated with a low fT4. Which of the following additional
laboratory findings are most likely to be noted in this patient?

A. Normal MCV.
B. Elevated folate.
C. Decreased ferritin.
D. Hyper-segmented neutrophils on peripheral blood smear.
E. Decreased TIBC.

Hematology: Answer Key

210
NBME Top Concepts ➤ Hematology Answer Key

Macrocytic Anemia
A 59-year-old female presents with progressive fatigue. She is noted to have a history of
hypothyroidism. Vital signs are 37.2C, 100/min, RR 15/min, BP 100/80. Laboratory studies are
notable for a low Hgb and an MCV of 114. Serum sodium is noted to be decreased. Which of the
following pathophysiologic mechanisms most likely is noted in this patient?

A. Autoimmune destruction of parietal cells.


B. Upregulation of renin-angiotensin-aldosterone system.
C. Chronic inflammation leading to elevated ferritin and anemia of chronic disease.
D. Decreased dietary folate.

Normocytic Anemia
A 20-year-old male presents with cough and fever. The cough is non-productive. He has had no
sick contacts and attends college. He is noted to be fatigued. Physical exam is notable for jaundice.
Laboratory studies are notable for anemia, a normal MCV, and low haptoglobin. CXR is shown. Which
of the following additional laboratory tests will be present in this patient?

A. Low indirect bilirubin.


B. Increased LFTs with low FEV1.
C. Increased ASO titers.
D. Coombs positivity.

Platelet Pathology
A 12-year-old male presents with prolonged gum bleeding. He recently was at the dentist for a
tooth extraction. His father and paternal aunt have similar issues following minor surgical
procedures. He is started on desmopressin (DDAVP). Which of the following most likely highlights
the therapeutic mechanism of desmopressin in this patient?

A. Increases vWF release by vascular endothelial cells.


B. Increases release of factor 10 from vascular endothelial cells.
C. Decreases factor 8 release from vascular endothelial cells.
D. Promotes clotting via release of fibrin.

Hematology: Answer Key

211
uploaded by medbooksvn
NBME Top Concepts ➤ Hematology Answer Key

Multiple Myeloma
A 72-year-old male presents with recurrent bacterial PNA. He is also noted to have abnormal urine
analysis notable for nitrites. His childhood PMH is normal. On exam, the patient endorses back pain.
He is found to have an abnormal peripheral blood smear. Which of the following findings are most
likely to be seen in this patient?

A. Hypocalcemia
B. Low serum free light-chain
C. Qualitative immunoglobulin defect
D. Normal creatinine

Hematology: Answer Key

212
CHAPTER 11

High Yield Images


for the USMLE
Step 1 - Hematology

213
uploaded by medbooksvn
High Yield Images & PE Findings ➤ Hematology Peripheral Blood Smears

NBME Style Question


A 4-year-old male presents with developmental delay. Over the past six weeks mother has noted
that the patient has had decreased bowel movements and has been feeling weak. His 2 yo sister has
had similar symptoms. What is the most likely cause of this patient’s symptoms?

A. Acute leukemia
B. Heavy metal poisoning
C. Nutritional defect
D. Immunodeficiency
E. Demyelination of CNS
F. CO poisoning

Understanding Lung PE for the USMLE


• Ferrochelatase
• Where is the enzyme located?
• Mitochondria
• Lead poisoning
• Inhibits ferrochelatase
• Inhibits Delta ALA Dehydratase
• What will blood smear show?
• Basophilic stippling
• Represents remnants of rRNA in RBCs
• USMLE Point:
• Lead poisoning presents with non-specific symptoms headaches,
constipation, developmental delay
• Patients in exam questions will usually be children with a microcytic
anemia.

Basophilic stippling on the usmle


When you see basophilic stippling:
1. Lead poisoning
• Pediatric patient + constipation + learning disability
• Inhibition of ferrochelatase
• What do the basophilic remnants represent?
• accumulated rRNA lead inhibits the denaturation of rRNA
• Lead denatures ribonuclease hence ribosomes
accumulate
2. Sideroblastic Anemia
• X linked deficiency in delta ALA synthase (B6)
Naeim F: Atlas of Bone Marrow and Blood Pathology, Philadelphia, Saunders, 2001, p 27, Fig. 2-22M

• Basophilic granules (Pappenheimer) contain iron in X-


linked disorder.

Hematology: Peripheral Blood Smears

214
High Yield Images & PE Findings ➤ Hematology Peripheral Blood Smears

Summary of Lead Poisoning

Creative Commons Attribution License 4.0

Hematology: Peripheral Blood Smears

215
uploaded by medbooksvn
High Yield Images & PE Findings ➤ Hematology RBC Inclusions

RBC Inclusions
A 23-year-old African American Male presents with jaundice and SOB after he had a common cold.
He had been using his mom’s TMP-SMX antibiotic. Hgb low. What is the likely diagnosis?
• G6PD Deficiency what would peripheral blood smear show?
• Heinz bodies. What do they represent?
• Oxidized Hemoglobin within RBCs
• Bite Cells. What do they represent?
• Result of phagocytic removal of Heinz bodies by splenic MΦ
• Test Taking Pearl: always isolate triggers or stressors with lead to pathology in USMLE questions.

Hemolysis due to G 6 PD deficiency

Creative Commons Attribution License 4.0

Pentose Phosphate Pathway

Hematology: RBC Inclusions

216
High Yield Images & PE Findings ➤ Hematology RBC Inclusions

Pentose Phosphate Pathway or HMP Shunt

Oxidative Portion of Pentose Phosphate Pathway

Creative Commons Attribution License 4.0

No G6PD = no reduced glutathione = oxidant stress wins hemolysis

Hematology: RBC Inclusions

217
uploaded by medbooksvn
High Yield Images & PE Findings ➤ Hematology RBC Inclusions

RBC Inclusions
A 5-year-old African American male presents with thrombocytopenia & extreme abdominal pain.
He has a history of several such episodes in the past two years. His most recent episode was for
extreme pain in his bilateral lower extremities. There is no trauma and patient has no history of
fractures. What is the likely mechanism underlying this diagnosis?
• Glutamic acid (normal) Valine (AR) at the sixth position in the Beta-change (missense
mutation)
• 60-90% HgbSS to be present for sickle cell disease
• If there is only a small percentage of Hgb SS on electrophoresis think of trait what is the genetic
advantage?
• Protection against plasmodium falciparum.

Hematology: RBC Inclusions

218
High Yield Images & PE Findings ➤ Hematology Sickle Cell Disease

Sickle Cell Disease

Sickle Cell Disease


A Patients with sickle cell are treated with hydroxyurea to decrease frequency and severity of vaso-
occlusive crisis. What is the direct effect on their hemoglobin physiology?
• Increases HbF less right shift of Hgb & O2 dissociation curve
• Hydroxyurea inhibits Ribonucleotide Reductase à what biochemistry pathway is affected?
• Pyrimidine Base Synthesis.

Hypoxemia leads to sickling of Red Blood Cells

Hematology: Sickle Cell Disease

219
uploaded by medbooksvn
High Yield Images & PE Findings ➤ Hematology Sickle Cell Disease

Sickle Cell Disease

USMLE Integrations

Hematology: Sickle Cell Disease

220
High Yield Images & PE Findings ➤ Hematology Sickle Cell Disease

Encapsulated Organisms

Sickle Cell & RBC Inclusions


• What is the mechanism behind splenic insufficiency in sickle cell disease?
• Sickling in spleen consumption & decreased blood flow splenic infarction
• Sickle cell patient post splenectomy may have which RBC inclusion on PBS?
• Howell Jolly Bodies what do these represent?
• Remnants of nuclear chromatin (do not contain iron).

Hematology: Sickle Cell Disease

221
uploaded by medbooksvn
High Yield Images & PE Findings ➤ Hematology Complications of Asplenia

Complications of Asplenia
• What types of vaccines are patients with asplenia or post splenectomy in need of:
• Polysaccharide
• Specific Vaccines:
• S. pneumonia:
• Post splenectomy PCV 13 PPSV23
• Meningococcus:
• Meningococcal ACWY conjugate vaccine
• Hib:
• Polysaccharide vaccine conjugated to toxoid
• What anti-microbial prophylaxis is important for patients without a spleen?
• Amoxicillin (up to at least 5 yo)

NBME Style Question


A 1-year-old male presents with severe swelling in his hands. He has a family history of brother
who died of pneumococcal sepsis. Which of the following serum lab values may be most abnormal
in this patient?

A. Cortisol
B. C5-C9
C. Tryptase
D. DAF (CD55-59)
E. Haptoglobin

Hematology: Complications of Asplenia

222
High Yield Images & PE Findings ➤ Hematology Summary of RBC Inclusions

Summary of RBC inclusions

NBME Style Question


A patient presents after cardiac surgery. Pre-operatively he was noted to have a holosystolic
murmur heard best at the 2nd right aortic area. Post operatively, Shischtocytes are seen on
peripheral blood smear. What is the likely diagnostic test which may be abnormal?

A. Decreased reticulocyte count


B. Skull X-ray showing hair-on-end
C. Increased serum iron due to anemia of chronic disease
D. Decreased haptoglobin

Hematology: Summary of RBC Inclusions

223
uploaded by medbooksvn
High Yield Images & PE Findings ➤ Hematology Mechanisms of Extravascular RBC Hemolysis

Mechanisms of RBC Hemolysis | Extravascular

Physical Exam Features of Hemolysis

Hematology: Mechanisms of Extravascular RBC Hemolysis

224
High Yield Images & PE Findings ➤ Hematology Mechanisms of Intravascular RBC Hemolysis

Mechanisms of RBC Hemolysis | Intravascular

NBME Style Question

A 5-year-old male presents with hepatosplenomegaly. He is diagnosed with beta-thalassemia. He


presents two years later and dies of overwhelming sepsis. On autopsy there are clumps of RBC
precursors which are found in the gallbladder and spleen. What is the likely mechanism?

A. Extravascular hemolysis
B. Intravascular hemolysis
C. EPO increase
D. Portal hypertension

Hematology: Mechanisms of Intravascular RBC Hemolysis

225
uploaded by medbooksvn
High Yield Images & PE Findings ➤ Hematology Normocytic Anemia

Normocytic Anemia

Normocytic Anemias with Reticulocytosis (>2%)

Hematology: Normocytic Anemia

226
High Yield Images & PE Findings ➤ Hematology Normocytic Anemia

Extravascular vs Intravascular Hemolysis

NBME Style Question


A 7-year-old male presents with PNA. He has been having frequent nose bleeds and fatigue. His
labs show pancytopenia. He undergoes bone-marrow biopsy which shows a profoundly hypocellular
marrow with increased lipid cells. What PE feature may be seen in this patient?

A. Jaundice
B. Splenomegaly
C. Gallbladder inflammation
D. Angiodysplasia of the nasal passages

Hematology: Normocytic Anemia

227
uploaded by medbooksvn
High Yield Images & PE Findings ➤ Hematology Aplastic Anemia

Aplastic Anemia

Key Vignettes

Creative Commons Attribution License 4.0

Hematology: Aplastic Anemia

228
High Yield Images & PE Findings ➤ Hematology Aplastic Anemia

Key Vignettes

Hematology: Aplastic Anemia

229
uploaded by medbooksvn
High Yield Images & PE Findings ➤ Hematology Cold and Warm AIHA

Auto-Immune Hemolytic Anemia

Creative Commons Attribution License 4.0

Hematology: Cold and Warm AIHA

230
High Yield Images & PE Findings ➤ Hematology HUS/TTP

Hemolytic Uremic Syndrome TTP

Hematology: HUS/TTP

231
uploaded by medbooksvn
High Yield Images & PE Findings ➤ Hematology Cells of the Immune System

NBME Style Question

A 50-year-old male presents with asthma exacerbation. He is noted to have chronic sinus
infections. PE shows wrist drop. His IgE count is noted to be elevated and peripheral blood smear
shows increased proliferation of cells with dumbbell shaped nuclei. What is the likely lab
abnormality in this patient?

A. Ab to myeloperoxidase
B. Ab to anti-proteinease-3
C. Seronegative spondyloarthropathy
D. IgM mediated cold agglutinin

Hematology: Cells of the Immune System

232
High Yield Images & PE Findings ➤ Hematology Causes of Eosinophilia on the USMLE

Causes of Eosinophilia on the USMLE

Hematology: Causes of Eosinophilia on the USMLE

233
uploaded by medbooksvn
High Yield Images & PE Findings ➤ Hematology Causes of Plasma Cells on the USMLE

Causes of Plasma Cells on the USMLE

Hematology: Causes of Plasma Cells on the USMLE

234
High Yield Images & PE Findings ➤ Hematology Causes of Mast Cells on the USMLE

Causes of Mast Cells on the USMLE

Hematology: Causes of Mast Cells on the USMLE

235
uploaded by medbooksvn
High Yield Images & PE Findings ➤ Hematology Answer Key

Peripheral Blood Smears


A 4-year-old male presents with developmental delay. Over the past six weeks mother has noted
that the patient has had decreased bowel movements and has been feeling weak. His 2 yo sister has
had similar symptoms. What is the most likely cause of this patient’s symptoms?

A. Acute leukemia
B. Heavy metal poisoning
C. Nutritional defect
D. Immunodeficiency
E. Demyelination of CNS
F. CO poisoning

Complications of Asplenia
A 1-year-old male presents with severe swelling in his hands. He has a family history of brother
who died of pneumococcal sepsis. Which of the following serum lab values may be most abnormal
in this patient?

A. Cortisol
B. C5-C9
C. Tryptase
D. DAF (CD55-59)
E. Haptoglobin

Summary of RBC Inclusions

A patient presents after cardiac surgery. Pre-operatively he was noted to have a holosystolic
murmur heard best at the 2nd right aortic area. Post operatively, Shischtocytes are seen on
peripheral blood smear. What is the likely diagnostic test which may be abnormal?

A. Decreased reticulocyte count


B. Skull X-ray showing hair-on-end
C. Increased serum iron due to anemia of chronic disease
D. Decreased haptoglobin

Hematology: Answer Key

236
High Yield Images & PE Findings ➤ Hematology Answer Key

Mechanisms of Intravascular RBC Hemolysis


A 5-year-old male presents with hepatosplenomegaly. He is diagnosed with beta-thalassemia. He
presents two years later and dies of overwhelming sepsis. On autopsy there are clumps of RBC
precursors which are found in the gallbladder and spleen. What is the likely mechanism?

A. Extravascular hemolysis
B. Intravascular hemolysis
C. EPO increase
D. Portal hypertension

Normocytic Anemia
A 7-year-old male presents with PNA. He has been having frequent nose bleeds and fatigue. His
labs show pancytopenia. He undergoes bone-marrow biopsy which shows a profoundly hypocellular
marrow with increased lipid cells. What PE feature may be seen in this patient?

A. Jaundice
B. Splenomegaly
C. Gallbladder inflammation
D. Angiodysplasia of the nasal passages

Cells of the Immune System


A 50-year-old male presents with asthma exacerbation. He is noted to have chronic sinus
infections. PE shows wrist drop. His IgE count is noted to be elevated and peripheral blood smear
shows increased proliferation of cells with dumbbell shaped nuclei. What is the likely lab
abnormality in this patient?

A. Ab to myeloperoxidase
B. Ab to anti-proteinease-3
C. Seronegative spondyloarthropathy
D. IgM mediated cold agglutinin

Hematology: Answer Key

237
uploaded by medbooksvn
CHAPTER 12

NBME Top Concepts:


Neurology

238
NBME Top Concepts ➤ Neurology Brain Hematomas

NBME Style Question

A 23-year-old male presents to trauma bay after MVC. He is noted to be awake and alert on
primary survey. Two hours later, he is noted to have a loss of consciousness. His vitals are 140/80,
pulse 60/min, RR 10. His pupil is mildly dilated, and bruising is noted in his temple. What is the likely
diagnosis?

A. Middle Cerebral Artery Ischemic Stroke.


B. Epidural Hematoma.
C. Subdural Hematoma.
D. Subarachnoid hemorrhage.
E. Rupture of Bouchard aneurysm.

Think Like the Test Maker: Epidural Hematoma


What would be possible scenarios in the test question for epidural hematomas?
• “Patient who got hit on the side of the head”
• “Skull fracture”
• “Patient after trauma who was AOx3, and then passes away 5 hours later.
• “Talk and Die”
What is the affected artery?
• Middle meningeal
• Branch off maxillary artery (one of terminal branches of external carotid).

Test Taking Tip for the USMLE

While studying anatomical lesions, integrate high-yield


pathologies with anatomy.
• If there is an affected artery, nerve, lymphatic, vein
track it back to a structure you are familiar with! (i.e.
aorta)

Neurology: Brain Hematomas

239
uploaded by medbooksvn
NBME Top Concepts ➤ Neurology Brain Hematomas

Foramen Spinosum

Creative Commons Attribution License 4.0

Neurology: Brain Hematomas

240
NBME Top Concepts ➤ Neurology Brain Hematomas

Think Like the Test Maker: Subdural Hematoma


What would be possible scenarios in the test question?
• “Baby who has bilateral retinal hemorrhages”
• Child abuse
• “Nursing home resident who fell” also “alcoholic”
• Both have atrophy of brain, and this increases the risk of bleed in subdural hematoma
• Slow onset of symptoms
What is the affected artery?
•Tearing of the bridging veins.

Subarachnoid
Patient with U/S confirmed cystic kidneys, has a sudden onset of headache, vomiting, and
photophobia. Where is the most likely location of this pathology?
• Diagnosis: Rupture of berry aneurysm Sub-Arachnoid Hemorrhage
• Classic pattern on CT imaging:
• Pattern of blood follows sulci and gyri
• CSF tap will show yellow-ish CSF consistent with xanthochromia
CCB nimodipine is given to reduce risk of vasospasm

Circle of Willis

Creative Commons Attribution License 4.0

ACA and Anterior communicating artery junction


Mechanism: At this junction there is no strong media, so aneurysms prone to occur
and rupture

Neurology: Brain Hematomas

241
uploaded by medbooksvn
NBME Top Concepts ➤ Neurology Brain Hematomas

Brain Aneurysms for the USMLE

USMLE Vignette

Neurology: Brain Hematomas

242
NBME Top Concepts ➤ Neurology Herniation Syndromes & Cerebral Physiology

NBME Style Question


A 6-year-old male presents to the intensive care unit after diagnosis of subarachnoid hemorrhage.
One hour after admission, his vital signs are notable for BP 140/80, HR 60, RR 12. He has a right
pupil minimally reactive and 4 mm in diameter, his left pupil is noted to be 2 mm and reactive. Given
normal cerebral autoregulation, which of the following interventions may reduce elevated
intracranial pressure in this patient?

A. Hypoventilation
B. No change in respirations
C. Increased blood pressure by stimulating alpha-receptors
D. Hyperventilation

Test Taking Strategy: Recognition of TRIADS on the USMLE

What is the pathophysiology of reflex bradycardia?


▹ Activation of baroreceptor (฀stretch) efferent parasympathetic
activity.

Neurology: Herniation Syndromes & Cerebral Physiology

243
uploaded by medbooksvn
NBME Top Concepts ➤ Neurology Herniation Syndromes & Cerebral Physiology

Cerebral Autoregulation

Cerebral Autoregulation

Neurology: Herniation Syndromes & Cerebral Physiology

244
NBME Top Concepts ➤ Neurology Herniation Syndromes & Cerebral Physiology

Uncal Herniation
What is the mechanism behind the dilated pupil?
• Compression of CN 3 as it exits the midbrain

Pathophysiology: Compression of pre-ganglionic


parasympathetic fibers

If you damage parasympathetic, what pupillary changes will


be present?
• Dilation of pupils.
Creative Commons Attribution License 4.0

Anatomy of Cranial Nerve 3

Creative Commons Attribution License 4.0

Neurology: Herniation Syndromes & Cerebral Physiology

245
uploaded by medbooksvn
NBME Top Concepts ➤ Neurology Cranial Nerve Path (1)

NBME Style Question


A 16-year-old female presents with fever and throat pain. She is noted to have increased fatigue
for the past week. Vital signs are notable for fever. The physical exam is notable for exudates on
tonsils and left-sided uvular deviation. CT of the neck shows concern for injury to a cranial nerve.
Which of the following nerves related to this patient’s exam findings is most likely to be affected?

A. Vagus
B. Glossopharyngeal
C. Hypoglossal
D. Trochlear
E. Trigeminal

Uvular Deviation

Uvular deviation away from the side of the lesion.


• R CN X involvement will lead to L uvular deviation.

Creative Commons Attribution License 4.0

USMLE Vignette

Neurology: Cranial Nerve Path (1)

246
NBME Top Concepts ➤ Neurology Cranial Nerve Path (1)

Neuroanatomy Integration

Creative Commons Attribution License 4.0

USMLE Vignette

Neurology: Cranial Nerve Path (1)

247
uploaded by medbooksvn
NBME Top Concepts ➤ Neurology Cranial Nerve Path (1)

Neuroanatomy Integration

Creative Commons Attribution License 4.0

USMLE Vignette

Neurology: Cranial Nerve Path (1)

248
NBME Top Concepts ➤ Neurology Bell’s Palsy

Bell’s Palsy
An avid hiker who presents with an inability to raise her eyebrows. She has dry eyes and is
hypersensitive to sound. What cranial nerve may be affected?
▹ Cranial Nerve 7 Bell’s Palsy (LMN). What is the mechanism behind the hyperacusis?
• Stapedius weakness more oscillations on oval window and conduction.
▹ What may happen to this patient’s sense of taste?
• Decreased. Chorda tympani is a branch of 7.
• Taste Anterior 2/3 of tongue.
▹ What pharyngeal arch is related to cranial nerve 7?
• 2nd

Microbiology Integration
What are organisms related to Bell’s Palsy?
▹ HSV
• Double stranded, DNA linear virus
▹ Borrelia Burgdorferi
• Obligate intracellular spirochete

Borrelia Burgdorferi

Neurology: Bell’s Palsy

249
uploaded by medbooksvn
NBME Top Concepts ➤ Neurology Cranial Nerve Path (2)

USMLE Vignette

Neurology: Cranial Nerve Path (2)

250
NBME Top Concepts ➤ Neurology Multiple Sclerosis

Multiple Sclerosis
A 24-year-old woman with MS has an MRI that shows demyelination of the optic nerve on the left
side. On physical exam, what will happen to both pupils when light hits the right pupil?
• They will constrict optic neuritis
• USMLE Point:
• Young female with focal neurological deficit think multiple sclerosis.

Neuroanatomy Integration

Creative Commons Attribution License 4.0

Vignettes for Multiple Sclerosis

Neurology: Multiple Sclerosis

251
uploaded by medbooksvn
NBME Top Concepts ➤ Neurology Dementia

NBME Style Question


A 40-year-old woman presents with a skin rash. She is noted by her family members to have
increased episodes of disorientation. The exam is notable for a skin rash on the arms along within
the buttocks region. Further history notes increased loose stools and BMI 18. The patient has poor
performance on the mental status exam. Which of the following is the most likely pathophysiologic
mechanism behind the diagnosis?

A. Nutritional deficiency.
B. Age-related changes.
C. Degeneration of the frontal and temporal regions.
E. Neurocutaneous syndrome.

Test Taking Strategy: Recognition of Triads on the USMLE

What is the likely amino acid precursor?


▹ Tryptophan

Neurology: Dementia

252
NBME Top Concepts ➤ Neurology Dementia

Dementia for the USMLE

Neurology: Dementia

253
uploaded by medbooksvn
NBME Top Concepts ➤ Neurology Dopamine Pathways

USMLE Neuroanatomy Integration

Neurology: Dopamine Pathways

254
NBME Top Concepts ➤ Neurology Trinucleotide Repeat Disorders

Trinucleotide Repeat Disorders


A 48-year-old male is found by law enforcement homeless
on the street, he has multiple domestic violence issues. His
family is contacted and says that he has been very threatening
over the past few months. The patient during the interview is
noted to be smiling intermittently and moving his arms and
legs in an uncontrollable manner. His paternal uncle had
“some neuro issues” and passed away in middle age. What
brain structure may be affected in this patient?
• Atrophy of caudate (striatum) Huntington’s Disease
• What genetic inheritance pattern?
• Anticipation and trinucleotide CAG repeats
• What chromosome is affected?
• Chromosome 4, AD
Creative Commons Attribution License 4.0

USMLE Trinucleotide Repeats

Neurology: Trinucleotide Repeat Disorders

255
uploaded by medbooksvn
NBME Top Concepts ➤ Neurology Neurocutaneous Syndromes

NBME Style Question


A 2-year-old female presents with episodes concerning for seizures. On exam, her skin is noted to
have small flesh-colored, acne-like lesions. Wood lamp exam shows hypo-pigmented lesions on the
trunk and extremities. A murmur is heard. Which of the following echocardiographic findings would
be most likely seen in this patient?

A. Regurgitation of the aortic valve due to abnormal vaso-vasorum.


B. Pulmonic valve stenosis due to failed neural crest migration.
C. Mitral regurgitation due to intercavitary mass.
D. Mitral valve prolapse due to connective tissue defect.

Tuberous Sclerosis

Creative Commons Attribution License 4.0

Neurology: Neurocutaneous Syndromes

256
NBME Top Concepts ➤ Neurology Neurocutaneous Syndromes

Neuro-fibromatosis

Creative Commons Attribution License 4.0

Neurology: Neurocutaneous Syndromes

257
uploaded by medbooksvn
NBME Top Concepts ➤ Neurology Neurocutaneous Syndromes

Neurocutaneous Syndromes

Creative Commons Attribution License 4.0

Neurology: Neurocutaneous Syndromes

258
NBME Top Concepts ➤ Neurology Brain Tumors

Organization of Brain Tumors for the USMLE

Brain Tumor Rapid Review


GFAP+ cells + Rosenthal fibers + cystic/solid components + infratentorial mass?
• Astrocytoma

PNET+ cells + cystic mass at cerebellar vermis + drop metastasis + Homer Wright?
• Medulloblastomas

Rapidly progressive mass + midline crossing on MRI + pseudo-palisading tumor cells?


• Glioblastoma Multiforme

Middle aged female + new onset seizure + ER positive brain mass + psammoma bodies?
• Meningioma

Neurology: Brain Tumors

259
uploaded by medbooksvn
NBME Top Concepts ➤ Neurology Brain Tumors

Psammoma Bodies for the USMLE

Creative Commons Attribution License 4.0

Neurology: Brain Tumors

260
NBME Top Concepts ➤ Neurology Strokes

Test Taking Strategy: Approach to Stroke Questions on the USMLE

Isolate focal neurological deficit in vignette.


▸ Sensory and motor dysfunction = large vessel stroke.
▸ Innervation is contralateral
▸ Relate to the homunculus:
▹Arms and face lateral brain MCA
▹Trunk and LE medial brain ACA.

NBME Style Question


An 80-year-old male presents with left sided numbness and weakness. The patient is noted to
have prominent left arm weakness more than the leg. Patient has sensory deficits in the left arm.
Visual fields are normal. MRI scan is notable for an ischemic stroke. Which of the following vascular
structures may be affected in this patient?

A. Left Middle Cerebral Artery


B. Right Middle Cerebral Artery
C. Left Anterior Cerebral Artery
D. Right Anterior Cerebral Artery
E. Posterior Cerebral Artery
F. Lacunar Artery

Strokes for the USMLE

Neurology: Strokes

261
uploaded by medbooksvn
NBME Top Concepts ➤ Neurology Strokes

NBME Style Question


A 40-year-old male presents with vertebral trauma. He is noted to have R facial numbness. The
exam is notable for hoarseness. There is a reduced sensation of pain and temperature on the right
side of the face, and the left side of the body. He is noted to have ataxia when asked to walk. Which
of the following arteries may be affected in this patient?

A. Posterior Inferior Cerebellar Artery


B. Anterior Inferior Cerebellar Artery
C. Anterior Cerebral Artery
D. Posterior Cerebral Artery

PICA vs. AICA strokes for the USMLE

Neurology: Strokes

262
NBME Top Concepts ➤ Neurology Answer Key

Brain Hematomas
A 23-year-old male presents to trauma bay after MVC. He is noted to be awake and alert on
primary survey. Two hours later, he is noted to have a loss of consciousness. His vitals are 140/80,
pulse 60/min, RR 10. His pupil is mildly dilated, and bruising is noted in his temple. What is the likely
diagnosis?

A. Middle Cerebral Artery Ischemic Stroke.


B. Epidural Hematoma.
C. Subdural Hematoma.
D. Subarachnoid hemorrhage.
E. Rupture of Bouchard aneurysm.

Herniation Syndromes & Cerebral Physiology


A 6-year-old male presents to the intensive care unit after diagnosis of subarachnoid hemorrhage.
One hour after admission, his vital signs are notable for BP 140/80, HR 60, RR 12. He has a right
pupil minimally reactive and 4 mm in diameter, his left pupil is noted to be 2 mm and reactive. Given
normal cerebral autoregulation, which of the following interventions may reduce elevated
intracranial pressure in this patient?

A. Hypoventilation
B. No change in respirations
C. Increased blood pressure by stimulating alpha-receptors
D. Hyperventilation

Cranial Nerve Path (1)


A 16-year-old female presents with fever and throat pain. She is noted to have increased fatigue
for the past week. Vital signs are notable for fever. The physical exam is notable for exudates on
tonsils and left-sided uvular deviation. CT of the neck shows concern for injury to a cranial nerve.
Which of the following nerves related to this patient’s exam findings is most likely to be affected?

A. Vagus
B. Glossopharyngeal
C. Hypoglossal
D. Trochlear
E. Trigeminal

Neurology: Answer Key

263
uploaded by medbooksvn
NBME Top Concepts ➤ Neurology Answer Key

Dementia
A 40-year-old woman presents with a skin rash. She is noted by her family members to have
increased episodes of disorientation. The exam is notable for a skin rash on the arms along within
the buttocks region. Further history notes increased loose stools and BMI 18. The patient has poor
performance on the mental status exam. Which of the following is the most likely pathophysiologic
mechanism behind the diagnosis?

A. Nutritional deficiency.
B. Age-related changes.
C. Degeneration of the frontal and temporal regions.
E. Neurocutaneous syndrome.

Neurocutaneous Syndromes
A 2-year-old female presents with episodes concerning for seizures. On exam, her skin is noted to
have small flesh-colored, acne-like lesions. Wood lamp exam shows hypo-pigmented lesions on the
trunk and extremities. A murmur is heard. Which of the following echocardiographic findings would
be most likely seen in this patient?

A. Regurgitation of the aortic valve due to abnormal vaso-vasorum.


B. Pulmonic valve stenosis due to failed neural crest migration.
C. Mitral regurgitation due to intercavitary mass.
D. Mitral valve prolapse due to connective tissue defect.

Strokes

An 80-year-old male presents with left sided numbness and weakness. The patient is noted to
have prominent left arm weakness more than the leg. Patient has sensory deficits in the left arm.
Visual fields are normal. MRI scan is notable for an ischemic stroke. Which of the following vascular
structures may be affected in this patient?

A. Left Middle Cerebral Artery


B. Right Middle Cerebral Artery
C. Left Anterior Cerebral Artery
D. Right Anterior Cerebral Artery
E. Posterior Cerebral Artery
F. Lacunar Artery

Neurology: Answer Key

264
NBME Top Concepts ➤ Neurology Answer Key

Strokes
A 40-year-old male presents with vertebral trauma. He is noted to have R facial numbness. The
exam is notable for hoarseness. There is a reduced sensation of pain and temperature on the right
side of the face, and the left side of the body. He is noted to have ataxia when asked to walk. Which
of the following arteries may be affected in this patient?

A. Posterior Inferior Cerebellar Artery


B. Anterior Inferior Cerebellar Artery
C. Anterior Cerebral Artery
D. Posterior Cerebral Artery

Neurology: Answer Key

265
uploaded by medbooksvn
CHAPTER 13

High Yield Images &


PE Findings:
Neurology

266
High Yield Images & PE Findings ➤ Neurology Neuroanatomy

NBME Style Question


A 40-year-old male presents with uncontrolled jerking movements. He has had gait disturbances
in the past 3 weeks. He has a family history of dementia in his father. On exam, he has a grimace on
his face. He undergoes MRI and findings show a lesion in the area marked by red. Which of the
following neuroanatomical structures may be implicated?

A. Red Nucleus
B. Caudate
C. Thalamus
D. Internal Capsule
E. Cingulate Gyrus

Caudate Nucleus

Creative Commons Attribution License 4.0

Neurology: Neuroanatomy

267
uploaded by medbooksvn
High Yield Images & PE Findings ➤ Neurology Trinucleotide Repeat Disorders

Trinucleotide Repeat Disorders

USMLE Vignette
A 6-year-old male has dull headaches. He is noted to have
dizziness and blurry vision. He is an avid baseball player and
states he has frequent tunnel vision. An MRI is completed and
shows a mass (red arrows). Compression of which structure
is most likely responsible for the patient’s visual deficits?
• Optic Chiasm
• What is the likely diagnosis?
• Craniopharyngioma
• What is the embryological origin of this tumor?
• Rathke’s Pouch surface ectoderm
• Endocrine integration: patients may present with pituitary
hypofunction on the USMLE (i.e. low growth hormone, low
ACTH, etc.).
Creative Commons Attribution License 4.0

Neurology: Trinucleotide Repeat Disorders

268
High Yield Images & PE Findings ➤ Neurology Pituitary Masses

Craniopharyngioma

Creative Commons Attribution License 4.0


Creative Commons Attribution License 4.0

Neurology: Pituitary Masses

269
uploaded by medbooksvn
High Yield Images & PE Findings ➤ Neurology Thalamic Stroke

NBME Style Question


A patient presents with sensory deficits. She has a history of diabetes and hypertension. The
patient has diminished right side sensation of the face and pain, touch, temperature of the entire
side of right body. The patient has muscle strength that is normal. Speech, vision, and hearing are
also normal. What is the likely stroke this patient is exhibiting?

A. Pons
B. Thalamus
C. Somatosensory Cortex
D. Motor Cortex

Thalamus sandwiches the Third Ventricle

Creative Commons Attribution License 4.0

Neurology: Thalamic Stroke

270
High Yield Images & PE Findings ➤ Neurology Central Pontine Myelinolysis

Central Pontine Myelinolysis


A 35-year-old male with history of small cell lung cancer
presents with tonic clonic jerking of the extremities. He is
diagnosed with a seizure. He continues to seize in the ER.
Preliminary labs are notable for a Na of 125. He is
administered hypertonic saline & presents to the ICU, where
over the next 2 hours he has a normal Na. The patient
continues to have CNS dysfunction. Head imaging is
underwent and shown. What is the likely diagnosis?
• Central Pontine Myelinolysis
• What are the cranial nerves which could be affected?
• CN 9, 10, and 11.
Creative Commons Attribution License 4.0

NBME Style Question


Which of the following scenarios would likely show the change
in this diagram (solid line original; change in dashed line)?

A. Administration of 3% hypertonic saline


B. Administration of isotonic saline
C. SIADH
D. Diarrhea

Neurology: Central Pontine Myelinolysis

271
uploaded by medbooksvn
High Yield Images & PE Findings ➤ Neurology Internal Capsule and Lacunar Strokes

Neuroanatomy for the USMLE

A patient with history of DM, dyslipidemia and


smoking presents with inability to speak. The
patient is noted to have preserved sensation in all
of his extremities however it is noted that the
musculature of the face, arm and leg on the right
side are weak and paralyzed. Which of the
following structures may be implicated?
• B. Internal Capsule

Creative Commons Attribution License 4.0

Lacunar Strokes

Neurology: Internal Capsule and Lacunar Strokes

272
High Yield Images & PE Findings ➤ Neurology Internal Capsule and Lacunar Strokes

Neuroanatomy for the USMLE

A patient presents to the pathologist for autopsy. He was


diagnosed with a malignant brain tumor which was from the
astrocyte lineage. Gross pathology shows areas of hemorrhage
and necrosis in bilateral hemispheres. What is the likely brain
tumor?
• Glioblastoma multiforme

What structure allowed for this patient to have bilateral


involvement?
• Corpus callosum.

Neurology: Internal Capsule and Lacunar Strokes

273
uploaded by medbooksvn
High Yield Images & PE Findings ➤ Neurology Intracranial Tumours for the USMLE

Intracranial Tumors for the USMLE

Neurology: Intracranial Tumours for the USMLE

274
High Yield Images & PE Findings ➤ Neurology Multiple Sclerosis

Vignettes for Mutltiple Sclerosis

Creative Commons Attribution License 4.0

Neurology:Multiple Sclerosis

275
uploaded by medbooksvn
High Yield Images & PE Findings ➤ Neurology Cerebral Edema

NBME Style Question


A patient presents with head trauma. 48 hours after the trauma the patient is noted to have a
clinical decompensation and goes into a coma. Damage to which of the following structures may be
implicated in this patient’s coma?

A. Thalamus
B. Cerebellar vermis
C. Reticular Activating System
D. Internal Capsule

Monroe Kelly Doctrine

Creative Commons Attribution License 4.0

Neurology: Cerebral Edema

276
High Yield Images & PE Findings ➤ Neurology Intracranial Pressure

Intracranial Pressure

Creative Commons Attribution License 4.0

NBME Style Question


A 6-year-old male presents to the intensive care unit after diagnosis of subarachnoid hemorrhage.
One hour after admission, his vital signs are notable for BP 140/80, HR 60, RR 12. He has a right
pupil minimally reactive and 4 mm in diameter, his left pupil is noted to be 2 mm and reactive. Given
normal cerebral autoregulation, which of the following interventions may reduce elevated
intracranial pressure in this patient?

A. Hypoventilation
B. No change in respirations
C. Increased blood pressure by stimulating alpha-receptors
D. Hyperventilation

Neurology: Intracranial Pressure

277
uploaded by medbooksvn
High Yield Images & PE Findings ➤ Neurology Cerebral Autoregulation

Cerebral Autoregulation

Creative Commons Attribution License 4.0

Neurology: Cerebral Autoregulation

278
High Yield Images & PE Findings ➤ Neurology Baroreceptor Reflex

Baroreceptor Reflex Integration

Creative Commons Attribution License 4.0

Increased Intracranial Pressure

Neurology: Baroreceptor Reflex

279
uploaded by medbooksvn
High Yield Images & PE Findings ➤ Neurology Baroreceptor Reflex

Question
Which set of changes in plasma concentration would be expected to cause the greatest
activation of the chemoreceptor reflex?

Cerebral Edema

NBME Style Question


A patient after TBI has rigid extension of the upper and lower extremities and mid-positioned fixed
pupils. The lesion which explains this patient’s posturing is most likely:

A. Pons
B. Thalamus
C. Substantia Nigra
D. Cerebral Hemisphere

Neurology: Baroreceptor Reflex

280
High Yield Images & PE Findings ➤ Neurology Upper Motor Neuron Signs & Posturing

UMN Signs

Neurology: Baroreceptor Reflex

281
uploaded by medbooksvn
High Yield Images & PE Findings ➤ Neurology Spinal Cord Pathologies

High Yield Spinal Cord Anatomy for the USMLE

Spinal Cord Pathologies

Neurology: Spinal Cord Pathologies

282
High Yield Images & PE Findings ➤ Neurology Spinal Cord Pathologies

Methylmalonic Acidemia occurs due to lack of B12

Think Like the Test Maker B12 Deficiency

Neurology: Spinal Cord Pathologies

283
uploaded by medbooksvn
High Yield Images & PE Findings ➤ Neurology Spinal Cord Pathologies

Spinal Cord Pathologies

Spinal Cord Pathologies

Neurology: Spinal Cord Pathologies

284
High Yield Images & PE Findings ➤ Neurology Pseudotumor Cerebri

Pseudotumor Cerebri Swelling of the Optic Disk

Key Triggers in USMLE Questions

Creative Commons Attribution License 4.0

Neurology: Pseudotumor Cerebri

285
uploaded by medbooksvn
High Yield Images & PE Findings ➤ Neurology Hydrocephalus

Hydrocephalus for the USMLE

Hydrocephalus

Neurology: Hydrocephalus

286
High Yield Images & PE Findings ➤ Neurology Hydrocephalus

Hydrocephalus

A 75-year-old woman who presents with her son who says he has noticed her forgetting things.
She has had a history of evacuated sub-dural hematomas, and states she is embarrassed to come
to the physician because she “wets herself.” What is the likely diagnosis?
• NPH stretching of the corona radiate.

• Pathophysiology:
• Descending sacral ganglia nerve fibers going to the bladder stretch causes a reflex filling and
release of the bladder

• Questions will have triad of “wet, wobbly, and wacky”


• Incontinence
• Gait described as “magnetic” like there is gum on the floor
• Look for dilated ventricles on imaging.

Neurology: Hydrocephalus

287
uploaded by medbooksvn
High Yield Images & PE Findings ➤ Neurology CNS Infections

Meningitis

Neurology: CNS Infections

288
High Yield Images & PE Findings ➤ Neurology Answer Key

Neuroanatomy
A 40-year-old male presents with uncontrolled jerking movements. He has had gait disturbances
in the past 3 weeks. He has a family history of dementia in his father. On exam, he has a grimace on
his face. He undergoes MRI and findings show a lesion in the area marked by red. Which of the
following neuroanatomical structures may be implicated?

A. Red Nucleus
B. Caudate
C. Thalamus
D. Internal Capsule
E. Cingulate Gyrus

Thalamic Stroke

A patient presents with sensory deficits. She has a history of diabetes and hypertension. The
patient has diminished right side sensation of the face and pain, touch, temperature of the entire
side of right body. The patient has muscle strength that is normal. Speech, vision, and hearing are
also normal. What is the likely stroke this patient is exhibiting?

A. Pons
B. Thalamus
C. Somatosensory Cortex
D. Motor Cortex

Central Pontine Myelinolysis

Which of the following scenarios would likely show the change in this diagram (solid line original;
change in dashed line)?

A. Administration of 3% hypertonic saline


B. Administration of isotonic saline
C. SIADH
D. Diarrhea

Neurology: Answer Key

289
uploaded by medbooksvn
High Yield Images & PE Findings ➤ Neurology Answer Key

Cerebral Edema
A patient presents with head trauma. 48 hours after the trauma the patient is noted to have a
clinical decompensation and goes into a coma. Damage to which of the following structures may be
implicated in this patient’s coma?

A. Thalamus
B. Cerebellar vermis
C. Reticular Activating System
D. Internal Capsule

Intracranial Pressure
A 6-year-old male presents to the intensive care unit after diagnosis of subarachnoid hemorrhage.
One hour after admission, his vital signs are notable for BP 140/80, HR 60, RR 12. He has a right
pupil minimally reactive and 4 mm in diameter, his left pupil is noted to be 2 mm and reactive. Given
normal cerebral autoregulation, which of the following interventions may reduce elevated
intracranial pressure in this patient?

A. Hypoventilation
B. No change in respirations
C. Increased blood pressure by stimulating alpha-receptors
D. Hyperventilation

Baroreceptor Reflex
A patient after TBI has rigid extension of the upper and lower extremities and mid-positioned fixed
pupils. The lesion which explains this patient’s posturing is most likely:

A. Pons
B. Thalamus
C. Substantia Nigra
D. Cerebral Hemisphere

Neurology: Answer Key

290
CHAPTER 14

Top NBME Concepts:


Renal

291
uploaded by medbooksvn
NBME Top Concepts ➤ Renal Renal Casts

NBME Style Question


A 69-year-old male comes in with back-pain, constipation & fatigue. Patient vital signs are stable.
On exam, the patient has dry mucous membranes.
Hgb: 8.6 (L)
MCV: 92
BUN: 68 (H)
Cr: 3.8 (H)
Renal biopsy is notable for atrophic tubules, waxy casts which stain intensely with eosin. What is
the most likely diagnosis?

a.Pyelonephritis
b.Urate Nephropathy
c.Renal Papillary Necrosis
d.Plasma cell Neoplasm
e.Hypersensitivity interstitial nephritis

Approach to Back Pain for the USMLE

Renal: Renal Casts

292
NBME Top Concepts ➤ Renal Renal Casts

Test-taking Strategy: Urinalysis


You will likely see this in the labs portion of your vignette on the USMLE.
▸UA:
▹ Specific gravity (1.005-1.030)
• High?
• Concentrated pee
• Low?
• Dilute pee
▹ Leukocyte esterase
• WBC in the pee (may indicate UTI)
▹ Nitrites
• Indicative of gram negative bacteria
▹ Other
• Heme, Protein, Glucose, Ketones.

Integrative Vignette
A patient presents after being found crushed under a fallen set of bricks. Bruising is noted
throughout the body. He is noted to have a UA which shows +heme. On urine microscopy he has no
RBCs noted. Creatinine is elevated. What is the likely diagnosis?
▹ Rhabdomyolysis:
• Concept: urine dipstick cannot differentiate between hematuria and myoglobinuria.

Urinalysis

Whenever you see casts on UA think about tubular or


glomerular damage.

Casts are proteins which take the form of the tubule

Creative Commons Attribution License 4.0

Renal: Renal Casts

293
uploaded by medbooksvn
NBME Top Concepts ➤ Renal Renal Casts

USMLE Vignette

Creative Commons Attribution License 4.0

Renal: Renal Casts

294
NBME Top Concepts ➤ Renal Kidney Stones

USMLE Test Taking Strategy: Regional Localization for Abdominal Pain

USMLE Questions localize pertinent areas of abdominal pain on PE allows you to isolate
diagnosis better

Use this as a pertinent positive as you paraphrase the vignette.

NBME Style Question


A 40-year-old male comes in with R flank pain and nausea which radiates to the groin. He is
tachycardic and has mild tenderness to percussion on the R side. Testicular US is negative. UA
shows dysmorphic RBCs. Which of the following recommendations would prevent this pathology?

a.Limit fluid intake.


b.High protein diet.
c.Increase calcium in diet.
d.High sodium diet.
e.Use of loop diuretics.

Renal: Kidney Stones

295
uploaded by medbooksvn
NBME Top Concepts ➤ Renal Kidney Stones

Risk Factors for Kidney Stones


Hypercalciuria:
• Hyperparathyroidism.
• Furosemide.
Diet:
• in sodium and protein
• calcium
• Dehydration
Other stones:
• Uric acid stones watch for patients with gout or CML, PV, etc.
• Struvite stones watch for Klebsiella or Proteus UTI.

Nephrolithiasis
A large kidney stone in the distal ureter may cause what ultrasound finding?
▹ Hydronephrosis.
In USMLE questions, what may urine microscopy show?
▹Free RBCs disruption of the ureteral epithelium
• Ureter blood supply:
• Proximal ureter: renal artery
• Distal ureter: superior vesicular artery (branch of internal iliac).

Renal: Kidney Stones

296
NBME Top Concepts ➤ Renal Kidney Stones

Ureteral Blood Supply

Creative Commons Attribution License 4.0

NBME Style Question


A middle age female presents with flank pain and hematuria. She has a history of UTIs. She has a
blood pressure of 150/80. Her prior urine cultures have grown Klebsiella, Proteus. CT is notable for
calyceal dilation. Which of the following describes the pathogenesis of this patient’s UTI?

a.Small stone in distal ureter.


b.Stone composed of Mg Ammonia and Ca2+
c.Hydrolysis of urea precipitating alkaline urinary pH.
d.Low urinary specific gravity.

Renal: Kidney Stones

297
uploaded by medbooksvn
NBME Top Concepts ➤ Renal Urea Cycle + Stones (pt. 2)

Urea Cycle for the USMLE


Why does our body need this pathway?
▸To take Ammonia (NH3) and convert it to a solubilized form Urea
Where in the body does this process occur?
▸Hepatocytes we excrete urea in urine
• Key USMLE question: Pt with liver failure + asterixis
What is the rate limiting enzyme of the urea cycle?
▸CPS I
• Where is CPS II found?
• In the cytosol and found in pyrimidine synthesis
What is the allosteric or hormonal feedback?
• Activated by n-acetylgultamate.

USMLE Questions related to the UREA Cycle

Renal: Urea Cycle + Stones (pt. 2)

298
NBME Top Concepts ➤ Renal Urea Cycle + Stones (pt. 2)

Pathogenesis of AMP Stones


A patient presents with stones that are obstructing the renal
pelvis. Proteus mirabilis is isolated on urine culture. What is
the mechanism behind these calculi?
• Proteus is urease+ hydrolyzes urea to NH3 (gives an
pH) AMP stones

What are other urease+ organisms?


• Proteus
• Ureaplasma
• Nocardia
• Cryptococcus
• H. pylori
Creative Commons Attribution License 4.0

Uric Acid stones

What radiologic characteristic is notable with these stones?


• Radiolucent.
• Along with cystine stones.
Uric acid and cysteine stones:
• Alkalinize the urine for management.
Remember:
• CaP04 & AMP stones form in alkaline urine whereas cysteine and uric acid stones form in acidic
urine
USMLE Question for uric acid stone:
• A patient who has recurrent gout presents with dehydration + kidney stone. A urine analysis
shows urine pH being low. What would urine microscopy show?
• Rhomboid shaped crystals.

Creative Commons Attribution License 4.0

Renal: Urea Cycle + Stones (pt. 2)

299
uploaded by medbooksvn
NBME Top Concepts ➤ Renal Urea Cycle + Stones (pt. 2)

Calcium Oxalate Stones


A patient with Chron’s disease presents with flank pain + hematuria. A diagnosis of nephrolithiasis is
made. What is the mechanism behind the likely stone?
• Malabsorption free lipids calcium binds to free lipids
• Oxalate is free gets circulated and deposited in the kidney stones.

USMLE Vignette

Creative Commons Attribution License 4.0

Renal: Urea Cycle + Stones (pt. 2)

300
NBME Top Concepts ➤ Renal Intro to Nephritic/Nephrotic

Broad Overview of Glomerular Disease for the USMLE

Renal: Intro to Nephritic/Nephrotic

301
uploaded by medbooksvn
NBME Top Concepts ➤ Renal Nephrotic Syndrome

Nephrotic Syndrome
A child presents with recurrent anasarca. The patient has recurrent infections and varicoceles. Labs
are notable for hyperlipidemia. UA shows fatty casts.
What is the mechanism behind:
▹Anasara?
• Loss of oncotic pressure
▹Hyperlipidemia?
• Reactive increase in lipoprotein synthesis (compensatory mechanism)
▹Recurrent infections?
• Loss of immunoglobulins
▹Recurrent varicoceles, DVT, renal/mesenteric vein thrombosis?
• Loss of ATIII

Nephrotic Syndrome
A child presents with scrotal edema and eye-puffiness. He had a history of URI one week prior. A
diagnosis of minimal change disease is made. What is the most likely light microscopy finding?
▸Normal glomeruli
Pathophysiology:
▸Cytokines loss of charge to basement membrane diffuse podocyte effacement on electron
microscopy.
What is the likely management?
▸Corticosteroids (very steroid responsive).

Minimal Change disease

Creative Commons Attribution License 4.0

Renal: Nephrotic Syndrome

302
NBME Top Concepts ➤ Renal Nephrotic Syndrome

Nephrotic Syndrome
A 50-year-old obese AA with PMH of HIV and sickle cell disease states he has been noticing
increased fatigue and swelling in his leg. He is non-compliant with his treatment and social history
demonstrates long standing heroin abuse. On labs, he has 3+ proteinuria and fatty casts. What is the
likely diagnosis?
▸Focal Segmental Glomerulosclerosis
▸What may electron microscopy show?
▹Podocyte effacement like minimal change disease
▹Corticosteroids (poorly steroid responsive).

FSGS

Creative Commons Attribution License 4.0

NBME Style Question


A 40-year-old female with history of lupus presents with proteinuria. She is noted to have ANA+,
and anti-dsDNA antibody. Kidney biopsy is determined to be the next best step. Sample is notable
for irregular spikes protruding from the GBM. What is the likely diagnosis?

a.Minimal change disease


b.Diffuse proliferative glomerulonephritis
c.Membranous glomerulopathy
d.Membranoproliferative glomerulonephritis

Renal: Nephrotic Syndrome

303
uploaded by medbooksvn
NBME Top Concepts ➤ Renal Nephrotic Syndrome

Membranous Glomerulopathy

Creative Commons Attribution License 4.0

Membranoproliferative Glomerulonephritis

Renal: Nephrotic Syndrome

304
NBME Top Concepts ➤ Renal Nephrotic Syndrome

USMLE Vignette

Summary of Membranous Diseases

Renal: Nephrotic Syndrome

305
uploaded by medbooksvn
NBME Top Concepts ➤ Renal Nephrotic Syndrome

Nephrotic Syndrome
What is the most common cause of ESRD in the United States?
▸Diabetic Glomerulonephropathy.

Pathophysiology of Diabetic Nephropathy

Renal: Nephrotic Syndrome

306
NBME Top Concepts ➤ Renal Nephritic Syndrome

Nephritic syndrome
A child presents with recurrent hematuria. The patient is noted to have hypertension, oliguria and
peri-orbital edema.
▸What is the mechanism behind:
▹Hypertension?
• Salt retention + inflammation
▹Peri-orbital edema?
• Salt retention fluid deposition in loose areolar tissue (potential space for fluid).
▸What does UA typically show?
▹Dysmorphic RBCs and RBC casts.

Pathophysiology of Inflammation in Nephritic Syndrome

Nephritic Syndrome
A 6-year-old male presents with blood in his urine after having a URI + sore-throat. He is noted to be
hypertensive. UA confirms hematuria + non-nephrotic range proteinuria. What is the most likely
diagnosis?
▸ Well.. it is difficult to know as there is no time period…
▸2-3 days after URI and sore-throat?
▸IgA nephropathy
▸2-4 wks after URI and sore-throat?
▸Post-streptococcal glomerulonephritis.

Renal: Nephritic Syndrome

307
uploaded by medbooksvn
NBME Top Concepts ➤ Renal Nephritic Syndrome

IgA Nephropathy

USMLE Integration
A 6-year-old child has bloody stools and colicky abdominal pain. He is noted to be tachycardic and
hypertensive. There are raised, purple red skin lesions along the buttocks and lower extremities.
What is the likely diagnosis?
▸Henoch Schoenlein Purpura
▸IgA vasculitis
▸USMLE Presentations:
▸Arthritis
▸Palpable purpura on LE
▸Renal disease
▸Intussusception

NBME Style Question


A child is noted to have red urine. He has a hx of atopic dermatitis. He was treated for a skin
infection few weeks ago. He is noted to have hypertension and a BUN:Cr notable for intrinsic renal
damage. What is the likely mechanism behind this patient’s condition?

A. CD8+ T Lymphocytes.
B. Histamine release.
C. Auto-immune IgG antibodies.
D. IgG immune complexes.

Renal: Nephritic Syndrome

308
NBME Top Concepts ➤ Renal Nephritic Syndrome

Post-streptococcal Glomerulonephritis

Creative Commons Attribution License 4.0

Lupus Associated Renal Disease

Renal: Nephritic Syndrome

309
uploaded by medbooksvn
NBME Top Concepts ➤ Renal Nephritic Syndrome

Hemoptysis & Hematuria Syndromes for the USMLE

How do you tell MPA from Churg Strauss?


P-ANCA positive for both.
Churg strauss has asthma and eosinophilia, and granuloma.

Renal: Nephritic Syndrome

310
NBME Top Concepts ➤ Renal Renal Failure

NBME Style Question


A 50-year-old male is admitted for heart failure. He is started on furosemide. He is noted to have a
baseline BUN of 12 mg/dL, and Cr of 1.2 mg/dL. On day 5 of hospitalization, it is noted his BUN has
risen to 40 mg/dL and Cr to 2.1. What is the likely mechanism?

A. Increased upregulation of aquaporins.


B. Ureteral compression leading to hydronephrosis.
C. Tubular necrosis.
D. Decreased efferent perfusion.

Pathophysiology of Pre-Renal Azotemia

USMLE Vignette

Renal: Renal Failure

311
uploaded by medbooksvn
NBME Top Concepts ➤ Renal Diuretics

NBME Style Question


A 65-year-old male presents with skin rash and fever. He is noted to have a history of gout and
CHF controlled on furosemide and NSAIDs. UA is notable for pyuria and no nitrites. Urine
microscopy shows eosinophils. What is the likely mechanism?

a. Immune complex deposition.


b. Auto-antibody inflammation.
c. IgE mediated hypersensitivity reaction.
d. CD8+ interstitial damage.

USMLE test-taking tip:


• Isolate triggers in test-questions NSAIDS + diuretics cause Acute Interstitial Nephritis.

Integrative Review of Diuretics for the USMLE Step 1

Renal: Diuretics

312
NBME Top Concepts ➤ Renal Diuretics

Pharmacology Compare and Contrast

Pathophysiology of Contraction alkalosis

Renal: Diuretics

313
uploaded by medbooksvn
NBME Top Concepts ➤ Renal Answer Key

Renal Casts
A 69-year-old male comes in with back-pain, constipation & fatigue. Patient vital signs are stable.
On exam, the patient has dry mucous membranes.
Hgb: 8.6 (L)
MCV: 92
BUN: 68 (H)
Cr: 3.8 (H)
Renal biopsy is notable for atrophic tubules, waxy casts which stain intensely with eosin. What is
the most likely diagnosis?

a.Pyelonephritis
b.Urate Nephropathy
c.Renal Papillary Necrosis
d.Plasma cell Neoplasm
e.Hypersensitivity interstitial nephritis

Kidney Stones
A 40-year-old male comes in with R flank pain and nausea which radiates to the groin. He is
tachycardic and has mild tenderness to percussion on the R side. Testicular US is negative. UA
shows dysmorphic RBCs. Which of the following recommendations would prevent this pathology?

a.Limit fluid intake.


b.High protein diet.
c.Increase calcium in diet.
d.High sodium diet.
e.Use of loop diuretics.

A middle age female presents with flank pain and hematuria. She has a history of UTIs. She has a
blood pressure of 150/80. Her prior urine cultures have grown Klebsiella, Proteus. CT is notable for
calyceal dilation. Which of the following describes the pathogenesis of this patient’s UTI?

a.Small stone in distal ureter.


b.Stone composed of Mg Ammonia and Ca2+
c.Hydrolysis of urea precipitating alkaline urinary pH.
d.Low urinary specific gravity.

Renal: Answer Key

314
NBME Top Concepts ➤ Renal Answer Key

Nephrotic Syndrome
A 40-year-old female with history of lupus presents with proteinuria. She is noted to have ANA+,
and anti-dsDNA antibody. Kidney biopsy is determined to be the next best step. Sample is notable
for irregular spikes protruding from the GBM. What is the likely diagnosis?

a.Minimal change disease


b.Diffuse proliferative glomerulonephritis
c.Membranous glomerulopathy
d.Membranoproliferative glomerulonephritis

Nephritic Syndrome
A child is noted to have red urine. He has a hx of atopic dermatitis. He was treated for a skin
infection few weeks ago. He is noted to have hypertension and a BUN:Cr notable for intrinsic renal
damage. What is the likely mechanism behind this patient’s condition?

A. CD8+ T Lymphocytes.
B. Histamine release.
C. Auto-immune IgG antibodies.
D. IgG immune complexes.

Renal Failure
A 50-year-old male is admitted for heart failure. He is started on furosemide. He is noted to have a
baseline BUN of 12 mg/dL, and Cr of 1.2 mg/dL. On day 5 of hospitalization, it is noted his BUN has
risen to 40 mg/dL and Cr to 2.1. What is the likely mechanism?

A. Increased upregulation of aquaporins.


B. Ureteral compression leading to hydronephrosis.
C. Tubular necrosis.
D. Decreased efferent perfusion.

Diuretics
A 65-year-old male presents with skin rash and fever. He is noted to have a history of gout and
CHF controlled on furosemide and NSAIDs. UA is notable for pyuria and no nitrites. Urine
microscopy shows eosinophils. What is the likely mechanism?

a. Immune complex deposition.


b. Auto-antibody inflammation.
c. IgE mediated hypersensitivity reaction.
d. CD8+ interstitial damage.

Renal: Answer Key

315
uploaded by medbooksvn
CHAPTER 15

High Yield USMLE


Images & PE Findings:
Renal

316
High Yield Images & PE Findings ➤ Renal Renal Stones

NBME Style Question


A patient presents with left groin pain and microscopic hematuria. Bowel sounds are hypoactive.
Which of the following recommendations would most likely prevent a representation of this patient’s
illness?

A. Safe sex
B. Avoidance of lactose
C. Low Ca diet
D. Increased fluid Intake
E. Follow high fiber diet

Renal Casts

What do casts represent?


• Entrapped cells, debris, protein which form in
tubular lumens as they leak through glomerulus.

Creative Commons Attribution License 4.0

Renal: Renal Stones

317
uploaded by medbooksvn
High Yield Images & PE Findings ➤ Renal Renal Stones

Urine Analysis Interpretation for the USMLE

NBME Style Question


A child presents unresponsive. He was playing in the garage last with some fluid stored in a
Gatorade bottle. Airway measures are undertaken. The patient passes away due to renal failure. An
autopsy shows multiple crystals in the tubular lumen. Which of the following substances are these
crystals likely composed of?

A. Cystine
B. Uric Acid
C. Ammonia Mag Phos
D. Oxalate
E. Lactate

Endocrine Integration

A patient presents with bone pain. She is noted to have an


elevated iCa. Serum studies show an elevated PTH level. UA
is notable for microscopic hematuria. What is the likely
mechanism of disease?
• HyperPTH increased serum calcium calciuria.
• Calcium stones are the most common in
nephrolithiasis.

From Goldman L, Schafer A: Goldman’s Cecil Medicine, 25th ed, Philadelphia, Elsevier Saunders, 2016, p 813, Fig. 126-2.

Renal: Renal Stones

318
High Yield Images & PE Findings ➤ Renal Renal Stones

Microbiology Integration
A patient presents with flank pain. The patient is afebrile. An ultrasound of the abdomen is
notable for cortical dilation. UA is positive for LE. A gram negative bacili is isolated on urine culture.
When plated, the bacteria catalyzes a reaction from NH2 to ammonia. What is the likely diagnosis?
• A Struvite Stone 2/2 Proteus Mirabilis UTI
• What is the composition of these stones?
• Ammonium, Magnesium, Phosphate
• Relate to Urease + organisms.

Staghorn Calculus

Creative Commons Attribution License 4.0

Gastrointestinal Integration
A patient with history of weight loss and foul smelling
stools presents with flank pain that radiates to the groin.
She has had endoscopies which have revealed non-
caseating granulomas in duodenum. ESR is elevated. The
patient is dx with a kidney stone in the mid ureter. What is
the likely mechanism of the nephrolithiasis?
• Calcium Oxalate Stones related to Chron’s Disease.

Renal: Renal Stones

319
uploaded by medbooksvn
High Yield Images & PE Findings ➤ Renal Renal Stones

Calcium Oxalate Stones in Inflammatory Bowel Disease

NBME Style Question


A patient with history of type 2 DM, obesity, and hyperlipidemia presents with vomiting and flank
pain. He has no fever. He is found to have hydronephrosis of the left kidney and proximal ureter
dilation on renal ultrasound. What is the most likely finding which would be found in this patient’s
urine analysis?

A. Malignant tumor cells


B. RBCs
C. Pyuria
D. Red cell casts
E. Low specific gravity

Renal: Renal Stones

320
High Yield Images & PE Findings ➤ Renal Renal Stones

Nephrolithiasis

Nephrolithiasis

Renal: Renal Stones

321
uploaded by medbooksvn
High Yield Images & PE Findings ➤ Renal Polycystic Kidneys

NBME Style Question


A patient is found to have recurrent pyelonephritis. She is noted to have an anatomical abnormality
of the kidney centered around a major aortic vessel. Which of the following vessels may be the likely
etiology of the pathology?

A. Inferior Mesenteric Artery


B. Inferior Vena Cava
C. Splenic Artery
D. Renal vein

Horseshoe Kidney

Creative Commons Attribution License 4.0

Renal: Polycystic Kidneys

322
High Yield Images & PE Findings ➤ Renal Polycystic Kidneys

Gastrointestinal Anatomy Integration

What is the degree of rotation of the Midgut?


• 270 degrees Counter-CLOCKWISE at week 10 of development

Renal Embryology

Creative Commons Attribution License 4.0

Renal: Polycystic Kidneys

323
uploaded by medbooksvn
High Yield Images & PE Findings ➤ Renal Polycystic Kidneys

Amniotic Fluid Abnormalities for the usmle

General Pathology Integration

Renal: Polycystic Kidneys

324
High Yield Images & PE Findings ➤ Renal Polycystic Kidneys

NBME Style Question


A 50-year-old male presents with sudden onset headache. Exam shows nuchal rigidity and
photophobia. The patient has no cranial nerve deficits or trauma. He is otherwise healthy. Family
history is notable for his brother having cystic kidneys. What is the likely diagnosis?

A. Cluster headache
B. Migraine
C. Subarachnoid hemorrhage
D. Lobar hemorrhage
E. Bacterial meningitis

Autosomal Dominant Polycystic Kidney Disease

Renal: Polycystic Kidneys

325
uploaded by medbooksvn
High Yield Images & PE Findings ➤ Renal Polycystic Kidneys

Polycystic Kidney Disease

Congenital Renal Tract Disorders

A 2-week male presents with oliguria and palpable mass


in lower abdomen. Prenatal records reveal
oligohydramnios. What is the likely finding seen on
ultrasound?
•Thick bladder with bilateral hydronephrosis diagnosis:
Posterior Urethral Valve

Creative Commons Attribution License 4.0

Renal: Polycystic Kidneys

326
High Yield Images & PE Findings ➤ Renal Polycystic Kidneys

Azotemia

NBME Style Question


A child presents with recurrent urinary tract infections. She undergoes renal ultrasound which
reveals dilated renal calyces and cortical atrophy of the upper and lower renal poles. There are
changes in the ureter anatomy as well. Which of the following mechanisms best explains her clinical
complaint?

A. Malignant hypertension
B. Childhood lupus
C. Vesicoureteral Reflux
D. AR Polycystic kidney disease

Renal: Polycystic Kidneys

327
uploaded by medbooksvn
High Yield Images & PE Findings ➤ Renal Nephrotic & Nephritic Syndromes

Azotemia

Approach to Understanding Glomerular Diseases

Renal: Nephrotic & Nephritic Syndromes

328
High Yield Images & PE Findings ➤ Renal Nephrotic & Nephritic Syndromes

Presentation of Glomerular Diseases on the USMLE

Normal Glomerular Anatomy

Renal: Nephrotic & Nephritic Syndromes

329
uploaded by medbooksvn
High Yield Images & PE Findings ➤ Renal Nephrotic & Nephritic Syndromes

Minimal Change Disease


• What will light microscopy show?
• Normal glomeruli
• What is the mechanism behind the electron microscopy findings?
• Loss of charge of basement membrane + effacement of the podocyte.
• What causes destruction of the podocyte?
• T cells from infection cytokines (IL-5, for Hodgkin's) effacement.

Minimal Change Disease

Focal Segmental Glomerulosclerosis


A 50-year-old obese, AA with PMH of HIV and sickle cell disease states he has been noticing
increased fatigue and swelling in his leg. He is non-compliant with his treatment and social history
demonstrates long standing heroin abuse. On labs, he has 3+ proteinuria and fatty casts.
• What may his Electron Microscopy show?
• Effacement of podocytes
• How do you tell the difference between this and MCD?
• Patient in FSGS do not respond to steroids
• MCD FSGS

Renal: Nephrotic & Nephritic Syndromes

330
High Yield Images & PE Findings ➤ Renal Nephrotic & Nephritic Syndromes

Minimal Change Disease


• What will light microscopy show?
• Normal glomeruli
• What is the mechanism behind the electron microscopy findings?
• Loss of charge of basement membrane + effacement of the podocyte.
• What causes destruction of the podocyte?
• T cells from infection cytokines (IL-5, for Hodgkin's) effacement.

Minimal Change Disease

Membranous Glomerulonephropathy
• Most common cause of nephrotic syndrome in a Lupus patient?
• Membranous nephropathy
• Most common overall renal syndrome associated with Lupus:
• Diffuse proliferative glomerulonephritis
• Membranous nephropathy can also be due to what autoimmune other phenomena?
• Antibodies to PLA2 Type II HS.

Renal: Nephrotic & Nephritic Syndromes

331
uploaded by medbooksvn
High Yield Images & PE Findings ➤ Renal Nephrotic & Nephritic Syndromes

Membranous Glomerulonephropathy

Nephrotic Syndrome

Renal: Nephrotic & Nephritic Syndromes

332
High Yield Images & PE Findings ➤ Renal Nephrotic & Nephritic Syndromes

Lupus Associated Renal Disease

Creative Commons Attribution License 4.0

Mebranoproliferative Glomerulonephritis

Renal: Nephrotic & Nephritic Syndromes

333
uploaded by medbooksvn
High Yield Images & PE Findings ➤ Renal Nephrotic & Nephritic Syndromes

Mebranoproliferative Glomerulonephritis

Summary of Membranous Disorders

Renal: Nephrotic & Nephritic Syndromes

334
High Yield Images & PE Findings ➤ Renal Nephrotic & Nephritic Syndromes

Nephritic Syndromes
A 6-year-old male presents with blood in his urine after having a URI + sore-throat. He is noted to
be hypertensive. UA confirms hematuria + non-nephrotic range proteinuria. What is the most likely
diagnosis?
• Well.. it is difficult to know as there is no time period…
• 2-3 days after URI and sore-throat?
• IgA nephropathy
• 2-4 wks after URI and sore-throat?
• Post-streptococcal glomerulonephritis.

IgA Nephropathy & Post Streptococcal Glomerulonephritis

Renal: Nephrotic & Nephritic Syndromes

335
uploaded by medbooksvn
High Yield Images & PE Findings ➤ Renal Nephrotic & Nephritic Syndromes

Glomerulonephritis Images

Nephritic Syndrome

Renal: Nephrotic & Nephritic Syndromes

336
High Yield Images & PE Findings ➤ Renal Tumors of the Urinary Tract

NBME Style Question


A 15 month old male brought in by mother after she noticed he had decreased appetite and a
mass felt while she was bathing him. On CT of the abdomen, the mass does not cross the midline
and is confined to the retroperitoneum. What is the likely diagnosis?

A. Hepatoblastoma
B. Neuroblastoma
C. Nephroblastoma
D. Hepatorenal syndrome

Differentiation of Wilms Tumor vs. Neuroblastoma

Renal: Tumors of the Urinary Tract

337
uploaded by medbooksvn
High Yield Images & PE Findings ➤ Renal Tumors of the Urinary Tract

Summary of Urinary Tumors

Renal: Tumors of the Urinary Tract

338
High Yield Images & PE Findings ➤ Renal Answer Key

Renal Stones

A patient presents with left groin pain and microscopic hematuria. Bowel sounds are hypoactive.
Which of the following recommendations would most likely prevent a representation of this patient’s
illness?

A. Safe sex
B. Avoidance of lactose
C. Low Ca diet
D. Increased fluid Intake
E. Follow high fiber diet

A child presents unresponsive. He was playing in the garage last with some fluid stored in a
Gatorade bottle. Airway measures are undertaken. The patient passes away due to renal failure. An
autopsy shows multiple crystals in the tubular lumen. Which of the following substances are these
crystals likely composed of?

A. Cystine
B. Uric Acid
C. Ammonia Mag Phos
D. Oxalate
E. Lactate

A patient with history of type 2 DM, obesity, and hyperlipidemia presents with vomiting and flank
pain. He has no fever. He is found to have hydronephrosis of the left kidney and proximal ureter
dilation on renal ultrasound. What is the most likely finding which would be found in this patient’s
urine analysis?

A. Malignant tumor cells


B. RBCs
C. Pyuria
D. Red cell casts
E. Low specific gravity

Renal: Answer Key

339
uploaded by medbooksvn
High Yield Images & PE Findings ➤ Renal Answer Key

Polycystic Kidneys

A patient is found to have recurrent pyelonephritis. She is noted to have an anatomical abnormality
of the kidney centered around a major aortic vessel. Which of the following vessels may be the likely
etiology of the pathology?

A. Inferior Mesenteric Artery


B. Inferior Vena Cava
C. Splenic Artery
D. Renal vein

A child presents with recurrent urinary tract infections. She undergoes renal ultrasound which
reveals dilated renal calyces and cortical atrophy of the upper and lower renal poles. There are
changes in the ureter anatomy as well. Which of the following mechanisms best explains her clinical
complaint?

A. Malignant hypertension
B. Childhood lupus
C. Vesicoureteral Reflux
D. AR Polycystic kidney disease

Tumors of the Urinary Tract


A 15 month old male brought in by mother after she noticed he had decreased appetite and a
mass felt while she was bathing him. On CT of the abdomen, the mass does not cross the midline
and is confined to the retroperitoneum. What is the likely diagnosis?

A. Hepatoblastoma
B. Neuroblastoma
C. Nephroblastoma
D. Hepatorenal syndrome

Renal: Answer Key

340
CHAPTER 16

Top NBME Concepts:


Reproductive

341
uploaded by medbooksvn
NBME Top Concepts ➤ Reproductive Disorders of Sexual Development

Test Taking Strategy for Embryology Reproductive Questions

NBME Style Question


A 15-year-old boy presents with delayed growth spurt. He has had decreased hair growth in his
pubic region. Intelligence test is normal. Audiometry evaluation is normal. He is unable to
distinguish smells. Ophthalmic exam is normal. Which of the following mechanisms may explain
this patient’s disease?

A. Primary seminiferous tubule absence.


B. Genetic X,O Karyotype.
C. Absence of GnRH secretory neurons.
D. Rathke’s pouch dysfunction.
E. Failure of somatotroph development in the pituitary.

Reproductive: Disorders of Sexual Development

342
NBME Top Concepts ➤ Reproductive Disorders of Sexual Development

Kalman Syndrome

An adult with anosmia and failure of pubertal


development presents to the physician for
evaluation of infertility. What laboratory
abnormalities may be expected?
Low GnRH, FSH, LH, and testosterone.
Failure of GnRH-releasing olfactory bulbs to
migrate. What bony structure does the
olfactory nerve exit the cranial cavity?
• Cribiform plate of the ethmoid bone.

Creative Commons Attribution License 4.0

Disorders of Sexual Development


A 16-year-old female presents for evaluation of primary amenorrhea. She has on growth chart
short stature, and on exam broad spaced nipples. What are the cardiovascular malformations
behind the likely diagnosis?
Coarctation of the aorta and bicuspid aortic valve.

Turner Syndrome

Creative Commons Attribution License 4.0

Reproductive: Disorders of Sexual Development

343
uploaded by medbooksvn
NBME Top Concepts ➤ Reproductive Disorders of Sexual Development

NBME Style Question


A medical school anatomy class is undergoing anatomy review in the cadaveric lab. On one of the
cadaveric specimen’s, it is noted that the patient has a bicuspid aortic valve. Which of the following
murmurs is consistent with this finding’s downstream pathology?

A. Diastolic murmur heard at the sternal border.


B. A systolic murmur with click heard at the apex.
C. Diastolic murmur heard at the R second ICS.
D. Systolic murmur heard at the R second ICS.

Bicuspid Aortic Valve can lead to Premature Aortic Stenosis (Triad)

Reproductive: Disorders of Sexual Development

344
NBME Top Concepts ➤ Reproductive Turner's Syndrome Integration

Systemic Manifestations of Turner's for the USMLE

Key Chest X-Ray Findings Related To Turner's Syndrome

Rib Notching = Engorgement of


Intercostal Arteries

Creative Commons Attribution License 4.0

Reproductive: Turner's Syndrome Integration

345
uploaded by medbooksvn
NBME Top Concepts ➤ Reproductive Turner's Syndrome Integration

Cardiology Integration – Genetic defects

Turner's Syndrome vs. Klinefelter's Syndrome

Reproductive: Turner's Syndrome Integration

346
NBME Top Concepts ➤ Reproductive Testosterone Disorders

NBME Style Question


A newborn is evaluated in the delivery room. The baby’s delivery course was unremarkable.
Mother’s pregnancy was uncomplicated. Ambiguous genitalia and clitoromegaly are notable on
baby’s exam. Mother states that during the latter half of her pregnancy she noticed her voice deepen
and have dark coarse facial hair. Remaining maternal and baby exam is normal. Which of the
following mechanisms accounts for this infant’s pathology?

A. Activation of 21-Hydroxylase.
B. Deficiency in 17-Hydroxylase.
C. Placental Aromatase Deficiency.
D. Increased HMG-CoA Reductase Activity.

Turner's Syndrome vs. Klinefelter's Syndrome


Children with 5-ɑ-reductase deficiency have male internal genital organs ambiguous genitalia
at birth
At puberty no phallic growth

Reproductive: Testosterone Disorders

347
uploaded by medbooksvn
NBME Top Concepts ➤ Reproductive Mullerian Agenesis VS. AIS

Question
A 16-year-old girl comes to her pediatrician with the complaint that she "has never had a
menstrual period.” Her exam reveals a short, blind-ending vagina, and normal breast and pubic
hair development. A uterus is not visualized on US. What is the likely diagnosis?
Mullerian Agenesis
• Key to these questions on the USMLE:
•Primary amenorrhea (XX) with fully developed secondary sex characteristics.

Mullerian Structures

NBME Style Question


A 16-year-old female presents with primary amenorrhea. She has had breast engorgement
however she has not had any pubic hair. On exam, she is noted to have a blind ended vaginal pouch.
Inguinal exam is notable for bilateral smooth masses. If genetic testing is obtained, what would be
the likely karyotype?

A. 46 X,Y
B. 46 X,X
C. 45 X,O
D. 47 XXY

Reproductive: Mullerian Agenesis vs. AIS

348
NBME Top Concepts ➤ Reproductive Mullerian Agenesis VS. AIS

Mullerian Agenesis vs. Androgen Insensitivity Syndrome

Reproductive: Mullerian Agenesis vs. AIS

349
uploaded by medbooksvn
NBME Top Concepts ➤ Reproductive Summary of Disorders of Sexual Development

Summary of Disorders of Sexual Development

Reproductive: Summary of Disorders of Sexual Development

350
NBME Top Concepts ➤ Reproductive Polycystic Ovarian Syndrome

Pathophysiology of PCOS

Creative Commons Attribution License 4.0


Creative Commons Attribution License 4.0

Management of PCOS

Reproductive: Polycystic Ovarian Syndrome

351
uploaded by medbooksvn
NBME Top Concepts ➤ Reproductive Polycystic Ovarian Syndrome

High Androgen States for the USMLE

Creative Commons Attribution License 4.0

NBME Style Question


A 22-year-old female presents to the emergency department with RUQ. She has a history of PCOS
controlled on OCPs. Ultrasound rules out cholelithiasis, however is notable for isoechoic, well
demarcated, in-homogenous collection of blood in the right lobe of the liver. In the ED, patient has
sudden tachycardia and hypotension. Which of the following pathophysiologic mechanisms most
likely underlies this patient’s acute symptoms?

A. Radio-opaque gallstone obstructing cystic duct


B. Benign liver tumor
C. Increased portal pressures due to hepatic fibrosis
D. Ovarian artery torsion leading to hypovolemic shock

Reproductive: Polycystic Ovarian Syndrome

352
NBME Top Concepts ➤ Reproductive Polycystic Ovarian Syndrome

Hepatocellular Adenoma OCP + RUQ/Shock

Creative Commons Attribution License 4.0

Reproductive: Polycystic Ovarian Syndrome

353
uploaded by medbooksvn
NBME Top Concepts ➤ Reproductive Ovarian Physiology

Study Tip: Anatomic Organization

Creative Commons Attribution License 4.0

Two Cell Hypothesis Reproductive Physiology Integration

Reproductive: Ovarian Physiology

354
NBME Top Concepts ➤ Reproductive Ovarian Tumors

Introduction to Ovarian Tumors

NBME Style Question


A 60-year-old female G0P0 presents with increased satiety. She feels that she has decreased her
appetite but continues to have increased waist circumference. Lipid panel and thyroid tests are un-
remarkable. There is adnexal fullness unilaterally. Which of the following markers is most likely
elevated?

A. CA-125
B. LDH
C. Inhibin
D. Testosterone
E. B-HcG

Reproductive: Ovarian Tumors

355
uploaded by medbooksvn
NBME Top Concepts ➤ Reproductive Ovarian Tumors

Types of Ovarian Tumors: Surface Ovarian Tumors

Creative Commons Attribution License 4.0

Types of Ovarian Tumors: Germ Cell Tumors

NBME Style Question


A 30-year-old female presents with progressive lower
abdominal pain. She is noted to have normal menses.
Ultrasound shows a heterogenous unilateral mass.
Doppler of ovarian vessels are normal. Patient
undergoes removal. Mass is notable of tri-laminar germ
disc derivatives. What is the likely diagnosis?

A. Ectopic Pregnancy
B. Endometriosis
C. Serous Papillary Ovarian Cancer
D. Ovarian Teratoma
Creative Commons Attribution License 4.0

Reproductive: Ovarian Tumors

356
NBME Top Concepts ➤ Reproductive Ovarian Tumors

Different Types of Germ Cell Tumors

Ovarian Germ Cell Tumors

Creative Commons Attribution License 4.0

Reproductive: Ovarian Tumors

357
uploaded by medbooksvn
NBME Top Concepts ➤ Reproductive Ovarian Tumors

Types of Ovarian Tumors: Stromal Tumors

NBME Style Question


A 49-year-old female presents with menstrual bleeding in-between her periods. She is otherwise
healthy. An ovarian mass is found on pelvic ultrasonography. It is also noted that the patient has an
increased endometrial stripe on ultrasound. She just completed her menses 3 days prior. Beta-hcg is
negative. What is the likely hormone which may be implicated in this pathology?

A. Estrogen
B. Progesterone
C. B-HCG
D. Testosterone

Different Types of Ovarian Stromal Tumors

Reproductive: Ovarian Tumors

358
NBME Top Concepts ➤ Reproductive Ovarian Tumors

Metastasis to Ovary

Mucinous Cell of Ovary

Creative Commons Attribution License 4.0

Reproductive: Ovarian Tumors

359
uploaded by medbooksvn
NBME Top Concepts ➤ Reproductive Repro Ligaments

NBME Style Question


A post-menopausal female is scheduled to undergo a resection of right ovary for a complex ovarian
cyst noted on trans-vaginal ultrasound. Prior to oophorectomy, which of the following anatomic
structures must be ligated to prevent hemorrhagic shock?

A. Utero-ovarian Ligament
B. Suspensory ligament of ovary
C. Round ligament of uterus
D. Cardinal Ligament

Anatomy Integration

Reproductive: Repro Ligaments

360
NBME Top Concepts ➤ Reproductive Uterine Disorders

Uterine Disorders

Uterine Pathology
A 24-year-old woman presents with pain and bleeding during menstruation. Her last three cycles
have had bad cramps and large amounts of blood. She states that her cycle has been irregular for
the last 6 months. She has dyspareunia.
What is the likely diagnosis?
• Endometriosis
• Presence of glands & stroma outside of the endometrial lining
• USMLE point:
• Watch for cyclical abdominal pain in a menstruating female.

Creative Commons Attribution License 4.0

Reproductive: Uterine Disorders

361
uploaded by medbooksvn
NBME Top Concepts ➤ Reproductive Uterine Disorders

Pathophysiology of Endometriosis

Endometriosis & Anatomic Locations

Creative Commons Attribution License 4.0

Reproductive: Uterine Disorders

362
NBME Top Concepts ➤ Reproductive Uterine Disorders

NBME Style Question


A 14-year-old girl presents to the emergency department for vaginal bleeding in between her cycles.
She is unsure of her menstrual cycles, and her mom states she may have “had a period 2 weeks
prior.” There is no trauma. There is no family history of bleeding disorders. Patient is otherwise
healthy. What is the likely cause of this patient’s complaints?

A. Endometrial atrophy.
B. Increased corpus luteum activity.
C. Increased spiral artery proliferation.
D. Increased FSH.

Anovulatory Bleeding

Reproductive: Uterine Disorders

363
uploaded by medbooksvn
NBME Top Concepts ➤ Reproductive Uterine Disorders

Unopposed Estrogen Risk Factors

General Pathology Integration

Reproductive: Uterine Disorders

364
NBME Top Concepts ➤ Reproductive Uterine Disorders

Endometrial Carcinoma

Papillary serous type characterized by serous glands with


papillary organization has a fibro-vascular core if that core
necrosis and calcifies, what is the histopathological finding?
• Psammoma Body

Creative Commons Attribution License 4.0

General Pathology Integration

How is a psammoma body described?


• Laminated, concentric spherules with
dystrophic calcification

Reproductive: Uterine Disorders

365
uploaded by medbooksvn
NBME Top Concepts ➤ Reproductive Cervical Disorders (HPV)

Pathophysiology of HPV causing Cervical Camcer

Creative Commons Attribution License 4.0

High Risk HPV Strains

Creative Commons Attribution License 4.0

Reproductive: Cervical disorders (HPV)

366
NBME Top Concepts ➤ Reproductive Answer Key

Disorders of Sexual Development


A 15-year-old boy presents with delayed growth spurt. He has had decreased hair growth in his
pubic region. Intelligence test is normal. Audiometry evaluation is normal. He is unable to
distinguish smells. Ophthalmic exam is normal. Which of the following mechanisms may explain
this patient’s disease?

A. Primary seminiferous tubule absence.


B. Genetic X,O Karyotype.
C. Absence of GnRH secretory neurons.
D. Rathke’s pouch dysfunction.
E. Failure of somatotroph development in the pituitary.

A medical school anatomy class is undergoing anatomy review in the cadaveric lab. On one of the
cadaveric specimen’s, it is noted that the patient has a bicuspid aortic valve. Which of the following
murmurs is consistent with this finding’s downstream pathology?

A. Diastolic murmur heard at the sternal border.


B. A systolic murmur with click heard at the apex.
C. Diastolic murmur heard at the R second ICS.
D. Systolic murmur heard at the R second ICS.

Testosterone Disorders
A newborn is evaluated in the delivery room. The baby’s delivery course was unremarkable.
Mother’s pregnancy was uncomplicated. Ambiguous genitalia and clitoromegaly are notable on
baby’s exam. Mother states that during the latter half of her pregnancy she noticed her voice deepen
and have dark coarse facial hair. Remaining maternal and baby exam is normal. Which of the
following mechanisms accounts for this infant’s pathology?

A. Activation of 21-Hydroxylase.
B. Deficiency in 17-Hydroxylase.
C. Placental Aromatase Deficiency.
D. Increased HMG-CoA Reductase Activity.

Mullerian Agenesis VS. AIS


A 16-year-old female presents with primary amenorrhea. She has had breast engorgement
however she has not had any pubic hair. On exam, she is noted to have a blind ended vaginal pouch.
Inguinal exam is notable for bilateral smooth masses. If genetic testing is obtained, what would be
the likely karyotype?

A. 46 X,Y
B. 46 X,X
C. 45 X,O
D. 47 XXY

Reproductive: Answer Key

367
uploaded by medbooksvn
NBME Top Concepts ➤ Reproductive Answer Key

Polycystic Ovarian Syndrome


A 22-year-old female presents to the emergency department with RUQ. She has a history of PCOS
controlled on OCPs. Ultrasound rules out cholelithiasis, however is notable for isoechoic, well
demarcated, in-homogenous collection of blood in the right lobe of the liver. In the ED, patient has
sudden tachycardia and hypotension. Which of the following pathophysiologic mechanisms most
likely underlies this patient’s acute symptoms?

A. Radio-opaque gallstone obstructing cystic duct


B. Benign liver tumor
C. Increased portal pressures due to hepatic fibrosis
D. Ovarian artery torsion leading to hypovolemic shock

Ovarian Tumors
A 60-year-old female G0P0 presents with increased satiety. She feels that she has decreased her
appetite but continues to have increased waist circumference. Lipid panel and thyroid tests are un-
remarkable. There is adnexal fullness unilaterally. Which of the following markers is most likely
elevated?

A. CA-125
B. LDH
C. Inhibin
D. Testosterone
E. B-HcG

A 30-year-old female presents with progressive lower abdominal pain. She is noted to have normal
menses. Ultrasound shows a heterogenous unilateral mass. Doppler of ovarian vessels are normal.
Patient undergoes removal. Mass is notable of tri-laminar germ disc derivatives. What is the likely
diagnosis?

A. Ectopic Pregnancy
B. Endometriosis
C. Serous Papillary Ovarian Cancer
D. Ovarian Teratoma

A 49-year-old female presents with menstrual bleeding in-between her periods. She is otherwise
healthy. An ovarian mass is found on pelvic ultrasonography. It is also noted that the patient has an
increased endometrial stripe on ultrasound. She just completed her menses 3 days prior. Beta-hcg is
negative. What is the likely hormone which may be implicated in this pathology?

A. Estrogen
B. Progesterone
C. B-HCG
D. Testosterone

Reproductive: Answer Key

368
NBME Top Concepts ➤ Reproductive Answer Key

Repro Ligaments

A post-menopausal female is scheduled to undergo a resection of right ovary for a complex ovarian
cyst noted on trans-vaginal ultrasound. Prior to oophorectomy, which of the following anatomic
structures must be ligated to prevent hemorrhagic shock?

A. Utero-ovarian Ligament
B. Suspensory ligament of ovary
C. Round ligament of uterus
D. Cardinal Ligament

Uterine Disorders
A 14-year-old girl presents to the emergency department for vaginal bleeding in between her cycles.
She is unsure of her menstrual cycles, and her mom states she may have “had a period 2 weeks
prior.” There is no trauma. There is no family history of bleeding disorders. Patient is otherwise
healthy. What is the likely cause of this patient’s complaints?

A. Endometrial atrophy.
B. Increased corpus luteum activity.
C. Increased spiral artery proliferation.
D. Increased FSH.

Reproductive: Answer Key

369
uploaded by medbooksvn
CHAPTER 17

Top NBME Concepts:


Musculoskeletal &
Rheumatology

370
NBME Top Concepts ➤ Musculoskeletal & Rheumatology NMJ Disorders

NBME Style Question


A patient presents with difficulty chewing and dysarthria. She notices that at the end of the day
her eyes droop. She attributes this to general tiredness. On exam, she is noted to have blurry vision
when asked to look to the left for a few minutes. Administration of AchE inhibitor improves her
symptoms. The mechanism of this disease is most like which pathology?

A. Atopic Dermatitis
B. Good Pasteur Syndrome
C. Rapidly Progressive Cresenteric Nephritis
D. Contact Dermatitis

Neuromuscular Junction Disorders

Musculoskeletal & Rheumatology: NMJ Disorders

371
uploaded by medbooksvn
NBME Top Concepts ➤ Musculoskeletal & Rheumatology NMJ Disorders

NBME Style Question


A 40-year-old male presents with weakness more prominent in the lower extremities. He also is
noted to have issues with incontinence and achieving erections. His muscle myography is notable
for an incremental increase in muscle contraction. Given this presentation, which of the following
studies may be abnormal in this patient?

A. CT scan of chest
B. MRI of brain
C. X-ray of vertebral spine
D. PET scan of lower extremities

USMLE Test-taking Strategy

Musculoskeletal & Rheumatology: NMJ Disorders

372
NBME Top Concepts ➤ Musculoskeletal & Rheumatology Bullous Diseases

Bullous Diseases

Immunofluorescence using IgG & C3

Creative Commons Attribution License 4.0

Musculoskeletal & Rheumatology: Bullous Diseases

373
uploaded by medbooksvn
NBME Top Concepts ➤ Musculoskeletal & Rheumatology Bullous Diseases

NBME Style Question


A 30-year-old male presents with bloody cough. He recently traveled to northern Africa however
he states that prior to his trip he had similar symptoms. He undergoes PFT testing which is notable
for increased DLCO. His labs are notable for an elevated creatinine and CRP. Which of the following
antibodies may also be positive on laboratory testing?

A. Topoisomerase I
B. Cardiolipin phospholipid
C. Double stranded DNA
D. Collagen Type IV

Hemoptysis & Hematuria for the USMLE

NBME Style Question


A 40-year-old female presents with new-onset rash. She has a history of diarrhea, gas, and weight
loss which she attributed to “irritable bowels.” Exam shows a vesicular rash in the extensor
distribution. She is scheduled to undergo endoscopy. Which of the following histopathological
findings may be present upon biopsy?

A. Anti-Insulin staining Ab
B. Crypt abscesses in the colon
C. Intra-intestinal epithelial lymphocytes
D. Hyperplastic villi with limited atrophy

Musculoskeletal & Rheumatology: Bullous Diseases

374
NBME Top Concepts ➤ Musculoskeletal & Rheumatology Hemoptysis & Hematuria

Celiac Disease for the USMLE

Creative Commons Attribution License 4.0

Hypersensitivities for the USMLE

Musculoskeletal & Rheumatology: Hemoptysis & Hematuria

375
uploaded by medbooksvn
NBME Top Concepts ➤ Musculoskeletal & Rheumatology Skin Cancers (with Neural Crest)

Test-Taking Strategy
In USMLE questions:
Look for location of rash (i.e. sun exposed regions)
Look for:
• Asymmetry
• Border irregularities (i.e. uneven)
• Color variegation
• Diameter (≥6 mm)
• Evolving (i.e. lesion is changing in size)
Creative Commons Attribution License 4.0

Neoplasia Integration

Creative Commons Attribution License 4.0

Musculoskeletal & Rheumatology: Skin Cancers (with Neural Crest)

376
NBME Top Concepts ➤ Musculoskeletal & Rheumatology Skin Cancers (with Neural Crest)

Skin Cancers for the USMLE

NBME Style Question


A farmer presents with several scaly lesions on the forehead. He is noted to have a rough grainy
texture on palpation of these lesions. He undergoes biopsy which is consistent with neoplasia as his
report reveals “atypical keratinocytes confined to the basement membrane.” The patient is most
likely to be at risk for which of the following conditions?

A. Kaposi Sarcoma
B. Melanoma
C. Dermatofibroma
D. Psoriasis
E. Squamous Cell Carcinoma

Actinic Keratoses can be a precursor to SCC

Creative Commons Attribution License 4.0

Creative Commons Attribution License 4.0

Musculoskeletal & Rheumatology: Skin Cancers (with Neural Crest)

377
uploaded by medbooksvn
NBME Top Concepts ➤ Musculoskeletal & Rheumatology Skin Cancers (with Neural Crest)

Squamous Cell Carcinoma of the Skin

Creative Commons Attribution License 4.0

Skin Cancer
What is the most common malignant skin tumor?
• Basal cell carcinoma
What anatomical lesion does it preferentially affect?
• Upper lip and inner canthus of eye in sun exposed
areas

A 59-year-old male presents with a raised papule on the


upper surface of his lip. The sides of the crater-like
lesion has multiple spiraled blood vessels. What may
histological biopsy of this lesion reveal?
Creative Commons Attribution License 4.0

• Cords of basophilic-staining dysplastic basal cells


in infiltrating the underlying dermis.

Basal Cell Carcinoma of the Skin

Creative Commons Attribution License 4.0

Musculoskeletal & Rheumatology: Skin Cancers (with Neural Crest)

378
NBME Top Concepts ➤ Musculoskeletal & Rheumatology Skin Cancers (with Neural Crest)

NBME Style Question


A 4-year-old male presents to the dermatology clinic for recurrent skin rash. Despite being inside
for majority of the year, the child is noted to have ”a sunburn like rash” consistently that waxes and
wanes. Mother recalls that her uncle had similar sun-burn issues & died from melanoma. Physical
exam shows skin atrophy. On the neck and hands there are 3 nevi which have been enlarging rapidly.
Which of the following is the likely mechanism behind the pathology?

A. DNA mismatch repair


B. Cyclin dysregulation
C. Nucleotide excision repair
D. Repair of DNA crosslinks

NBME Style Question


A patient presents with a skin rash that has been having irregular borders. A biopsy is taken and
is notable for BRAF mutation. Which of the following is the most likely diagnosis?

A. Melanoma
B. Metastatic GBM
C. Sezary Syndrome
D. HTLV-1
E. Squamous Cell Carcinoma

Melanoma

Creative Commons Attribution License 4.0

Musculoskeletal & Rheumatology: Skin Cancers (with Neural Crest)

379
uploaded by medbooksvn
NBME Top Concepts ➤ Musculoskeletal & Rheumatology Skin Cancers (with Neural Crest)

USMLE Integration
A 27-year-old Caucasian female presents with weight loss and weakness. She feels dizzy and
lightheaded. Physical exam reveals several areas of her skin including her elbows and knees are
more tan than other areas. The cells which are stimulated are derived from which embryological
layer?
Neural crest melanocytes
• Adrenal insufficiency (primary) POMC is a precursor for three hormones:
• ACTH, MSH, beta-endorphin.

POMC is a precursor for MSH

Neural Crest Derivative

Musculoskeletal & Rheumatology: Skin Cancers (with Neural Crest)

380
NBME Top Concepts ➤ Musculoskeletal & Rheumatology Skin Cancers (with Neural Crest)

Melanin
Biochemistry review: What amino acid is the precursor to melanin?
• Tyrosine
• Phenylalanine is a precursor to tyrosine.
• What is the pathology related to the lack of an enzyme which converts phenylalanine to
tyrosine?
• PKU – patients present pale and fair, blue eyes and characteristic, musty body odor

Musculoskeletal & Rheumatology: Skin Cancers (with Neural Crest)

381
uploaded by medbooksvn
NBME Top Concepts ➤ Musculoskeletal & Rheumatology RA vs OA

Integration of Joint Disorders

Big-Picture:
• Rheumatoid Arthritis “systemic symptoms”
• Osteoarthritis “wear & tear”

Creative Commons Attribution License 4.0


Creative Commons Attribution License 4.0

Rheumatoid Arthritis vs. Osteoarthritis

Musculoskeletal & Rheumatology: RA vs OA

382
NBME Top Concepts ➤ Musculoskeletal & Rheumatology RA vs OA

Rheumatoid Arthritis
A 44-year-old woman comes for swollen fingers in the past 6 months. She says that she has some
stiffness in the morning of her extremities that takes over an hour to resolve. What physical exam
findings may you find on UE exam?
• Swelling of the PIP and MCP joints Rheumatoid Arthritis
• Systemic symptoms
• Associated with which genetic marker?
• HLA-DR4.

Pathophysiology
Synovium lined by infiltrating lymphocytes edema
pannus formation granulation tissue
General pathology integration:
• 3 characteristics of granulation tissue (III):
• Blood vessels, fibroblasts, and
myofibroblasts
This contraction is the mechanism for ulnar
deviation.
Creative Commons Attribution License 4.0

Rheumatoid Arthritis (Concept: Neutrophil Recruitment in Joint)

Musculoskeletal & Rheumatology: RA vs OA

383
uploaded by medbooksvn
NBME Top Concepts ➤ Musculoskeletal & Rheumatology RA vs OA

Rheumatoid Arthritis:(Systemic Integration)

Creative Commons Attribution License 4.0

Rheumatoid Arthritis (Concept: Neutrophil Recruitment in Joint)


“RA + blunted costophrenic angles”
• Pleural effusions
“RA + intubation + spinal cord paralysis?”
• Atlantoaxial instability
“RA + CP worse with lying down”
• Pericarditis
“RA + Apple Green BRF”
• AA amyloid
“RA + growth on back of knee”
• Baker’s cyst
“RA + anemia”
• Anemia of chronic disease hepcidin inhibiting iron utilization.
• ADD SPLENOMEGALY and neutropenia?
• Felty syndrome.

NBME Style Question


A 48-year-old female presents for joint pain. She is noted to have an elevated ESR and CRP. X-ray
of her joints are notable for joint space narrowing in the MCP. An auto-antibody, IgM, is detected.
Which of the following substances is most likely the target of this auto—Ab?

A. Fc portion of IgG
B. Sheep erythrocytes
C. Sphingomyelin
D. ds-DNA
E. U1-RNP

Musculoskeletal & Rheumatology: RA vs OA

384
NBME Top Concepts ➤ Musculoskeletal & Rheumatology Seronegative Spondyloarthritis

Seronegative Spondyloarthropathies

NBME Style Question


A 28-year-old make presents with dull low back pain. He is noted to have this back-pain in the
morning. There is no trauma. Physical exam is significant for decreased extension of spine while
standing. PFTs are abnormal. Which of the following features is most likely present in this patient’s
work-up?

A. Class II HLA positivity.


B. M-protein molecular mimicry.
C. IgM to IgG.
D. Vertebral fusion on CXR.
E. Peri-vertebral abscesses with lytic lesions.

Psoriatic Arthritis
Patient presents with a scaly rash that bleeds
when disrupted. He has joint pain. What is the
classic radiographic finding?
• Patient presents with a scaly rash. He has
arthritis. What is the classic radiographic
finding?
• Pencil-in-cup Psoriatic Arthritis
Creative Commons Attribution License 4.0

Musculoskeletal & Rheumatology: Seronegative Spondyloarthritis

385
uploaded by medbooksvn
NBME Top Concepts ➤ Musculoskeletal & Rheumatology Seronegative Spondyloarthritis

Ankylosing Spondylitis
A 22-year-old man presents to office with lower back pain,
and stiffness. He has burning in his eyes. X-ray of the hip
demonstrates abnormalities in the central skeleton. What is
the likely pulmonary complication associated with this
condition?
• This usually can affect chest wall movement
• Pulmonary association:
• Restrictive lung disease
• Ophthalmologic association?
• Anterior uveitis
• X-ray finding is a bamboo spine. Creative Commons Attribution License 4.0

Reactive Arthritis
A young man who works in a daycare had bloody diarrhea
one week prior and now presents with a red eye. He feels pain
during urination and says his ankles hurt when he runs after
the toddlers. What is the likely diagnosis?
• Reactive Arthritis after a Shigella infection
• “Can’t See, Can’t Pee, Can’t Climb a Tree.”
• Classical presentation: asymmetric joint arthritis
• What is the morphology of the most common
organisms?
• Shigella gram negative rod
• Chlamydia gram negative coccobacllis
• Skin finding?
• Kertoderma blemorrhagicum which affects palms and
soles. Creative Commons Attribution License 4.0

Musculoskeletal & Rheumatology: Seronegative Spondyloarthritis

386
NBME Top Concepts ➤ Musculoskeletal & Rheumatology Seronegative Spondyloarthritis

Palms & Soles Integration

Summary

Musculoskeletal & Rheumatology: Seronegative Spondyloarthritis

387
uploaded by medbooksvn
NBME Top Concepts ➤ Musculoskeletal & Rheumatology Lupus

NBME Style Question


A 30-year-old female presents with SOB. She is diagnosed with a unilateral pleural effusion via
CXR. Her further laboratory studies are notable for anemia with high ferritin. Her 3 month follow up
shows increased Cr and UA positive for blood and protein. Which of the following mechanisms most
likely explains her renal dysfunction?

A. Auto-immune attack of podocytes


B. Light chain amyloid deposition
C. Deposition of immune complexes
D. Sub-epithelial humps secondary to chronic streptococcal infection

Pathophysiology of Lupus

Musculoskeletal & Rheumatology: Lupus

388
NBME Top Concepts ➤ Musculoskeletal & Rheumatology Lupus

Lupus
A 22-year-old woman with Lupus has a f/u exam. She is tachypneic and tachycardic and imaging
shows deep venous thrombosis. 2 yrs ago she delivered a female stillborn at 23wks. Platelet: 250K
(normal). PT normal, increased PTT. What is the likely mechanism behind this patient’s current
presentation?
• Antibodies directed against phospholipids APLS
• Can also have false positive syphilis
• PTT increased however still you are hypercoagulable.

Diagnosis:
• Sensitive but not specific?
• ANA
• Specific markers?
• Anti-dsDNA, and Anti-Smith antibodies.
• Anti-Smith directed towards snRNPs
• Anti-dsDNA = flairs, and poor prognosis
• Low complement.

• Patient on medication for WPW now has lupus like syndrome. Diagnosis?
• Drug induced lupus. Lab abnormalities?
• Anti-histone Ab.

Musculoskeletal & Rheumatology: Lupus

389
uploaded by medbooksvn
NBME Top Concepts ➤ Musculoskeletal & Rheumatology Lupus

Systemic Manifestations of Lupus


CNS:
• Fever
• Cognitive dysfunction
• Seizures
Cardiovascular:
• Pericarditis
• Libman sacks endocarditis
Respiratory:
• Pleurisy and effusions
Renal:
• Membranous Glomerulonephropathy (“spike and dome”)
• Diffuse Proliferative Glomerulonephritis (“capillary wire loops”)
• Hypocomplementemia.
Heme:
• Anemia of chronic disease
• Thrombocytopenia (thrombolic events from anti-phospholipid antibodies)
• Leukopenia
MSK:
• Migratory arthritis
Skin:
• Butterfly rash and photosensitivity
Rheum:
• ANA (sensitive)
• Anti-dsDNA and anti-Sm (specific).

Musculoskeletal & Rheumatology: Lupus

390
NBME Top Concepts ➤ Musculoskeletal & Rheumatology Anti Body Rapid Fire (USMLE)

Multi-System Involvement + Ab in Labs

NBME Style Question


A 31-year-old female presents with general malaise, myalgias, low grade fever. She is noted to
have painful extremities especially in the winter-time as she states her fingers “turn blue.” She was
recently hospitalized for a myositis flare. On physical exam, patient has a malar rash on the nose.
The patient is also noted to have joint tenderness in the MCP and DIP joints. Lab testing is positive
for anti-nuclear antibodies. Which of the following anti-bodies may also be present given this clinical
presentation?

A. C-ANCA
B. Rheumatoid Factor Inhibitor
C. Anti-U1-RNP Ab
D. P-ANCA
E. Anti-Smith Ab

Musculoskeletal & Rheumatology: Anti Body Rapid Fire (USMLE)

391
uploaded by medbooksvn
NBME Top Concepts ➤ Musculoskeletal & Rheumatology Anti Body Rapid Fire (USMLE)

Antibody and USMLE Significance

Musculoskeletal & Rheumatology: Anti Body Rapid Fire (USMLE)

392
NBME Top Concepts ➤ Musculoskeletal & Rheumatology Anti Body Rapid Fire (USMLE)

Antibody and USMLE Significance

Musculoskeletal & Rheumatology: Anti Body Rapid Fire (USMLE)

393
uploaded by medbooksvn
NBME Top Concepts ➤ Musculoskeletal & Rheumatology Answer Key

NMJ Disorders
A patient presents with difficulty chewing and dysarthria. She notices that at the end of the day
her eyes droop. She attributes this to general tiredness. On exam, she is noted to have blurry vision
when asked to look to the left for a few minutes. Administration of AchE inhibitor improves her
symptoms. The mechanism of this disease is most like which pathology?

A. Atopic Dermatitis
B. Good Pasteur Syndrome
C. Rapidly Progressive Cresenteric Nephritis
D. Contact Dermatitis

A 40-year-old male presents with weakness more prominent in the lower extremities. He also is
noted to have issues with incontinence and achieving erections. His muscle myography is notable
for an incremental increase in muscle contraction. Given this presentation, which of the following
studies may be abnormal in this patient?

A. CT scan of chest
B. MRI of brain
C. X-ray of vertebral spine
D. PET scan of lower extremities

Bullous Disease

A 30-year-old male presents with bloody cough. He recently traveled to northern Africa however
he states that prior to his trip he had similar symptoms. He undergoes PFT testing which is notable
for increased DLCO. His labs are notable for an elevated creatinine and CRP. Which of the following
antibodies may also be positive on laboratory testing?

A. Topoisomerase I
B. Cardiolipin phospholipid
C. Double stranded DNA
D. Collagen Type IV

A 40-year-old female presents with new-onset rash. She has a history of diarrhea, gas, and weight
loss which she attributed to “irritable bowels.” Exam shows a vesicular rash in the extensor
distribution. She is scheduled to undergo endoscopy. Which of the following histopathological
findings may be present upon biopsy?

A. Anti-Insulin staining Ab
B. Crypt abscesses in the colon
C. Intra-intestinal epithelial lymphocytes
D. Hyperplastic villi with limited atrophy

Musculoskeletal & Rheumatology: Answer Key

394
NBME Top Concepts ➤ Musculoskeletal & Rheumatology Answer Key

Skin Cancers (With Neural Crest)


A farmer presents with several scaly lesions on the forehead. He is noted to have a rough grainy
texture on palpation of these lesions. He undergoes biopsy which is consistent with neoplasia as his
report reveals “atypical keratinocytes confined to the basement membrane.” The patient is most
likely to be at risk for which of the following conditions?

A. Kaposi Sarcoma
B. Melanoma
C. Dermatofibroma
D. Psoriasis
E. Squamous Cell Carcinoma

A 4-year-old male presents to the dermatology clinic for recurrent skin rash. Despite being inside
for majority of the year, the child is noted to have ”a sunburn like rash” consistently that waxes and
wanes. Mother recalls that her uncle had similar sun-burn issues & died from melanoma. Physical
exam shows skin atrophy. On the neck and hands there are 3 nevi which have been enlarging rapidly.
Which of the following is the likely mechanism behind the pathology?

A. DNA mismatch repair


B. Cyclin dysregulation
C. Nucleotide excision repair
D. Repair of DNA crosslinks

A patient presents with a skin rash that has been having irregular borders. A biopsy is taken and
is notable for BRAF mutation. Which of the following is the most likely diagnosis?

A. Melanoma
B. Metastatic GBM
C. Sezary Syndrome
D. HTLV-1
E. Squamous Cell Carcinoma

RA vs OA
A 48-year-old female presents for joint pain. She is noted to have an elevated ESR and CRP. X-ray
of her joints are notable for joint space narrowing in the MCP. An auto-antibody, IgM, is detected.
Which of the following substances is most likely the target of this auto—Ab?

A. Fc portion of IgG
B. Sheep erythrocytes
C. Sphingomyelin
D. ds-DNA
E. U1-RNP

Musculoskeletal & Rheumatology: Answer Key

395
uploaded by medbooksvn
NBME Top Concepts ➤ Musculoskeletal & Rheumatology Answer Key

Seronegative Spondyloarthritis
A 28-year-old make presents with dull low back pain. He is noted to have this back-pain in the
morning. There is no trauma. Physical exam is significant for decreased extension of spine while
standing. PFTs are abnormal. Which of the following features is most likely present in this patient’s
work-up?

A. Class II HLA positivity.


B. M-protein molecular mimicry.
C. IgM to IgG.
D. Vertebral fusion on CXR.
E. Peri-vertebral abscesses with lytic lesions.

Lupus
A 30-year-old female presents with SOB. She is diagnosed with a unilateral pleural effusion via
CXR. Her further laboratory studies are notable for anemia with high ferritin. Her 3 month follow up
shows increased Cr and UA positive for blood and protein. Which of the following mechanisms most
likely explains her renal dysfunction?

A. Auto-immune attack of podocytes


B. Light chain amyloid deposition
C. Deposition of immune complexes
D. Sub-epithelial humps secondary to chronic streptococcal infection

Anti Body Rapid Fire (USMLE)


A 31-year-old female presents with general malaise, myalgias, low grade fever. She is noted to
have painful extremities especially in the winter-time as she states her fingers “turn blue.” She was
recently hospitalized for a myositis flare. On physical exam, patient has a malar rash on the nose.
The patient is also noted to have joint tenderness in the MCP and DIP joints. Lab testing is positive
for anti-nuclear antibodies. Which of the following anti-bodies may also be present given this clinical
presentation?

A. C-ANCA
B. Rheumatoid Factor Inhibitor
C. Anti-U1-RNP Ab
D. P-ANCA
E. Anti-Smith Ab

Musculoskeletal & Rheumatology: Answer Key

396
CHAPTER 18

Top NBME Concepts:


Psychiatry

397
uploaded by medbooksvn
NBME Top Concepts ➤ Psychiatry Mood Disorders

NBME Style Question


A 50-year-old female presents with sad mood. She is irritable and states she feels worthless and
helpless for the last 2 weeks. She just broke up with her husband a month ago. She finds that at
work it is tough to concentrate. She has lost weight unintentionally. She did not show up to work
today because she “doesn’t have the motivation anymore.” Her vital signs are stable. What is the
likely diagnosis?

A. Major Depressive Disorder


B. Dysthmia
C. Adjustment Disorder
D. Normal Grief

Test Taking Strategy


For Psychiatry Questions on the USMLE:
• Isolate functional impairments in ADLs
• Pay attention to time period for dx.
• Think medical causes of psychiatry disorders.

Mood Disorders

Psychiatry: Mood Disorders

398
NBME Top Concepts ➤ Psychiatry Bipolar + Li Pharm

NBME Style Question


A 17-year-old male presents to the adolescent clinic. He was in a car accident 2 months ago and
witnessed his mother pass away. He states that he wakes up really early and thinks about his
mother option. He feels very overwhelmed and says “it’s tough to go to school.” When asked about
his mom, he tearfully states that she was an amazing woman. What is the most likely diagnosis?

A. PTSD
B. Complex bereavement disorder
C. Normal Grief
D. Major Depressive Episode

Teasing out Grief from Depression on the USMLE

Teasing out Grief from Depression on the USMLE


A 60-year-old male presents to the emergency department intoxicated. He was found to be near a
bridge and having a firearm pointed to his head before law enforcement stopped him. His hx is
notable for MDD and recent hospitalization for a prior attempt at suicide. On exam he says, “my
brother died by hanging himself and I feel worthless as well”…”I am alone and life is not worth living.”
What is the most important risk factor which poses the highest risk for completion?
• Prior attempt
• Other risk factors:
• Psychiatric disorders
• Firearm possession
• Family member passing by suicide
•M>F
• Poor social support

Psychiatry: Bipolar + Li Pharm

399
uploaded by medbooksvn
NBME Top Concepts ➤ Psychiatry Bipolar + Li Pharm

Key Depression Variants for the USMLE

NBME Style Question


A 40-year-old male presents with feeling unhappy. He says that he has been so burnt-out at work.
He left his previous job as he felt like everything was a chore. His fiancé states that he has little
energy throughout the day. He has a history of smoking marijuana as a teenager. When asked how
long he has felt these symptoms, he states “idk, Dr., it’s been years.” His toxicology screen is
negative. What is the likely diagnosis?

A. Adjustment Disorder
B. Cyclothymia
C. Avoidant personality disorder
D. Dysthmia

Medical Causes of Depression

Psychiatry: Bipolar + Li Pharm

400
NBME Top Concepts ➤ Psychiatry Bipolar + Li Pharm

NBME Style Question


A 30-year-old male presents to the county physician. He is brought in by police for medical
examination. He was arrested last week after being found at Walmart assaulting customers. He
states that he is ”a knight in shining armor” and says that he does not need sleep. When asked about
his mood, he tells a long story about his childhood and fascination with swords. Tox screen is
negative What is the likely diagnosis?

A. Brief psychotic disorder


B. Bipolar 1 with psychotic features
C. Schizotypal
D. Hypomania

Understanding Hypomania
A 42-year-old woman presents due to fatigue and weight
gain over the past month. She says she has been struggling
with depression for a long time, but that it has gotten worse
since her husband was diagnosed with cancer. She also
says that she previously had an episode of decreased need
for sleep, irritability, and increased goal-directed activity
which lasted about a week, but it never significantly
impaired her work or home life. What is the likely diagnosis?

NBME Style Question


A 40-year-old female with CHF presents with tremor. She has a wobbly gait on exam. On review of
medications, the physician notes a history of bipolar well treated on a mood stabilizer. She was
started on diuretic therapy for CHF on her last visit two weeks ago. Which of the following drug
interactions likely contributed to her tremor?

A. Li & HCTZ
B. Li & Amiloride
C. Li & Metoprolol
D. Li & Clonidine

Psychiatry: Bipolar + Li Pharm

401
uploaded by medbooksvn
NBME Top Concepts ➤ Psychiatry Bipolar + Li Pharm

Lithium has a Narrow Therapeutic Index

Pharmacology Integration

Psychiatry: Bipolar + Li Pharm

402
NBME Top Concepts ➤ Psychiatry Bipolar + Li Pharm

Long Term Bipolar


A CEO presents with sleep issues. Over past two years he has felt unmotivated and has filed for
bankruptcy twice as he has made poor purchases and investments in the past 2-3 years. He is sad
and states that he only has slept 4-5 hours at night for many months to focus on work. What is the
likely diagnosis?
• Cyclothymia
• Symptoms for 2 years and are never absent for two months

Long term depression is to dysthymia whereas long term bipolar is to cyclothymia.

Psychiatry: Bipolar + Li Pharm

403
uploaded by medbooksvn
NBME Top Concepts ➤ Psychiatry Psychotic Disorders

Teasing Out Schizo-Diagnoses on the USMLE

Personality Disorders: Schizoid and schizotypal

Psychiatry: Psychotic Disorders

404
NBME Top Concepts ➤ Psychiatry Psychotic Disorders

NBME Style Question


A 30-year-old male is found at a park urinating in a fountain. He states that he must hide from the
CIA as they know of his radioactive lab. He used to live this his girlfriend but over the past year, he
lives on his friends’ couch, broke up with his gf, and quit his job. When questioned he has a flat
affect and says “I don’t care.” What is the likely diagnosis?

A. Schizophreniform
B. Bipolar 1
C. Depression with Psychotic Features
D. Schizophrenia

Triad of Psychosis for the USMLE

Timing for Schizo Disorders

Psychiatry: Psychotic Disorders

405
uploaded by medbooksvn
NBME Top Concepts ➤ Psychiatry Psychotic Disorders

NBME Style Question


A patient is newly diagnosed with psychosis. She is started on an anti-psychotic. The patient is at
follow-up three weeks later and states that it has been “challenging to have sex…I just don’t feel
interested.” Exam is notable for increased glandular nodularity of breast with intermittent bilateral
expulsion of white discharge. What is the likely pathway which is affected?

A. Arcuate fasciculus
B. Nigrostriatal
C. Mesolimbic
D. Tubuloinfundibular

USMLE Neuroanatomy Integration

NBME Style Question


A 30-year-old male is found at a park urinating in a fountain. He states that he must hide from the
CIA as they know of his radioactive lab. He used to live with his girlfriend but over the past year he
broke up with her. He is noted to have 10 psych hospital admissions for depression and was
discharged 2 weeks ago. He says that since d/c he doesn’t feel depressed and wants to end the
interview because the CIA will be here to kidnap him. What is the likely diagnosis?

A. Schizoaffective
B. Bipolar 1
C. Depression with Psychotic Features
D. Schizophrenia

Psychiatry: Psychotic Disorders

406
NBME Top Concepts ➤ Psychiatry Psychotic Disorders

USMLE Significance

Delusional Disorder
A 25-year-old woman believes she is an editor of a famous fashion magazine. She is irritated how
is unable to get employment as she states she is the best dressed at work. Her sister states that she
works as a waitress and otherwise has no abnormalities on exam.
• What is the time period for diagnosis?
• ≥ 1 month
•Isolated delusions no other psychotic symptoms + not marked impairment

Psychiatry: Psychotic Disorders

407
uploaded by medbooksvn
NBME Top Concepts ➤ Psychiatry Eating Disorders

NBME Style Question


A 17-year-old female is admitted to the hospital due to weight loss and syncope. NG tube is
placed and feeds are initiated. On day two of admission, the patient reports shortness of breath and
has recurrent episodes of ventricular tachycardia. She has tachycardia and hypotension. Labs are
notable for low phos, low Mg, and low K. Which of the following hormones best explains the
patient’s deterioration?

A. T4
B. Cortisol
C. Aldosterone
D. Insulin

Re-feeding Syndrome (Concept: Electrolyte Shifts End Organ)

Psychiatry: Psychotic Disorders

408
NBME Top Concepts ➤ Psychiatry Eating Disorders

Difference between Anorexia Nervosa and Bulemia

Acid Base Integration

Psychiatry: Psychotic Disorders

409
uploaded by medbooksvn
NBME Top Concepts ➤ Psychiatry Eating Disorders

NBME Style Question


A 40-year-old male was recently started on a medication for depression. Six weeks later, he has
been having issues with erectile dysfunction. He is considering stopping the medication due to this
and requests alternative treatment. What is the most appropriate selection for this patient?

A. Trazodone
B. Impipramine
C. Citalopram
D. Buproprion

Medications for the USMLE which decrease seizure threshold

Psychiatry: Psychotic Disorders

410
NBME Top Concepts ➤ Psychiatry Drugs of Abuse (Toxicology)

NBME Style Question


A patient presents after being brought in by his roommate for weird behavior. He was totally fine
two days ago in class. His roommate noticed that he was yelling and was constantly locking and
unlocking the door last night. He is noted to have hypertension, tachycardia, and dilated pupils. His
roommate states the patient has not slept well due to exams. What is the likely dx?

A. Substance induced psychosis


B. Delusional Disorder
C. Brief Psychotic Disorder
D. Adjustment disorder with psychotic features

Uppers vs. Downers

Introduction to Drugs of Abuse

UPPERS

DOWNERS

Psychiatry: Drugs of Abuse (Toxicology)

411
uploaded by medbooksvn
NBME Top Concepts ➤ Psychiatry Drugs of Abuse (Toxicology)

USMLE Vignettes
A patient who is crying, has slurred speech, blacks out, has wrist drop, AST > ALT?
• Alcohol intoxication
A patient with history of cirrhosis undergoes a gastric bypass surgery. On day two of
hospitalization, patient states that he is seeing the nurses have funny hats and sees bugs on the
walls. What is most severe complication of this likely diagnosis?
• Delirium Tremens Alcohol withdrawal
• USMLE: Confusion, agitation, fever, tachycardia give a benzodiazepine.

USMLE Vignettes
A homeless male with respiratory depression +
pupillary constriction + decreased GI motility and
constipation. 85% on RA. What is the likely cause of
the hypoxemia?
• Hypoventilation Opioid intoxication
• Violent yawning + increased secretions.
• Signs of opioid withdrawal
• What is the likely antidote for opioid intoxication?
• Naloxone (shorter duration of action).

NBME Style Question


A 40-year-old female presents to the ED with a fracture. She states that she ran into a car in the
parking lot because she has super-human strength. She is hypertensive and tachycardic. Her exam
is notable for vertical nystagmus. She refuses to give a urine exam. What is the likely MOA of the
drug of abuse seen in this patient?

A. Mu agonism
B. NMDA antagonism
C. GABA activation
D. D2 antagonism

NBME Style Question


At a high-school health fair, a nurse notices strange behavior in a 16 yo male. She notices that he
seems “slow” and when she asks his name he giggles and says “where’s the vending machine.” His
urine tox is pending. Which of the following exam features may be most seen in this patient?

A. Conjunctival injection
B. Nasal perforations
C. Miosis
D. Bradycardia

Psychiatry: Drugs of Abuse (Toxicology)

412
NBME Top Concepts ➤ Psychiatry Drugs of Abuse (Toxicology)

Understanding Drugs of Abuse

Salient Features of Drugs of Abuse

Creative Commons Attribution License 4.0

Psychiatry: Drugs of Abuse (Toxicology)

413
uploaded by medbooksvn
NBME Top Concepts ➤ Psychiatry Answer Key

Mood Disorders
A 50-year-old female presents with sad mood. She is irritable and states she feels worthless and
helpless for the last 2 weeks. She just broke up with her husband a month ago. She finds that at
work it is tough to concentrate. She has lost weight unintentionally. She did not show up to work
today because she “doesn’t have the motivation anymore.” Her vital signs are stable. What is the
likely diagnosis?

A. Major Depressive Disorder


B. Dysthmia
C. Adjustment Disorder
D. Normal Grief

Bipolar + Li Pharm
A 17-year-old male presents to the adolescent clinic. He was in a car accident 2 months ago and
witnessed his mother pass away. He states that he wakes up really early and thinks about his
mother option. He feels very overwhelmed and says “it’s tough to go to school.” When asked about
his mom, he tearfully states that she was an amazing woman. What is the most likely diagnosis?

A. PTSD
B. Complex bereavement disorder
C. Normal Grief
D. Major Depressive Episode

A 40-year-old male presents with feeling unhappy. He says that he has been so burnt-out at work.
He left his previous job as he felt like everything was a chore. His fiancé states that he has little
energy throughout the day. He has a history of smoking marijuana as a teenager. When asked how
long he has felt these symptoms, he states “idk, Dr., it’s been years.” His toxicology screen is
negative. What is the likely diagnosis?

A. Adjustment Disorder
B. Cyclothymia
C. Avoidant personality disorder
D. Dysthmia
A 30-year-old male presents to the county physician. He is brought in by police for medical
examination. He was arrested last week after being found at Walmart assaulting customers. He
states that he is ”a knight in shining armor” and says that he does not need sleep. When asked about
his mood, he tells a long story about his childhood and fascination with swords. Tox screen is
negative What is the likely diagnosis?

A. Brief psychotic disorder


B. Bipolar 1 with psychotic features
C. Schizotypal
D. Hypomania

Psychiatry: Answer Key

414
NBME Top Concepts ➤ Psychiatry Answer Key

Bipolar + Li Pharm
A 40-year-old female with CHF presents with tremor. She has a wobbly gait on exam. On review of
medications, the physician notes a history of bipolar well treated on a mood stabilizer. She was
started on diuretic therapy for CHF on her last visit two weeks ago. Which of the following drug
interactions likely contributed to her tremor?

A. Li & HCTZ
B. Li & Amiloride
C. Li & Metoprolol
D. Li & Clonidine

Psychotic Disorders
A 30-year-old male is found at a park urinating in a fountain. He states that he must hide from the
CIA as they know of his radioactive lab. He used to live this his girlfriend but over the past year, he
lives on his friends’ couch, broke up with his gf, and quit his job. When questioned he has a flat
affect and says “I don’t care.” What is the likely diagnosis?

A. Schizophreniform
B. Bipolar 1
C. Depression with Psychotic Features
D. Schizophrenia

A patient is newly diagnosed with psychosis. She is started on an anti-psychotic. The patient is at
follow-up three weeks later and states that it has been “challenging to have sex…I just don’t feel
interested.” Exam is notable for increased glandular nodularity of breast with intermittent bilateral
expulsion of white discharge. What is the likely pathway which is affected?

A. Arcuate fasciculus
B. Nigrostriatal
C. Mesolimbic
D. Tubuloinfundibular

A 30-year-old male is found at a park urinating in a fountain. He states that he must hide from the
CIA as they know of his radioactive lab. He used to live with his girlfriend but over the past year he
broke up with her. He is noted to have 10 psych hospital admissions for depression and was
discharged 2 weeks ago. He says that since d/c he doesn’t feel depressed and wants to end the
interview because the CIA will be here to kidnap him. What is the likely diagnosis?

A. Schizoaffective
B. Bipolar 1
C. Depression with Psychotic Features
D. Schizophrenia

Psychiatry: Answer Key

415
uploaded by medbooksvn
NBME Top Concepts ➤ Psychiatry Answer Key

Eating Disorders
A 17-year-old female is admitted to the hospital due to weight loss and syncope. NG tube is
placed and feeds are initiated. On day two of admission, the patient reports shortness of breath and
has recurrent episodes of ventricular tachycardia. She has tachycardia and hypotension. Labs are
notable for low phos, low Mg, and low K. Which of the following hormones best explains the
patient’s deterioration?

A. T4
B. Cortisol
C. Aldosterone
D. Insulin

A 40-year-old male was recently started on a medication for depression. Six weeks later, he has
been having issues with erectile dysfunction. He is considering stopping the medication due to this
and requests alternative treatment. What is the most appropriate selection for this patient?

A. Trazodone
B. Impipramine
C. Citalopram
D. Buproprion

Drugs of Abuse (Toxicology)


A patient presents after being brought in by his roommate for weird behavior. He was totally fine
two days ago in class. His roommate noticed that he was yelling and was constantly locking and
unlocking the door last night. He is noted to have hypertension, tachycardia, and dilated pupils. His
roommate states the patient has not slept well due to exams. What is the likely dx?

A. Substance induced psychosis


B. Delusional Disorder
C. Brief Psychotic Disorder
D. Adjustment disorder with psychotic features

A 40-year-old female presents to the ED with a fracture. She states that she ran into a car in the
parking lot because she has super-human strength. She is hypertensive and tachycardic. Her exam
is notable for vertical nystagmus. She refuses to give a urine exam. What is the likely MOA of the
drug of abuse seen in this patient?

A. Mu agonism
B. NMDA antagonism
C. GABA activation
D. D2 antagonism

Psychiatry: Answer Key

416
NBME Top Concepts ➤ Psychiatry Answer Key

Drugs of Abuse (Toxicology)


At a high-school health fair, a nurse notices strange behavior in a 16 yo male. She notices that he
seems “slow” and when she asks his name he giggles and says “where’s the vending machine.” His
urine tox is pending. Which of the following exam features may be most seen in this patient?

A. Conjunctival injection
B. Nasal perforations
C. Miosis
D. Bradycardia

Psychiatry: Answer Key

417
uploaded by medbooksvn
CHAPTER 19

Top NBME Concepts:


Oncology

418
NBME Top Concepts ➤ Oncology Cardiac Oncology

NBME Style Question


A high school athlete presents with a 2-week history of shortness of breath on exertion. He has
dizziness and vital signs show bradycardia with normal blood pressure. A diastolic murmur is heard
that abruptly stops. The echo shows a pedunculated heterogeneous mass in the left atrium. Which of
the following accurately describes this diagnosis?

A. Myxoma
B. Teratoma
C. Rhabdomyosarcoma
D. Thrombus
E. Lipoma
F. Astrocytoma

Primary Cardiac Tumors

Oncology: Cardiac Oncology

419
uploaded by medbooksvn
NBME Top Concepts ➤ Oncology Cardiac Oncology

Tuberous Sclerosis – Multi-System Involvement

Oncology: Cardiac Oncology

420
NBME Top Concepts ➤ Oncology Endocrine Oncology

Thyroid Carcinoma

Creative Commons Attribution License 4.0

Thyroid Cancer for the USMLE Step 1

Oncology: Endocrine Oncology

421
uploaded by medbooksvn
NBME Top Concepts ➤ Oncology Endocrine Oncology

Psammoma Bodies for the USMLE

How is a psammoma body described?


Laminated, concentric spherules with dystrophic calcification

Creative Commons Attribution License 4.0

NBME Style Question


A 50-year-old male has a lump in his neck. His exam is notable for a mass in the right thyroid lobe.
FNA reveals thyroid carcinoma. He undergoes thyroid resection. Three days post-operatively he is
found to be “whispering in a raspy voice.” Which of the following complications likely occurred with
this patient?

A. Injury to the accessory nerve


B. Injury to a branch of the vagus nerve
C. Injury to the phrenic nerve
D. Injury to the hypoglossal nerve.

Embryology Integration

Oncology: Endocrine Oncology

422
NBME Top Concepts ➤ Oncology Gastrointestinal Oncology

Colon Cancer Overview

What are risk factors which increase the risk for carcinoma from a polyp?
Size ≥2 cm
Sessile growth (versus pedunculated)
Villous histology (versus tubular)

Creative Commons Attribution License 4.0

Adenoma Carcinoma Sequence

Oncology: Gastrointestinal Oncology

423
uploaded by medbooksvn
NBME Top Concepts ➤ Oncology Gastrointestinal Oncology

NBME Style Question


A 60-year-old male presents with fatigue and passing out. He is noted to have dark stools and his
wife states he has become increasingly “thin” over the past few months. He undergoes stool testing
which is positive for heme. Further workup reveals adenocarcinoma. Which of the following gene
mutations is likely responsible for the transformation of a colonic adenoma to adenocarcinoma?

A. n-MYC
B. c-MYC
C. COX
D. p53

Associated Colonic Conditions

Oncology: Gastrointestinal Oncology

424
NBME Top Concepts ➤ Oncology Gastrointestinal Oncology

Pathophysiology of Colonic Carcinoma for the USMLE

Colonic Carcinoma
Most common site of mets?
Liver
Pt presents with a fever and a new-onset murmur. Blood
culture grows S.bovis (gallolyticus). What is the next best
step in management?
Colonscopy
Bovis gram+ cocci chains; grows in bile, not salt
A 40-year-old male is seen in clinic for health-maintenance.
His brother had colon cancer at age 50. Should patient get
colonoscopy?
Yes, first degree screen at age 40 or 10 years prior to
presentation (whichever comes first) Creative Commons Attribution License 4.0

Patient is diagnosed with colonic cancer what tumor


marker may be helpful to track progression?
CEA (seen also in COPD, pancreatitis)

Oncology: Gastrointestinal Oncology

425
uploaded by medbooksvn
NBME Top Concepts ➤ Oncology Gastrointestinal Oncology

Esophageal Cancer for the USMLE Step 1

Creative Commons Attribution License 4.0

Neoplasia – Precursor Lesions to Cancer for the USMLE

NBME Style Question


A 60-year-old male with a history of hypertension and dyslipidemia presents with chest pain. He
also has difficulty swallowing. He states that his chest pain is worse when lying down. There is no
relation to activity. CXR and abdominal CXR reveal a “heterogenous air-filled collection behind the
retrocardiac border on the left side.” What is the likely pathophysiology behind the diagnosis?

A. Laxity of the diaphragmatic-lower esophageal region


B. Congenital abnormality of the diaphragm
C. Acute trauma leading to colonic contents in the thorax
D. Lack of abdominal malrotation

Oncology: Gastrointestinal Oncology

426
NBME Top Concepts ➤ Oncology Gastrointestinal Oncology

Esophageal Carcinoma

Oncology: Gastrointestinal Oncology

427
uploaded by medbooksvn
NBME Top Concepts ➤ Oncology Lymphoma & Vascular Tumors

Lymphomas on the USMLE Compare & Contrast

Non-Hodgkin's Lymphoma

NBME Style Question


A patient was recently diagnosed with HIV. He has been non-compliant with his therapy. He is
noted to have headaches. MRI of the brain reveals a “solitary mass with no calcification in the
frontotemporal region.” Biopsy reveals a tumor with T-cell lineage. Which of the following genomes
may be present in this patient’s biopsy?

A. Bartonella hensale
B. Ebstein Barr Virus
C. HHV-6
D. CMV

Oncology: Lymphoma & Vascular Tumors

428
NBME Top Concepts ➤ Oncology Lymphoma & Vascular Tumors

Non-Hodgkin's Lymphoma

NBME Style Question


A patient with Sjogren syndrome has a rapidly enlarging parotid mass. Exam shows a unilateral
heterogeneous mass that is not tender to palpation obscuring the R mandibular region. Parotid
excision is likely to reveal which of the following tumor derivatives?

A. Malignant squamous cells with keratin pearls


B. Anaplastic round blue cells surrounding salivary glands
C. S-100 neuro-ectodermal cells surrounding salivary glands
D. Germinal centers with distorted lymph node architecture

Oncology: Lymphoma & Vascular Tumors

429
uploaded by medbooksvn
NBME Top Concepts ➤ Oncology Lymphoma & Vascular Tumors

Vascular Tumors

Creative Commons Attribution License 4.0

Creative Commons Attribution License 4.0

NBME Style Question


A patient is diagnosed with a highly vascularized tumor
surrounding normal cellular epithelium. Increased capillary
proliferation is seen on microscopic pathology. An increase in which
of the following cellular ligands may be expected in this patient?

A. EGF
B. FGF
C. PDRF
D. IGF-1

Oncology: Lymphoma & Vascular Tumors

430
NBME Top Concepts ➤ Oncology Renal Oncology

Genitourinary Cancer for the USMLE Step 1

Renal Cell Carcinoma


The patient presents with hematuria, and flank pain. He has a fever and hyperglycemia. He has had
some weight gain and has felt fatigued. Exam shows truncal obesity and atrophy of extremities. What
is the likely mechanism?
ACTH paraneoplastic syndrome RCC
Other USMLE presentations:

Oncology: Renal Oncology

431
uploaded by medbooksvn
NBME Top Concepts ➤ Oncology Renal Oncology

Renal Cell Carcinoma

What is the common histopathological presentation of RCC?


Polygonal cells with high glycogen clear cell type which
is most common
Where in the nephron do these “clear cells” arise?
Proximal tubule
What is the difference between a renal cell carcinoma and
renal oncocytoma?
Renal Oncocytoma arises from collecting ducts, benign
tumor, and has large eosinophilic cells with increased
mitochondria.

Creative Commons Attribution License 4.0

NBME Style Question


A mother brings her son in due to abdominal distension. His mother states that her son has been
having normal bowel movements. She noticed while bathing him last night that he had increased
abdominal fullness “more on the left side.” CT scan reveals a homogenous mass at the inferior pole
of the kidney that does not cross the midline. If this oncological lesion was associated with a genetic
syndrome, which of the following additional findings may be present?

A. Episcleritis
B. Aniridia
C. Uveitis
D. Glaucoma

Oncology: Renal Oncology

432
NBME Top Concepts ➤ Oncology Renal Oncology

Nephroblastoma
A 15-month-old male brought in by mother after she
noticed he had decreased appetite and a mass felt while
she was bathing him. He is evaluated and found to be
hypertensive. What is going to be the pathological hallmark
of this diagnosis?
Blastema primitive cell which makes glomerulus and
tubules related to Wilm’s Tumor
• Genetic hallmark of Wilm’s Tumor?
• Loss of WT1 gene (tumor suppressor)
Creative Commons Attribution License 4.0 2-month-old who presents with hepatosplenomegaly and
tongue hypertrophy. Exam shows increased bulk on the left
side compared to the right. What is the likely diagnosis?
Beckwith Weidemen Syndrome (Wilms, hemi-
hypertrophy, organomegaly)
Wilm’s Tumor, Aniridia, GU malformations, Retardation.

Renal Embryology

What pathologies are related to an abnormal


interaction between ureteric bud and
metanephros?
Unilateral renal agenesis (failure of ureteric
bud to form no kidney or ureter)
Multicystic dysplastic kidney (failure of
ureteric bud to induce mesenchyme
unilateral, nonfunctional, cystic kidney).

Oncology: Renal Oncology

433
uploaded by medbooksvn
NBME Top Concepts ➤ Oncology Respiratory Oncology

USMLE Test Taking Strategy: Lung Tumors

Creative Commons Attribution License 4.0

USMLE Questions will presentations related to:


Constitutional Symptoms
Paraneoplastic Syndromes:
• Physiology tie-ins.

Lung Cancer for the USMLE Step 1

Oncology: Respiratory Oncology

434
NBME Top Concepts ➤ Oncology Respiratory Oncology

Adenocarcinoma

Small Cell

Oncology: Respiratory Oncology

435
uploaded by medbooksvn
NBME Top Concepts ➤ Oncology Respiratory Oncology

Squamous Cell

Creative Commons Attribution License 4.0

Oncology: Respiratory Oncology

436
NBME Top Concepts ➤ Oncology Neuro Oncology

Neuro-oncology for the USMLE Step 1

Organization of Brain Tumors for the USMLE

Oncology: Neuro Oncology

437
uploaded by medbooksvn
NBME Top Concepts ➤ Oncology Neuro Oncology

Brain Tumors

Oncology: Neuro Oncology

438
NBME Top Concepts ➤ Oncology Breast Oncology

Risk Factors for Breast Cancer on the USMLE

NBME Style Question


A 60-year-old female presents concerned about a right breast mass that she felt in the shower. She
notes that there is associated dimpling. There is no redness, fluctuance to the breast. There is no
family history of breast cancer. There is prominent lymphadenopathy in the right upper quadrant of
her breast. She is found to have an irregular immobile mass on palpation. Which of the following
affected structures may be most contributing to her nipple dimpling?

A. Axillary lymph node


B. Suspensory ligament
C. Lactiferous ducts
D. Mammary artery

Oncology: Breast Oncology

439
uploaded by medbooksvn
NBME Top Concepts ➤ Oncology Breast Oncology

Breast Cancer Summary for the USMLE Step 1

Creative Commons Attribution License 4.0

NBME Style Question


A patient presents with unilateral breast redness. She is 3 weeks postpartum and had a breast
mass which was being worked up prior to pregnancy. She has completed a course of dicloxicillin with
no relief of her breast redness. She is currently not breastfeeding. Ultrasound of breast shows no
abscess. There are hard lymph nodes in the axilla. Which of the following pathophysiologic
mechanisms may be contributing to the underlying pathology?

A. Lymphatic duct obstruction


B. Fibrotic suspensory ligament of breast
C. Infection of the superficial epidermis by MSSA
D. Deep vein thrombosis of the L brachial vein

Breast Cancer Staining


Questions typically refer to histo-chemical staining
ER
PR
HER2/neu (gene amplification, TK receptor)

The patient is found to have an ER/PR+ breast cancer. What is the characteristic of the receptor this
cancer expresses?
Intra-nuclear receptor. Drug?
• Tamoxifen.

Patient is started on a chemo-therapy treatment for an immuno-histochemical staining positive breast


CA. She is found to have an echo with a low ejection fraction. What is the likely mechanism of the
chemo-therapy treatment?
Her2Neu receptor inhibitor Trastuzumab Cardiotoxic.

Oncology: Breast Oncology

440
NBME Top Concepts ➤ Oncology Neoplasia (Get Path Integration)

Grading vs. Staging for the USMLE

Metastasis

Oncology: Neoplasia (Get Path Integration)

441
uploaded by medbooksvn
NBME Top Concepts ➤ Oncology Answer Key

Cardiac Oncology
A high school athlete presents with a 2-week history of shortness of breath on exertion. He has
dizziness and vital signs show bradycardia with normal blood pressure. A diastolic murmur is heard
that abruptly stops. The echo shows a pedunculated heterogeneous mass in the left atrium. Which of
the following accurately describes this diagnosis?

A. Myxoma
B. Teratoma
C. Rhabdomyosarcoma
D. Thrombus
E. Lipoma
F. Astrocytoma

Endocrine Oncology
A 50-year-old male has a lump in his neck. His exam is notable for a mass in the right thyroid lobe.
FNA reveals thyroid carcinoma. He undergoes thyroid resection. Three days post-operatively he is
found to be “whispering in a raspy voice.” Which of the following complications likely occurred with
this patient?

A. Injury to the accessory nerve


B. Injury to a branch of the vagus nerve
C. Injury to the phrenic nerve
D. Injury to the hypoglossal nerve.

Gastrointestinal Oncology
A 60-year-old male presents with fatigue and passing out. He is noted to have dark stools and his
wife states he has become increasingly “thin” over the past few months. He undergoes stool testing
which is positive for heme. Further workup reveals adenocarcinoma. Which of the following gene
mutations is likely responsible for the transformation of a colonic adenoma to adenocarcinoma?

A. n-MYC
B. c-MYC
C. COX
D. p53
A 60-year-old male with a history of hypertension and dyslipidemia presents with chest pain. He
also has difficulty swallowing. He states that his chest pain is worse when lying down. There is no
relation to activity. CXR and abdominal CXR reveal a “heterogenous air-filled collection behind the
retrocardiac border on the left side.” What is the likely pathophysiology behind the diagnosis?

A. Laxity of the diaphragmatic-lower esophageal region


B. Congenital abnormality of the diaphragm
C. Acute trauma leading to colonic contents in the thorax
D. Lack of abdominal malrotation

Oncology: Answer Key

442
NBME Top Concepts ➤ Oncology Answer Key

Lymphomas and Vascular Tumors


A patient was recently diagnosed with HIV. He has been non-compliant with his therapy. He is
noted to have headaches. MRI of the brain reveals a “solitary mass with no calcification in the
frontotemporal region.” Biopsy reveals a tumor with T-cell lineage. Which of the following genomes
may be present in this patient’s biopsy?

A. Bartonella hensale
B. Ebstein Barr Virus
C. HHV-6
D. CMV

A patient with Sjogren syndrome has a rapidly enlarging parotid mass. Exam shows a unilateral
heterogeneous mass that is not tender to palpation obscuring the R mandibular region. Parotid
excision is likely to reveal which of the following tumor derivatives?

A. Malignant squamous cells with keratin pearls


B. Anaplastic round blue cells surrounding salivary glands
C. S-100 neuro-ectodermal cells surrounding salivary glands
D. Germinal centers with distorted lymph node architecture

A patient is diagnosed with a highly vascularized tumor surrounding normal cellular epithelium.
Increased capillary proliferation is seen on microscopic pathology. An increase in which of the
following cellular ligands may be expected in this patient?

A. EGF
B. FGF
C. PDRF
D. IGF-1

Renal Oncology
A mother brings her son in due to abdominal distension. His mother states that her son has been
having normal bowel movements. She noticed while bathing him last night that he had increased
abdominal fullness “more on the left side.” CT scan reveals a homogenous mass at the inferior pole
of the kidney that does not cross the midline. If this oncological lesion was associated with a genetic
syndrome, which of the following additional findings may be present?

A. Episcleritis
B. Aniridia
C. Uveitis
D. Glaucoma

Oncology: Answer Key

443
uploaded by medbooksvn
NBME Top Concepts ➤ Oncology Answer Key

Breast Oncology
A 60-year-old female presents concerned about a right breast mass that she felt in the shower. She
notes that there is associated dimpling. There is no redness, fluctuance to the breast. There is no
family history of breast cancer. There is prominent lymphadenopathy in the right upper quadrant of
her breast. She is found to have an irregular immobile mass on palpation. Which of the following
affected structures may be most contributing to her nipple dimpling?

A. Axillary lymph node


B. Suspensory ligament
C. Lactiferous ducts
D. Mammary artery

A patient presents with unilateral breast redness. She is 3 weeks postpartum and had a breast
mass which was being worked up prior to pregnancy. She has completed a course of dicloxicillin with
no relief of her breast redness. She is currently not breastfeeding. Ultrasound of breast shows no
abscess. There are hard lymph nodes in the axilla. Which of the following pathophysiologic
mechanisms may be contributing to the underlying pathology?

A. Lymphatic duct obstruction


B. Fibrotic suspensory ligament of breast
C. Infection of the superficial epidermis by MSSA
D. Deep vein thrombosis of the L brachial vein

Oncology: Answer Key

444
CHAPTER 20

Top NBME Concepts:


Dermatology

445
uploaded by medbooksvn
NBME Top Concepts ➤ Dermatology Approach to Urticaria

NBME Style Question


A 40-year-old female presents with sore throat and a skin rash. The patient was recently started on
oral amoxicillin. The skin rash is pruritic and notable for raised papules which spare the palms and
soles. The patient has a remote history of asthma. What is the likely microscopic finding which will
be seen on biopsy of this rash?

A. Ab to hemidesmosomes
B. Dermal transudative edema
C. Ab to desmosomes
D. Epidermal spongiosis

Pathophysiology of Urticaria

Approach to Urticaria on the USMLE Step 1


Identify triggers in exam questions:
• Drugs, food, recent prodrome of illness
• Physical Exam:
• Raised, palpable well-circumscribed pruritic
papule or plaque
• Transient

Dermatology: Approach to Urticaria

446
NBME Top Concepts ➤ Dermatology Approach to Urticaria

Hives for the USMLE Sstep 1

NBME Style Question


A 50-year-old farmer presents with a rough rash on the scalp. The patient noted this rash a few
months ago however now the rash has spread and is itchy. The rash is notable for scaly papules
with mild erythema at the base. What is the likely diagnosis?

A. Atopic dermatitis
B. Actinic keratosis
C. Pityriasis rosea
D. Psoriasis
E. Seborrheic Keratosis

USMLE Dermatology Test Taking Tip


Pay attention to the Physical Exam for dermatology questions it will describe the basic
terminology of the rash.

Big Picture:
• What are flat lesions?
• Macules or Patch
• What are raised lesions?
• Papules or Plaque
• What are fluid containing lesions?
• Vesicles or Bullae

Dermatology: Approach to Urticaria

447
uploaded by medbooksvn
NBME Top Concepts ➤ Dermatology Dermatology Nomenclature

USMLE Dermatology (Basic Terms)

NBME Style Question


A 50-year-old female presents with a skin rash after vacation. The patient is noted to have tan skin
with multiple hypopigmented patches & macules on the extremities. She undergoes KOH
preparation which is notable for short, cigar shaped appearance hyphae. Which of the following is
the most likely cause of this lesion?

A. Blastomycosis dermatiditis
B. Malassezia globosa
C. Sporothrix schenckii
D. Histoplasmosa capsulatum

Dermatology: Dermatology Nomenclature

448
NBME Top Concepts ➤ Dermatology Dermatology Nomenclature

Pityriasis Versicolor

Creative Commons Attribution License 4.0

Infection of stratum corneum

Dermatology: Dermatology Nomenclature

449
uploaded by medbooksvn
NBME Top Concepts ➤ Dermatology Plaques

Plaques
A 34-year-old woman with chronic skin rash. Shows well
demarcated coral-colored plaques with silvery scales over
the scalp, elbows and knees. You remove the scales and
see pinpoint bleeding. What is the clinical physical exam
feature just observed?
• Auspitz sign
• Scaling lesion that has HLA relationship +
environmental factors. Creative Commons Attribution License 4.0

Microscopic Pathology of Psoriasis on the USMLE

Dermatology: Plaques

450
NBME Top Concepts ➤ Dermatology Viral & Bacterial Exanthems

Microbiology & Dermatology Integration for the USMLE

Viral Exanthems

A child in daycare presents with a rash on the trunk and


extremities. Rash is notable for multiple well demarcated,
flesh-colored papules with central umbilication. An
infectious etiology is suspected. What is the likely
morphology of the agent behind this diagnosis?
• Pox virus
• Enveloped, double stranded DNA virus
Creative Commons Attribution License 4.0

• Largest DNA virus replicates in the cytoplasm


• What is the mechanism of transmission?
• Skin to skin
• Fomite.

Dermatology: Viral & Bacterial Exanthems

451
uploaded by medbooksvn
NBME Top Concepts ➤ Dermatology Viral & Bacterial Exanthems

Rubella, Rubeola, Roseola

NBME Style Question


A newborn baby is in the nursery. The patient is noted to have white pupils and a harsh murmur
heard over the L side of precordium. There is no hepatosplenomegaly. Newborn screening indicates
a failed hearing exam. What is the likely intervention which may have prevented this pathology?

A. Maternal live attenuated vaccine before pregnancy


B. Maternal polysaccharide booster before pregnancy
C. Maternal screening of glucose at 20 weeks gestation
D. Newborn screening of mucopolysaccharidosis

Dermatology: Viral & Bacterial Exanthems

452
NBME Top Concepts ➤ Dermatology Rubella

Rubella - High Yield Feature

Creative Commons Attribution License 4.0

Viral Exanthems
An unimmunized child presents with fever to 102 F, runny nose, and eye discharge. Patient on
exam has a benign lung exam however is coughing frequently. White spots on oral mucosa are
present. What is the likely diagnosis?
• Measles also known as rubeola.
• What is the morphology of this virus?
• RNA virus, orthomyxovirus (ss –, linear, non-segmented RNA virus)
• What are the oral lesions on the buccal mucosa?
• Koplik spots.

Oral Mucosal Lesions (Viral)

Creative Commons Attribution License 4.0

Creative Commons Attribution License 4.0

Dermatology: Rubella

453
uploaded by medbooksvn
NBME Top Concepts ➤ Dermatology Measles

Measles (Rubeola)

NBME Style Question


A child presents to the PICU with a generalized seizure. During this episode vitals are notable for
a temperature of 40.2℃. Acetaminophen is administered. A few days after admission the patient is
noted to have a maculopapular rash on her trunk. What is the likely diagnosis?

A. HSV-1
B. HSV-2
C. Human Herpes-6
D. Scarlet fever
E. Kawasaki Disease

Dermatology: Measles

454
NBME Top Concepts ➤ Dermatology Roseola

Roseola (HHV-6)

Creative Commons Attribution License 4.0

• Febrile seizures:
• Vital Signs (>38℃)
• Generalized shaking of upper & lower extremities
• Fever then rash on trunk which spreads to extremities.

Dermatology: Roseola

455
uploaded by medbooksvn
NBME Top Concepts ➤ Dermatology Erythema Infectiosum

Erythema Infectiosum
A pregnant woman presents with joint pain and low-grade fevers. She has joint pain that is
transient in upper and lower extremities. History is notable for febrile contacts throughout the child
care center which she is employed. What is the likely morphology of the virus behind her symptoms?
• Parvo-virus B19
• Single-Stranded DNA virus
• Watch for hydrops fetalis on pre-natal ultrasound
She now presents for follow-up as she continues to have chronic arthritis. On routine blood work,
she is found to be anemic and thrombocytopenic with low reticulocyte count. What is the likely
diagnosis?
• Aplastic anemia.

Erythema Infectiosum Slapped Cheek Rash

Creative Commons Attribution License 4.0

NBME Style Question


A 22-year-old female presents to the OBGYN clinic with chief complaint of pruritis and vulvar pain.
She is noted to have a prodrome of fever and on exam tender vesicle lesions covering the mons
pubis and labia. What is the likely sequela of this patient’s disease?

A. Resolution with treatment


B. Recurrence of genital lesions
C. Squamous cell vulvar carcinoma
D. Co-infection with HPV 6, 11
E. Inguinal abscesses

Dermatology: Erythema Infectiosum

456
NBME Top Concepts ➤ Dermatology Erythema Infectiosum

NBME Style Question


A patient is noted to be altered, confused and febrile. A continuous EEG is placed to detect
seizures. The patient is diagnosed with temporal lobe epilepsy. The patient continues to be febrile
throughout admission in the hospital. Which of the following infectious etiologies is most likely
related to this patient’s presentation?

A. Toxoplasma
B. Streptococcus PNA
C. Neisseria Meningitis
D. Neurocysticercosis
E. Herpes Simplex Virus

Dermatology: Erythema Infectiosum

457
uploaded by medbooksvn
NBME Top Concepts ➤ Dermatology Systemic Manifestations of Herpes

Systemic Manifestations of herpes for the USMLE

NBME Style Question


A patient is noted to have a vesicular rash involving the T10 dermatome. There is crusting near the
umbilicus. The rash does not cross the midline. What is the likely complication which may be seen in
this patient on 5 month follow up?

A. Polyarthralgia
B. Persistent pain in the dermatomal region
C. Gross hematuria
D. Recurrent disseminated pan-sclerosing encephalitis

Dermatology: Systemic Manifestations of Herpes

458
NBME Top Concepts ➤ Dermatology Varicella Infections on USMLE

Varicella Infections on the USMLE

Microbiology & Dermatology Integration for the USMLE

Dermatology: Varicella Infections on USMLE

459
uploaded by medbooksvn
NBME Top Concepts ➤ Dermatology Erysipelas vs Cellulitis

Bacterial Skin Infections for the USMLE Step 1

Dermatology: Erysipelas vs Cellulitis

460
NBME Top Concepts ➤ Dermatology Staph and Strep Infections

Staphylococcal & Streptococcal Infections

A child presents with URI symptoms and a peri-oral rash.


Exam of the face is notable for a rash with crusted secretions
and a yellow-tan appearance. What is the likely diagnosis?
• Non-bullous impetigo
• S. aureus (gm +, catalase & coagulase +)
• Group A Streptococcus (gm +, catalase -, β hemolytic)
• Watch for superinfection with herpes or cola colored urine
(PSGN).

Creative Commons Attribution License 4.0

Staphylococcal Infections
An infant presents with fever and
hypotension diffuse blisters in the inguinal
creases are seen which disappear upon slight
pressure. What is the likely physical exam sign
seen described in this patient?
• Nikolsky sign (+)
• Pathophysiology of Staph Scalded Skin
Syndrome
• Epidermolytic toxins A+B bind to
desmoglein 1 of desmosomes à skin splits
Creative Commons Attribution License 4.0
at the stratum granulosum.

Staph Scalded Skin Syndrome

Dermatology: Staph and Strep Infections

461
uploaded by medbooksvn
NBME Top Concepts ➤ Dermatology Nikolsky Sign Manifestations for the USMLE

High Yield PE Feature

Creative Commons Attribution License 4.0

Nikolsky Sign for the USMLE Step 1

NBME Style Question


A couple is on their honeymoon. Upon return home, the newly wed wife notices a pruritic,
papulopustular rash on the trunk. Two days later, the husband notice the same rash. What is the
likely organism which may be behind the presentation?

A. Staph aureus
B. GAS
C. Pseudomonas Aeruginosa
D. Pasturella Multicoda

Dermatology: Nikolsky Sign Manifestations for the USMLE

462
NBME Top Concepts ➤ Dermatology Neonatal Rashes

Neonatal Rashes
An infant presents for routine examination. Exam is
notable for a red 2.5 cm lesion on scalp with fine
telangiectasias surrounding the lesion. The patient is
afebrile & is painless. What is the next best response to
family?
• “This is a benign rash; rash will grow before it
spontaneously involutes.”
Creative Commons Attribution License 4.0

Neonatal Skin Lesions

Dermatology: Neonatal Rashes

463
uploaded by medbooksvn
NBME Top Concepts ➤ Dermatology Precursor Lesions to Neoplasia

NBME Style Question


A 50-year-old farmer presents with a rough rash on the scalp. The patient noted this rash a few
months ago however now the rash has spread and is itchy. The rash is notable for scaly papules
with mild erythema at the base. What is the likely complication of this disease presentation?

A. Basal cell carcinoma


B. Super-imposed Staph infection
C. Kaposi sarcoma
D. Squamous cell carcinoma
E. Seborrheic Keratosis

Actinic Keratosis Squamous Cell Carcinoma

Seborrheic Keratosis Gi Adenocarcinoma

Dermatology: Precursor Lesions to Neoplasia

464
NBME Top Concepts ➤ Dermatology Summary

Rash on Palms and Soles

Creative Commons Attribution License 4.0

Erythema

Creative Commons Attribution License 4.0

Dermatology: Summary

465
uploaded by medbooksvn
NBME Top Concepts ➤ Dermatology Answer Key

Approach to Urticaria
A 40-year-old female presents with sore throat and a skin rash. The patient was recently started on
oral amoxicillin. The skin rash is pruritic and notable for raised papules which spare the palms and
soles. The patient has a remote history of asthma. What is the likely microscopic finding which will
be seen on biopsy of this rash?

A. Ab to hemidesmosomes
B. Dermal transudative edema
C. Ab to desmosomes
D. Epidermal spongiosis

A 50-year-old farmer presents with a rough rash on the scalp. The patient noted this rash a few
months ago however now the rash has spread and is itchy. The rash is notable for scaly papules
with mild erythema at the base. What is the likely diagnosis?

A. Atopic dermatitis
B. Actinic keratosis
C. Pityriasis rosea
D. Psoriasis
E. Seborrheic Keratosis

Dermatology Nomenclature
A 50-year-old female presents with a skin rash after vacation. The patient is noted to have tan skin
with multiple hypopigmented patches & macules on the extremities. She undergoes KOH
preparation which is notable for short, cigar shaped appearance hyphae. Which of the following is
the most likely cause of this lesion?

A. Blastomycosis dermatiditis
B. Malassezia globosa
C. Sporothrix schenckii
D. Histoplasmosa capsulatum

Viral & Bacterial Exanthems


A newborn baby is in the nursery. The patient is noted to have white pupils and a harsh murmur
heard over the L side of precordium. There is no hepatosplenomegaly. Newborn screening indicates
a failed hearing exam. What is the likely intervention which may have prevented this pathology?

A. Maternal live attenuated vaccine before pregnancy


B. Maternal polysaccharide booster before pregnancy
C. Maternal screening of glucose at 20 weeks gestation
D. Newborn screening of mucopolysaccharidosis

Dermatology: Answer Key

466
NBME Top Concepts ➤ Dermatology Answer Key

Measles
A child presents to the PICU with a generalized seizure. During this episode vitals are notable for
a temperature of 40.2℃. Acetaminophen is administered. A few days after admission the patient is
noted to have a maculopapular rash on her trunk. What is the likely diagnosis?

A. HSV-1
B. HSV-2
C. Human Herpes-6
D. Scarlet fever
E. Kawasaki Disease

Erythema Infectiosum
A 22-year-old female presents to the OBGYN clinic with chief complaint of pruritis and vulvar pain.
She is noted to have a prodrome of fever and on exam tender vesicle lesions covering the mons
pubis and labia. What is the likely sequela of this patient’s disease?

A. Resolution with treatment


B. Recurrence of genital lesions
C. Squamous cell vulvar carcinoma
D. Co-infection with HPV 6, 11
E. Inguinal abscesses
A patient is noted to be altered, confused and febrile. A continuous EEG is placed to detect
seizures. The patient is diagnosed with temporal lobe epilepsy. The patient continues to be febrile
throughout admission in the hospital. Which of the following infectious etiologies is most likely
related to this patient’s presentation?

A. Toxoplasma
B. Streptococcus PNA
C. Neisseria Meningitis
D. Neurocysticercosis
E. Herpes Simplex Virus

Systemic Manifestations of Herpes


A patient is noted to have a vesicular rash involving the T10 dermatome. There is crusting near the
umbilicus. The rash does not cross the midline. What is the likely complication which may be seen in
this patient on 5 month follow up?

A. Polyarthralgia
B. Persistent pain in the dermatomal region
C. Gross hematuria
D. Recurrent disseminated pan-sclerosing encephalitis

Dermatology: Answer Key

467
uploaded by medbooksvn
NBME Top Concepts ➤ Dermatology Answer Key

Nikolsky Sign Manifestations for the USMLE


A couple is on their honeymoon. Upon return home, the newly wed wife notices a pruritic,
papulopustular rash on the trunk. Two days later, the husband notice the same rash. What is the
likely organism which may be behind the presentation?

A. Staph aureus
B. GAS
C. Pseudomonas Aeruginosa
D. Pasturella Multicoda

Precursor Lesions to Neoplasia


A 50-year-old farmer presents with a rough rash on the scalp. The patient noted this rash a few
months ago however now the rash has spread and is itchy. The rash is notable for scaly papules
with mild erythema at the base. What is the likely complication of this disease presentation?

A. Basal cell carcinoma


B. Super-imposed Staph infection
C. Kaposi sarcoma
D. Squamous cell carcinoma
E. Seborrheic Keratosis

Dermatology: Answer Key

468

You might also like